Krok I 2004

Posted: May 5, 2012 in Krok I
Tags:

MINISTRY OF PUBLIC HEALTH OF UKRAINE

Board of education and science of MPH of Ukrai

 

Test items for licensing examination

Krokl

MEDICINE

2004

 

List of abbreviations

 

A/G          Albumin/globulin ratio

A-ANON   Alcoholics anonymous

ACT         Abdominal computed tomography

ADP         Adenosine diphosphate

ALT          Alanin aminotranspherase

AMP         Adenosine monophosphate

AP            Action potential

ARF          Acute renal failure

AST          Aspartat aminotranspherase

ATP          Adenosine triphosphate

BP            Blood pressure

bpm         Beats per minute

C.I.             Color Index

CBC         Complete blood count

CHF           Chronic heart failure

CT             Computer tomography

DIC            Disseminated intravascular coagualtion

DCC          Doctoral controlling committee

DM-2         Non-Insulin dependent diabetes mellitus

DTP           Anti diphtheria-tetanus vaccine

ECG          Electrocardiogram

ESR           Erythrocyte sedimentation rate

FC             Function class

FAD           Flavin adenine dinucleotide

FADH2      Flavin adenine dinucleotide restored

FEGDS      Fibro-esphago-gastro-duodenoscopy

FMNH2      Flavin mononucleotide restored

GIT            Gastrointestinal tract

GMP          Guanosine monophosphate

Hb             Hemoglobin

HbA1c      Glycosylated hemoglobin

Hct            Hematocrit

HIV            Human immunodeficiency virus

 

HR Heart rate
IDDM Insulin dependent diabetes mellitus
IFA Immunofluorescence assay
IHD Ischemic heart disease
IU International unit
LDH Lactate dehydrogenase
MSEC Medical and sanitary expert committee
NAD Nicotine amide adenine dinucleotide
NADPH Nicotine amide adenine dinucleotide
  phosphate restored
NIDDM Non-Insulin dependent diabetes mellitus
PAC Polyunsaturated aromatic carbohydrates
PAS Periodic acid & sniff reaction
pC02 C02 partial pressure
p02 CO2 partial pressure
pm Per minute
Ps Pulse rate
r roentgen
RBC Red blood count
RDHA Reverse direct hemagglutination assay
Rh Rhesus
(R)CFT Reiter’s complement fixation test
RIHA Reverse indirect hemagglutination assay
RNA Ribonucleic acid
RR Respiratory rate
S1 Heart sound 1
S2 Heart sound 2
TU Tuberculin unit
U Unit
USI Ultrasound investigation
V/f Vision field
WBC White blood count
X-ray Roentgenogram

 

E9U02

1

 

 

 

1. Collagen, elaslin and reliculin belong to the fibrillar elements of connective tissue. Indicate the arninoacid which constitutes only collagen, and identification of which in biological fluids is used for the diagnosing of the connective tissue diseases.

W. Hydroxyproline

B. Proline
C Lysine

D. Phenylalanine

E. Glycine

2.    A histological spacemen presents
parenchymal organ, which has cortex and
medulla. Cortex consists of epitheliocytes
bars, between them there are blood capi­
llaries; the bars form three zones. Medulla
consists of chromaffinocytes and venous si­
nusoids. What organ has these morphologi­
cal features?

A. Kidney <tJ? Adrenal gland

C. Lymph node

D.      Thyroid

E. Thymus

3. Genetic structure of eukaryote is “exon-
intron-exon”. This structure-functional
organization of gene caused transcripti­
on peculiarities. What will be pro-i-RNA
according to the scheme?

A.      Intron-exon

B. Exon-in tron

C Exon-exon-intron (Pi Exon-intron-exon E. Exon-exon

4. A patient underwent the surgical removal
of pathologically disordered distal quarter
of small intestine. How will it influense the
absorption of nutritional substances in case
of patient’s ordinary diet?

A. Proteins absorption will be reduced w) Absorption will not be altereded

C. Carbohydrates absorption will be reduced

D.      Water absorption will be reduced

E. Lipids absorption will be reduced

5. The auscultation of a patient with dry
pleuritis has revealed plueral friction rub.
What epithelium type can cause such signs?

A- Laminated epithelium (MBp Simple flat epithelium

C. Simple prismatic epithelium

D.       Simple cubical epithelium

E. Transitional epithelium

6.  A patient with liver disease revealed
the decreasing of prothrombin level in the
blood. It can, first of all, result in the impai­
rment of:

vv The second phase of the coagulatory

hemostasis

B. Vascular-thrombocytic hemostasis

C Anticoagulative properties of the blood

D.   The   first  phase  of  the  coagulatory
hemostasis

E. Fibrinolysis

7.  A 58-year-old woman complaints of
increased tiredness, decreased capasity for
work, somnolence and dyspnea during fast
walking. Blood test revealed: erythrocytes
– 4.6 * 1012/1, hemoglobin – 92 g/1, colour
index – 0,6. Blood smear demonstrated hi­
gh contents of microcytes and anulocytes.
What anemia is it typical for?

A. Sickle cell

B.  Hemolytic
(O Iron deficiency

D. Acute posthemorrhagic

E. Penicious

8. A 39-year-old patient after radiotherapy
because of hepatoma developed ulcer of
small intestine. It was caused by the inhibi­
tion of mytotic activity of the cells, which
are responsible for regeneration of small
intestine surface epithelium. Inhibition of
what cells mitotic activity does this patient-
have?

A.       Exocryocytes with acidophilic granules

B. Columnar cells
C Endocrine cells

M>2 Crypt columnar cells without margins E. Caliciform exocrynocytes

9. A 42-year-old man was admitted to the
cardiological department because of angi­
na pectoris. Among the medicines admi­
nistered to the patient there was inhibitor
of phosphodiesterase. The concentration of
what substance in the heart muscle will be
increased?

A. ATP
B.ADP
C.GMP
D.AMP

(ft Cyclo-AMP

10. A patient with prolonged bronchopneumonia
was     admitted     to     the     therapeutic
department.    Antibiotic    therapy    was
ineffective. What medicine should be added

to the complex therapy of the patient to increase the immune status?

C^/Timalin

B. Analgin

C. Dimedrol

D. Paracetamol

E. Sulfocamphocain

 

E9H02

o

 

 

 

11.  Buffer capacity of blood was decreased
in the worker due to exhausting muscular
work. Entry of what acid substance to the
blood can this state be explained by?

A. a-ketoglutarate

B. 1,3-bisphosphoglycerate

C. Lactate

D. Pyruvate

E. 3-phosphogIycerate

12.  Galactosemia is revealed in a child.
Concentration of glucose in the blood is not
considerably changed. Deficiency of what
enzyme caused this illness?

A.       Amylo-l,6-glucosidase

B. Hexokinase

C Galactose-1-phosphate uridyltransferase

D. Galactokinase

E. Phosphoglucomutase

13.  A 35-year-old woman with chronic renal
disease has developed osteoporosis. Defi­
ciency of which from the belowmentioned
substances causes such complication?

A. A?

B. Cholesterol
CD3

D. 1.25(OH)2D3

E. 250HDS                                              m

14.  A patient with ascariasis was admi­
nistered a medicine which influenses
the immune system and is used as
immunomodulator. What medicine was
administered?

A. Piperazine adipinate

B. Naphthamon
C Pyrantel

D. Phenasol

E. Levamisole

15.  A patient with bronchial asthma duri­
ng 2 months was taking pills of predni-
solon. After she began feeling much better
she suddenly stopped taking the medicine.
What complication can develop in this case?

A. Hypotension

B. Obesity of upper part of the body
C Itsenko-Cushing syndrome

D. Abstinence syndrome

E. Gastric hemorrhage

16.i )A 16-year-old patient died because [Ofiliffuse fibrinopurulent peritonitis. The autopsy in the inferior region of small intestine revealed an ulcer with a form of Peyer’s plaque, with the perforation of intestine wall. The microscopic examinati­on revealed lymph tissue pattern vagueness, the displacement of this tissue with-proli-ferating monocytes, which were forming

granulomas. Complication of what disease caused the death?

A.       Nonspecific ulcerative colitis

B. DVsentry

C. Cholera

D. Brucellosis

E. Typhoid

17. A patient with bronchial asthma is admi­
nistered inhalation of 0,5% isadrine soluti­
on. Bronchospasm was releaved, but the
patient began complaining of pain in the
heart region and palpitation. It is connected
with the stimulation of:

A.        M-cholinergic receptors

B. Acetylcholine synthesis

C. /?j2-adrenoreceptors

D. /?i -adrenoceptors

E. ai-adrenoceptors

18. A 65-year-old man with signs of profuse
obesity and risk of fatty liver dystrophy
was recommended high-lipotropic diet. The
contents of what substance in his diet is the
most effective in this case?

A.        Glucose

B. Methionine

C. Glycin

D.        Cholesterol

E. Vitamin C

19. A malarial Plasmodium (haemamoeba)
– the pathogene of vivax malaria – has two
strains: southern and northern. They differ
by the duration of their incubation period:
the southern has short and the northern –
long one. What selection works in this case?

A. Artificial

B. Sexual
C Cutting

D. Moving

E. Stabilizing

20. After brain injuiry a patient has lost
his vision. What zone of the brain cortex
is damaged in this case?

A.        Occipital

B. Frontal

C. Temporal and parietal
O. Parietal

E. Temporal

21. The microscopic examination of wound
lavage of a patient with acute woundy
process of his shin revealed big contents of
irregular extended-formed cells, with tough
nucleus, the basophilic cytoplasm of which
includes many lysosomes, phagosomes and
pinocytotic bubbles. What cells are found
out in the wound?

 

E9II02

$

 

 

 

A. Fibroblasts

B. Connective tissue macrophages

C. Plasmocytes

D. Tissue basophils

E. Fibrocytes

22.  The electronic microphoto of kidney
fragment has demonstrated the afferent
glomerular arteriole, which under its
endothelium has giant cells, containing
secretory granules. Name the type of these
cells:

A.       Interstitial

B. Mcsangial

C. Juxtavascular

D. Juxtaglomerular

E. Smoothmuscular

23.  A patient has myocardial infarction in
the region ofanterior wall of the left ventri­
cle. What artery basin has the circulatory
impairment occured in?

A.  Atrioventricular branch of left coronal
artery

B. Circumflex branch of left coronal artery

C. Left marginal branch of left coronal artery

D.  Anterior interventricular branch of left
coronal artery

E.  Anterior ventricular branches of right
coronal artery

24.  A 50-year-old man had been blocked up
for 2 days because of earthquake. After he
was rescued, the diagnosis of compression
syndrome was made. What further compli­
cation is the most likely to develop in this
case?

A. Acute hepatic failure

B. Acute respiratory failure

C. Acute cardiac failure

D. Acute renal failure

E. Acute vascular failure

25.  A child, suffering from pylorostenosis
accompanied by frequent vomiting,
developed signs of dehydration. What form
of acid-base disbalance can develop in this
case?

A. Metabolic acidosis

B. Gaseous acidosis

C. Nongaseous alkalosis

D. Nongaseous acidosis

E. Gaseous alkalosis

2(5. A woman came to the surgeon because of the solid mass in her mamma. What di­rection should the surgeon make an incision in during the operation to injure the lobe less?

A. Transverse

B. Radial
G-

D. Arcuate

E. Vertical

(27J A 59-year-old man has signs of the parenchymatous jaundice and portal hypertension. On histological examinati­on of the puncture of the liver bi op talc, it was revealed: beam-lobule structure is affected, part of hepatocytes has signs of fat dystrophy, port-portal connective tissue septa with formation of pseudo-lobulcs.with periportal lympho-macrophage infiltrations. What Is the most probable diagnosis?

A. Toxic dystrophy

B. Alcohol hepatitis

C. Viral hepatitis

D. Liver cirrhosis

E. Chronic hepatosis

28.  During the bacteriological tests of the
purulent secreta from urethra there were
found bacteria, which according to Gramm
were negatively staining, looked like coffee
beans. These bacteria were splitting glucose
and maltose to acid, they were located insi­
de the leucocytes. The aetiological agent of
what disease are these microorganisms?

A. Syphilis

B. Chancroid

C. Melioidosis

D. Venereal lymphogranulomatosis

E. Gonorrhoea

29.  Shock and signs of acute renal failure
(ARF) developed in the patient due to
severe injury. What is the leading cause of
development of ARF in the case?

A.       Decreased arterial pressure

B. Decreased oncotic BP

C. Increased pressure in the nephron capsule

D. Urine excretion violation

E. Increased pressure in the renal arteries

30.  During the examination of blood gases
a patient with chronic respiratory diseases
accompanied by dyspnea, tachycardia and
cyanosis demonstrated the development of
hypoxemia and hypercapnia. What disorder
of the external respiration does the patient
have?

A. Hypoperfusion

B. Hyperdiffusion

C. Hypoventilation

D. Hyperventilation

E. Hyperperfusion

31.  A 25-year-old patient complained of vi­
sion reduction. Accommodation disorder,
dilated pupil, not reacting on the light were

 

E9I102

4

 

 

 

revealed on examination. Function of what muscles is disturbed?

A. Pupil narrowing muscle, ciliary

B.  Pupil narrowing and dilating muscle

C.  Pupil dilating muscle, ciliary

D. Inferior oblique muscle, ciliary

E. Lateral rectus muscle, pupil narrowing

32. A woman, who was keeping a diet for a long time and eating refined rice, has developed polyneuritis (beriberi disease). The absence of what vitamin in the diet causes the development of such disease?

A.     Pyridoxal

B. Folic acid

C. Thiamine

D. Riboflavin

E. Ascorbic acid

 

,330 A patient with diabetes mellitus Suddenly began having sharp pajn in his ri­ght foot. The examination revealed black hallux, swallen foot tissues, focuses of epi­dermis detachment, sccreia with unpleasant smell. What clinico-morphological form of necrosis developed in a patient?

A.                                                           Moist gangrene

B. Bedsore

C. Dry gangrene

D.     Sequester

E. Infarction

34. A young man increased his energy inputs from 500 to 2000 kJ per hour. What from the suggested can cause it?

A. Food taking

B. Physical activity
C Rising of external temperature

D. Mental activity

E. Change of sleeping for waking

A patient died from acute cardiac *—msufficiency.The histological examination of his heart revealed in myocardium of the left ventricle the necrotized section, whi­ch was separated from-undamaged tissue by the zone of hyperimic vessels, small hemorrhages and leukocytic infiltration. What is the most likely diagnosis?

A.     Focal exudate myocarditis

B. Diffuse exudate myocarditis

C. Productive myocarditis

D.     Myocardial ischemic dystrophy

E. Myocardial infarction

36, Substitution of the glutamic acid on valine was revealed while examining initi­al molecular structure. For what inherited pathology is this typical?

A. Favism

B. Sickle-cell anemia

C. Hemoglobinosis

D. Minkowsky-Shauffard disease

E. Thalassemia

37. The heart rate and the systemic arteri­
al blood pressure of a man have increased
due to voluntary respiratory delay for 40 c.
Realisation of what regulation mechanism
caused these changes?

A.        Unconditioned parasympathetic reflexes

B. Unconditioned sympathetic reflexes

C. Conditioned sympathetic reflexes
D.-

E. Conditioned parasympathetic reflexes

38.   To anaesthetize the manipulation
connected with burn surface treatment, a
patient was intravenously injected a medi­
cation for short-acting narcosis. 1 minute
later the patient being under anaesthesia
had elevated blood pressure, tachycardia,
increased tone of sceletal muscles: the
reflexes were reserved. After awakeni­
ng the patient had desorientation and vi­
sual hallucinations. What medication was
injected to the patient?

A. Sombrevin

B. Thiopental sodium

C. Nitrous oxide

D. Ketamine

E. Diethyl ether

39. A woman with myasthenia developed
respiratory disorders, which required arti­
ficial ventilation. What typey^f respiratory
insufficiency did the woman have?

A. Obstructive

B. Thoracophrenic

C. Neuromuscular

D.        Restrictive

E. Centrogenic

40.  Dystrophic changes of the heart
muscle are accompanied with cardiac cavi­
ty enlargement, decrease of the strength
of heart contraction, increased amount of
blood, which remains in the heart during
systolic phase, overfilled veins. For what
state of heart is it characteristic?

A.  Emergency stage of hyperfunction and
hypertrophy

B. Myogenic dilatation

C. Cardiosclerosis

D. Tonogenic dilatation

E. Tamponage of the heart

41.   Because of suspected inlrahospital
infection in the neonatal department of the
maternity home the inspection was carried
out. In some children and on some general

 

E9H02

5

 

 

 

things Staphylococcus aureus was revealed. What properties of these cultures allow to establish their origin from one source?

A. Phagotype

B. Biochemical activity

C. Antibioticogramma

D. Antigenic structure

E. Chromogenesis

42.  A patient with paroxysmal attacks of
asphyxia, which appear after inhalation
of different aromatic substances has been
made a diagnosis of bronchial asthma. Ig E
rate is elevated. What type of reaction is it
the most typical for?

A. Delayed type of hypersensitivity

B. Immunocomplcx
C Cytotoxic

D. Anaphylactic

E. Autoimmune

43.  A 33- year-old woman who has been
treated for a long time from chronic
polyarthritis complains of increased blood
pressure, changes in the distribution of
the adipose tissue and dysmenorrhea. The
usage of what medicine could cause such
complaints?

A.        Butadion

B. Indometacin
C Sinafian

D. Beclometasone

E. Prednisolone

44.    The electrocardiogramme analysis
demonstrated that the duration of man’s
heart cycle is Isec. What is the heart rate
per minute?

A. 80

B.50 C.60 D. 100 E.70

45. A 2-year-old boy has some enlargement
of his liver and spleen and cataract. Sugar
concentration in blood is elevated but
glucose tolerance test is normal. The
congenital metabolic disorder of what
substance is the cause of this condition?

A. Sucrose

B. Fructose

C. Galactose

D. Maltose

E. Glucose

46.   On the 8th day since the patient
was inoculated with antitclanic serum
because of dirty wound of his foot he
has developed rising temperature up to
38°C, pains in the joints, rash and itch.

The blood tests revealed leukopenia and thrombocytopenia. Allergic reaction of what type has developed in this case?

A. Anaphylactic

B. Cytotoxic

C. Immunocomplex

D. Delayed type of hypersensitivity

E. Stimulating

47.  A 42-year-old woman with trigeminal
nerve neuralgia complaints of redness of ri­
ght part of her face and neck, feeling of heat
rush and increased dermal sensitiveness.
What is the arterial hyperemia in this case
according to pathophysiological mechani­
sm?

A. Metabolic

B. Neuroparalytic

C. Neurotonic

D. Working

E. Reactive

48. A 6-months-old baby has got frequent
and extensive subdermal hemorrhages. The
administration of the synthetic analogue of
vitamin K (vicasol) was effective. Gamma-
carboxylation of glutamic acid of what
protein of blood coagulation system does
this vitamin take part in?

A. Rosental’s factor

B. Fibrinogen

C. Prothrombin

D. Antihemophilic globulin A

E. Hagcman’s factor

49. Analeptical remedy of reflective type
from the H-cholinomimctics group was gi­
ven to the patient for restoration of breathi­
ng after poisoning with carbon monoxide.
What medicine was prescribed to the pati­
ent?

A.        Adrenalin hydrochloride

B. Pentamin

C. Atropine sulphate

D. Lobeline hydrochloride

E. Mesaton

50. A 39-year-old man has got an increased
risk of infectious processes development,
hyperkeratinizattion, disorder of mesopic
vision. What vitamin should be admini­
stered?

A. Pyridoxine hydrochloride

B. Ergocalciferol

C. Tocopherol acetate

D. Riboflavine

E. Retinoi acetate

51. During the examination of a patient wi­
th bleeding wounds the doctor found out
that the tissue was damaged by maggots,

 

E9HU2

6

 

 

 

there were local maturations. The diagnosis was obligate myiasis. The maggots of what insect caused the disease?

A. Filth (house) fly

B. Tsetse fly (Glossina)

C. Stable fly

D. Blow fly (Musca Volfarti)

E. Triatomic bug

/52>tA padent died 3 days after the operati-S-tinf because of perforated colon with the manifestations of diffuse purulent peritoni­tis. The autopsy demonstrated: colon mucos membrane was thickened and covered wi­th a fibrin film, isolated ulcers penetrated into different depth. Results of histology: mucous membrane necrosis, leukocytes infi­ltration with hemorrhages focuses. The complication of what disease caused the patient’s death?

A. Typhoid

B. Amebiasis

C Nonspecific ulcerative colitis

D. Crohn’s disease

E. Dysentery

53.  Patient with pneumonia has intolerance
to antibiotics. Which of the combined
sulfanilamide medicines should be prescri­
bed to the patient?

A. Aethazol

B. Natrium sulfacyi
C Sulfadimethoxine

D. Biseptol

E. Streptocid

54.  The examination of blood serum of
a patient with immunodeficiency signs
revealed antybodies to gP120 i gP41 protei­
ns. The presence of what infection of this
patient does it confirm?

A. HLTV-1-infection

B. HBV-infcction
CHIV-infection
D.TORCH-infection
E. ECHO-infection

55.  During the expirement performed on
a rabbit the superior cervical node of
sympathaticstem was removed. On the side
of the extraction there is redness and rising
temperature of the head skin. What form of
peripheral circulation disorder developed in
this case?

A. Neuroparalytic arterial hyperemia

B. Neurotonic arterial hyperemia
C Metabolic arterial hyperemia

D. Stasis

E. Venous hyperemia

pfi.^During the examination of a patient,

who had been to the mountain pasture and had been hospitalized in a bad condi­tion with fever, the doctor found out the enlargement of inguinal lymph nodes to 8 cm, which were attached to the surroundi­ng tissues, immovable, the skin above them was red and tender. The microscopic examination of the node revealed acute serohemorrhagic inflammation. What di­sease is it typical for? 6

A. Tularemia

B. Plague

C. Brucellosis

D. Syphilis

E. Anthrax

57.  A patient has tissue ischemia lower the
knee joint, accompanied with “intermittent
claudication”. Which artery is occluded in
this case?

A.       Anterior tibial

B. Proximal part of the femoral

C. Tibial

D. Popliteal

E. Posterior tibial

58.  An old man had infarction of the ri­
ght cerebrum hemisphere. One year later,
taking into consideration the absence of
movements of his left extremities, the pati­
ent underwent computer tomography of
the cerebrum, which revealed a smooth-
walled cavity, full of liquor. What pathologi­
cal process is found out in the cerebrum?

A.        Gray maceration of the brain

B. Hydrocephaly

C. Hematoma

D. Postinfarction cyst

E. Brain infarction

59.  A patient with fibrillation, who has had
bronchial asthma in his anamnethis, should
be administered antiarrhythmic drug. Whi­
ch preparation from the suggested group is
contramdicated for this patient?

A. Nifedipine

B. Novocainamid

C. Ajmaline

D. Anapriline (Propranolol)

E. Verapamil

60.   Chronic glomerulonephritis was di­
agnosed in a 34-year-old patient 3 years ago.
Edema has developed for the last 6 months.
What caused it?

 

E9UQ2

7

 

 

 

A. Hyperosmolar! ly of plasma

B. Hyperaldosleronism

C. Proteinuria

D. Hyperproduction of vasopressin

E.   Disorder the livers protein formation
function

61. Patients with collagenousis have the
destruction of the connective tissue. It is
confirmed by increasing in the blood of:

A. Contents of oxyprolin and oxylysin

B. Activity of transaminases

G. Contents of creatine and creatinine

D. Acyivity of isoenzymes of LDH

E. Contents of urates

62. During metabolic process active forms
of the oxygen including superoxide anion
radical are formed in the human body. With
help of what enzyme is this anion activated?

A.     Peroxidase

B. Glutathionereductase

C. Catalase

D. Superoxide dismutase

E. Glutathioneperoxidase

63. Processes of repolarisation are disturbed
in ventricular myocardium in examined
person. It will cause amplitude abnormali­
ties of configuration and duration of the
wave:

A.S B.Q C.R D.P

E.T

[ 6-Ojnpainfull formation without marked Mwfaers_appeared in the soft tissues of the * thigh”(lep^of the young man. On the tissue bioptate the formation lookes like a meat of a fish, consisting of the immature fibroblast-like cells with multiple mitosis, which grow through the muscles. What is the most likely diagnosis?

A.     Fibroma

B. Myosarcoma

C. Myoma

D. Fibrosarcoma

E. Cancer

(©5.X 30-year-old patient with bacteriologi-cally proved dysentery developed the signs of paraproctitis. What is the stage of local changes of this patient?

A. Fibrinous colitis

B. Healing of the ulcers stage
C Catarrhal colitis

D. Ulceration stage

E. Follicular colitis

66.  During ultrasound examination of the
heart the doctor observes the leaves of the
mitral valve. What happends to them during
the systole?

A.      They clasp with wall of the vessel

B. They close up covering the lumen of the
orifice

C. They turn inside the cavity of the atrium

D. They clasp with the walls of the atrium

E. They turn inside the cavity of the ventricle

67.  An experimental dog by means of
stomach tube was given 150 ml of meat
broth. The concentration of which of
the belowmentioned substances will be
increased in animal’s blood?

A. Neurotensin

B. Somatostatin

C Vasoactive intestinal polypeptide

D. Insulin

E. Gastrin

68.  A female patient complaining of indi­
gestion and diffuse abdominal pain came
to the doctor. During the examination the
doctor found out considerably decreased
hemoglobulin level in the blood. During the
inquiry it turned out that the woman, living
in the Far East, was frequently eating lightly
salted caviar. Some relatives, who were livi­
ng with her, had the same complaints. What
was the diagnosis?

A.     Ascariasis

B. Trichinosis

C. Echinococcosis

D. Thenyosis

E. Diphyllobothriasis

69.   4 months ago a 43-year-old man
underwent traumatic amputation of his left
lower extremity. At the time of his exami­
nation the patient complaints of feeling of
present amputated extremity and constant
sometimes unbearable pain in it. What pain
type does the patient have?

A. Phantom limb

B. Thalamic
C Reflex

D. Neuralgia

E. Causalgia

/vOJExtensive thromboembolic infarction of She left cerebral hemispheres, large septic spleen, immunocomplex glomerulonephri­tis, ulcers on the edges of the aortic valves, covered with polypous thrombus with colonies of staphylococcus were revealed on autopsy of the young man who di­ed in coma. What disease caused cerebral thromboemboly?

 

E9H02

X

 

 

 

A. Septic bacterial endocarditis

B. Acute rheumatic valvulitis
C Septicopyemia

D. Septicemia

E. Rheumatic thromboendocarditis

(TlJTbe autopsy of the dead body revealed Naflarged, solid with rounded margins liver, on section the tissue was yellow-brown in colour with dark red spots and stripes, whi­ch resembled nutmeg. What pathological process led to such liver changes?

A. Chronic venous polyemia

B. Chronic hemorrhage
C Arterial anemia

D. Arterial polyemia

E. Acute venous polyemia

72.    A patient with clinical signs of
encephalitis was admitted to the infectious
hospital. In his anamnesis there is informati­
on about tick bite. During the hemaggluti­
nation ihibition reaction there were found
antybodies against the pathogen of tick-
borne encephalitis in diluton 1:20, which is
not diagnostic What should the doctor do
next, after he has got such results?

A. Reexamine the same serum

B. Remake the examination of the serum
taken 10 days later

C  Canceal  the  diagnosis  of tick-borne encephalitis

D. Use more sensitive reaction

E. Remake the examination with another
diagnosticum

73.   A patient with urolithiasis after the
examination was administered allopurinol
– competitive inhibitor of xanthine oxidase.
It was influenced by chemical analysis of the
calculuses, which consisted mainly of:

A.      Calcium phosphate

B. Sodium urate

C. Calcium oxolate dihydrate

D. Calcium sulphate

E. Calcium oxolate monohydrate

74.   A patient suffers from nocturnal
paroxysms of bronchial asthma accompani­
ed by bradycardia, spastic intestinal pains
and diarrhea. Medicine of what group can
releavc these symptoms?

A.      /?-adrenoblockers

B. M-cholinergic receptors

C. Saluretics

D. Sympatholytics

E. H-cholinergicreceptors

75. In the microspacemen of spinal medulla
it is necessary to analyse the condition of the
nucleus, neurons of which form the motor
endings in the skeletal musculature. What

nucleus of the spinal medulla is it about?

A. Nucleus thoracicus (Clarke’snucleus)

B. Nucleus intermediolaterallis

C. Posterior horn proper nucleus

D. Disseminate nucleus

E. Gray substance proper nucleus

76. A 50-year-old patient complains of thi­
rst, drinking of a lot of water, marked
polyuria. Blood glucose is 4,8mmol/L. uri­
ne glucose and acetone bodies are absent,
urine is colorless, specific gravity is 1,002-
1,004. What is the cause of polyuria?

A.       Insulin insufficiency

B. Vasopressin insufficiency

C. Aldosteronism

D. Hypothyroidism

E. Thyrotoxicosis

77.    The examination of an youth
with mental retardation revealed
eunuchoid body construction and geni­
tals underdevelopment. The cells of the
oral cavity cantained chromatine. What
method of genetic investigation should
be performed to make more specified di­
agnosis?

A.       Population-statistic

B. Dermatoglyphics

C. Biochemical

D. Cytological

E. Clinico-genealogical

78.   A patient, who suffers from congeni­
tal erythropoietic porphyria, has skin
photosensitivity. The accumulation of which
compound in the skin can cause it?

A. Uroporphyrinogen 1

B. Coproporphyrinogen 3

C. Uroporphyrinogen 2

D. Protoporphyrin

E. Heme

79.   A patient who came to the doctor
because of his infertility was administered
to make tests for toxoplasmosis and chronic
gonorrhoea. Which reaction should be
performed to reveal latent toxoplasmosis
and chronic gonorrhoea in this patient?

A. Immunoblot analysis

B. RIHA – Reverse indirect hemagglutinati­
on assay

C. RDHA – Reverse direct hemagglutination
assay

D. I FA – Immunofluorescence assay

E.  (R)CFT- Reiter’s complement fixation
test

80.   For serological diagnostics of the
whooping cough there was made a large-
scale   reaction   with   parapertussis   and

 

E9U02

9

 

 

 

pertussis diagnostics. At the bottom of the test-tubes with diagnosticum of Bordetella parapertussis grain-like sediment was formed. What antibodies have this reacti­on revealed?

A.       Bacteriolysins

B. Precipitins

C. Opsonins

D. Agglutinins

E. Antitoxins

81. A patient, undergoing a course of curati­
ve starvation, has a normal glucose level of
the blood mainly due to glucogenesis. What
aminoacid in human liver the most actively
synthesizes glucose?

A.       Valine

B. Leucine

C. Alanine

D. Glutamic acid

E. Lysine

82. Where should the cathetor for evacuati­
on of the lymph from the thoracic lymph
duct be inserted?

A.       To the right venous corner

B. To the inferior vena cava
C To the superior vena cava

D. To the left inguinal vein

E. To the left venous corner

83. During total (with water) alimentary
starvation the generalized edemata have
developed. Which of the pathogenic factors
is dominant in this case?

A. Increased osmotic pressure of interstitial
fluid

B.  Reduced hydrostatic pressure of intersti­
tial fluid

C.   Reduced  oncolitic pressure of blood
plasma

D. Increased oncolitic pressure of interstitial

ifiuid                                                           sy

E.   Reduced  osmotic pressure  of blood
plasma

84. The alternate usage of dichlotiazide,
etacrin acid and lasex didn’t cause substanti­
al diuretic effect in patient with evi­
dent peripheral edema. There is increased
amount of aldosterone in the blood. Indi­
cate the medicine to be prescribed.

A.        Mannit

B. Urea                                           ■

C. Spironolacton

D. Clopamid

E. Amilorid

85. Prozerin, given systematically to the rat,
increases skeletal muscles tone. Halothane
(phthoroihan) causes the relaxation of the

skeletal muscles and weakens the effect of prozerin. Determine the nature of interacti­vity of prozerin and halothane.

A. Incompetitive antagonism

B. Independent antagonism

C. Direct functional antagonism

D. Competitive antagonism

E. Indirect functional antagonism

86.  A 50- year-old man suddenly began
having severe palpitation, pain in the
heart, sharp weakness, high blood pressure;
arrhythmic pulse, with deficiency. The ECG
revealed no P waves and there was different
duration of R-R intervals. What disorder of
heart rhythm did the patient have?

A. Paroxysmal tachicardia

B. Respiratory arrhythmia

C. Sinus extrasystolia

D. Atrial fibrillation

E. Transversal heart block

87.   A patient during a visit to his denti­
st developed severe hypotension. Which
of the medicines, stimulating adrenergic
structures, should be used to normalise
blood pressure?

A. Doxazozin

B. Ergotamine
C Sanorin

D. Mesaton

E. Xylomethazolin hydrochloride

88rfThe intraoperative biopsy of mammal feland has revealed the signs of atypical ti­ssue with disorder of parenchyma-stroma proportion with domination of the last, gland structures of different size and shape, fined with single-layer proliferative epi­thelium. What is the most appropriate di­agnosis?

A. Infiltrative cancer

B. Papilloma

C. Noninfiltrative cancer

D. Mastitis

E. Fibroadenoma

89.   Blood sampling for bulk analysis is
recommended to be performed on an empty
stomack and in the morning. What changes
in blood count can occur if to perform blood
sampling after food intake?

A. Increased contents of leukocytes

B. Reduced contents of erythrocytes

C. Increased contents of erythrocytes

D. Reduced contents of thrombocytes

E. Increased plasma proteins

90. After the genealogy analysis a geneticist
came to the conclusion: a feature is mani­
fested in each generation, men and women

 

E9H02

10

 

 

 

inherit the feature with equal frequency, parents in the equal way give this feature to their offspring. What type of inheritance does the investigated feature have?

A. Autosomal-dominant

B.  X-linked dominant inheritance
C X-linked recessive inheritance

D. Autosomal-recessive

E. Y-linked inheritance

91.  A patient suffers from hemeralopia.
Which of the suggested substances will have
curable effect?

A. Carotene

B. Keratin

C. Creatine

D. Carnosine

E. Carnitine

92.   Patients with diabetes mellitus and
people during starvation have the increased
contents of acetone bodies in their blood,
which are used as energy material. What
substance are they synthesized from?

A.        Malate

B. Ketoglutarate

C. Succinyl-CoA carboxylase

D.        Citrate

E. Acetyl-CoA carboxylase

93.   The person was selling “homemade
pork”sausages on the market. State sani­
tary inspector suspected falcification of
the sausages.With help of what serologi­
cal immune reaction can food substance be
identified?

A. Precipitation test

B. Immunofluorescence test

C Indirect hemagglutination test

D. Complement-fixation test

E. Agglutination test

94.   A 44-year-old woman was admitted
to the therapeutic department because of
right-side pleuritis. The examination confi­
rmed the presence of liquid in the pleural
cavity. What sinus of the pleura will have
the biggest accumulation or the serosity?

A. Right mediasiinodiaphragmatic

B. Lett costomediastinal

C. Right costomediastinal

D. Right costodiaphragmatic

E. Left mediastinodiaphragmatic i

95. A 45-year-oldInfan with domes-tic apper
arm injuiry applied, to the trauma unit. The
objective data are: no extension, adduction
and pronation functions of the arm. What
muscle damage caused this condition?

A. Teres minor

B. Subscapular

C. Supraspinous

D. Subspinous

E. Teres major

96. A 30-year-old man with culling wound
of his forearm has got an impairment of his
fingers extension. What nerve is damaged?

A.       Radial

B. Middle

C. Ulnar

D. Muscularcutaneous

E. Medial antebrachial cutaneous nerve

97.  Systemic amebiasis with involvment of
intestines, liver, lungs was diagnosed in a
52-year-old patient. What drug should be
prescribed?

A.       Metronidasol

B. Tetracycline

C. Quiniofone

D.       Quingamine

E. Enteroseptol

98.  Some time later after the intensi­
ve physical training a sportman activates
his gluconeogenesis. What is its main
substance?

A. Serine

B. a-ketoglutaratc

C. Lactate

D. Glutamic acid

E. Aspartic acid

99.  A woman complaining of coryza,
phonastenia, eyelids redness and lacrymati-
on during spring period came to the doctor.
What type of allergic reaction by Gell ano
Coombs classification develops in this case?

A. Cytotoxic

B. Immunocomplex

C. Delayed type of hypersensitivity

D. Stimulating

E. Anaphylactic

AOOThe autopsy of the man’s body, that was wofking for a long time as a miner and di­ed beacuse of chronic pulmonary-cardiac insufficiency, revealed that lungs were not in their ful volume, significantly thickened, sclerotic, the apaxis were emphysematous^ changed, the^Sorface was gray-black in colour, on section the lung tissue was aspid-black in colour. What disease caused the death?

A.       Aluminosis

B. Talcosis

C. Asbcstosis

D.       Anthracosis

E. Silicosis

 

EQIlOZ

III

 

 

 

101.   A 38-year-old patient after taking
aspirin and sulphanilamides has profound
hemolysis of erythrocytes, which is caused
by glucose 6-phosphate dehydrogenase
insufficiency. The disorder of formation of
what coenzyme causes this pathology?

A. Pyridoxalphosphate

B.NADPH

OFMNHn

D. Ubiquinone

E. FADH-i

102.   The examination of a foreigner
revealed intestinal schistosomiasis. How
could the patient be infected?

A.        Through dirty hands

B. While eating meat

C. While eating fish

D.        During river swimming

E. Through insects bites

103. The study of the genealogy of a family
with hypertrichosis (helix excessive pilosis)
demonstrated, that this feature is mani­
fested in all generations only in men and
is inherited by son from his father. What is
the type of hypertrichosis inheritance?

A. X-linked recessive chromosome

B. X-linked dominant chromosome
G. Y-linked chromosome

D. Autosome-recessive

E. Autosome-dominant

104.   Synthesis of phospholipids is di­
sordered due to fat infiltration of liver. Indi­
cate which of the presented substances can
enhance the process of methylation during
phospholipids synthesis?

A. Methionine

B. Citrate

C. Ascorbic acid

D. Glycerin

E. Glucose

105.   Patient with vomiting, dizziness,
sensation of dubble vision, difficult
swallowing was admitted to the hospi­
tal. Doctor suspects botulism. What di­
agnostic methods should be used for di­
agnosis approving?

A.        Bacteriological, mycological

B. Biological test, bacteriological

C. Allergic test, serological

D.        Protozoological, microscopical
E.-

106.  A patient complaining of weight loss
(10 kg during 2 months), pallpilalion and
exophthalmos came to the endocrinologi­
st. For the hyperfunction of what endocri­
ne gland (glands) are these complaints the

most typical?

A. Thyroid

B. Pancreas

|C Adrenal glands

D. Parathyroid glands

E. Ovaria

107.  A man because of 1,5 litre blood
loss has suddenly reduced diuresis. The
increased secretion of what hormone
caused such diuresis alteration?

A. Natriuretic

B. Corticotropin

C. Cortisol

D. Parathormone

E. Vasopressin

108.     A patient has increased contents of uric
acid in his blood, what is clinically mani­
fested by pain syndrome due to accumulati­
on of urates in his joints. What process does
the formation of this acid result from?

A.        Purine bases re-using

B. Purine nucleotide decay

C. Heme catabolism

D.        Pyrimidine nucleotide decay

E. Proteolysis

109.     In case of enterobiasis acrihine – the
structural analogue of vitamin B-2 – is admi­
nistered. The synthesis disorder of which
enzymes does this medicine cause in mi­
croorganisms?

A. FAD – dependent dehydrogenases

B. Peptidases

C Cytochromeoxidases

D. NAD – dependent dehydrogenases

E. Aminotransferases

110. Patient in the unconscious state was
admitted to the emergency room. Skin
is cold, pupils are delayed, breathing is
heavy, with cycles of the Cheyne-Stokes
type, blood pressure is decreased, urinary
bladder is overloaded. Poisoning with what
substance is the most likely?

A. Non-narcotic analgesics

B. Narcotic analgesics

C. Sedatives
D.-

E. M-choIinergic antagonists

111. A 42-year-old man was administered
prymajan in order to prevent the distant
results pf quartan malaria. 3 days since the
beginning of the treatment with therapeutic
dosage of this medicine the patient has
developed pain in the cardiac and abdomi­
nal regions, dyspepsia and general cyanosis.
What could cause such side effects?

 

E9U02

12

 

 

 

A. Potential activity with other preparations

B. Delay preparation excretion with urine

C.   Reduced activity of microsomal liver
enzymes                               – ;>.

D. Accumulation of the medication

E.    Genetic   insufficiency   of   glucose   6-
phosphate dehydrogenase

112. During the postsynthetic period of mi­
totic cycle the synthesis of proteins – tubuli-
nes, which take part in the mitosis formati­
on, was destroyed. It can cause the impai­
rment of:

A. Duration of mitosis

B. Chromosome spiralization
C Chromosome despiralization

D. Chromosome separation

E. Cytokinesis

113. After the usage of acetylsalicylic acid
a patient developed epigastric pain because
oi exacerbation of his ulcer, what are the
principles of this medication ulcerogenity?

A. Stimulation of pepsin secretion

B. Spasm of vessels

C. Immunodepressive effect

D. Cholagogic effect

E. Antiprostaglandinic effect

[lJ4pThe microscopic examination of liver Biopsy revealed granulomata, consisting of plasmatic, lymphoid, giant polynuclear cells of Pirogov-Laehgans type, small vessels wi­th the signs of endo- and perivasculitis, and there were focuses of caseation necrosis. What disease are these granulomata typical for?

A.        Rhinoscleroma

B. Leprosy

C. Syphilis

D.        Tuberculosis

E. Glanders

115. A patient after trauma has developed paralyses, algesthia impairment on the ri-ghf; there are no paralyses on the left but temperature and pain sensivity are also impaired. What is the cause of this?

A. Brainstem injuiry

B. Unilateral right-side spinal cord injuiry

C. Midbrain injuiry

D. Cerebellum injuiry

E. Motor zone brain cortex injuiry

11<5. During the breakout of acute respi­ratory infection in order to diagnose influenza the express-diagnosis, based on revealing of specific viral antigen in the examined material (nasopharyngial lavage), is carried out. Which reaction is used for this?

A. Agglutination

B. Immunofluorescence

C. Complement binding

D. Opsonization

E. Precipitation

117.  A patient with thrombophlebitis is
administered the complex therapy, which
influences different stages of clotforming.
Which of the given substances contributes
to the restoration of the vascular permeabi­
lity?

A. Acetylsalicylic acid

B. Dipiridamol

C. Heparin

D. Ncodykumarin

E. Fibrinolysin                                       H

118. In the micropreparation made from
patient’s regional lymph node punctate
and stained according to Romanovsky-
Giemsa method, the doctor found out thin
microorganisms with 12-14 equal ringlets
and pale-pink sharp pointes 10-13 mkm in
length. The pathogen of what disease is it
about?

A. Surra

B. Syphilis

C. Relapsing fever

D. Leptospirosis

E. Leishmaniasis

119.  A 27-year-old man has purulent
inflammation of gallblader. What region of
the peritoneal cavity will the pus fall into if
gallblader ruptures in his typical position?

A. Into the left lateral duct

B. Into the superior duodenal sinus
C Into hepatic bursa

D. Into pancreatic bursa

E. Into epiploic bursa

120.  In the microspecimen of red bone
marrow there were revealed multiple capi­
llars through the walls of which mature
blood cells penetrated. What type of capi­
llars is it?                                                &$,

A.        Somatical

B. Sinusoidal

C. Fenestrational

D. Lymphatic

E. Visceral

121.  A patient who had been suffering
for many years from bronchial asthma di­
ed from asphyxia. The histological exami­
nation of his lungs revealed the following:
much mucus with eosinophiles contents in
the lumen of bronchioles and small bronci,
sclerosis of interalveolar septa, dilation of
alveolar lumen. Which of the mechanisms
of allergy development is it?

 

E9H02

13

 

 

 

A.       Granulomatosis

B. Reagin

C. Cytotoxic

D. Cytolysis, caused by lymphocytes

E. Immunocomplex

n22!) During the intraoperative biopsy of rthyroid gland the histological examination *? revealed lymphoid structures with growth centers, which were among the folliculi full of colloid. What disease was it?

A. Endemic goiter

B.  Ridel’s goiter

C Hashimoto’s thyroiditis (lymphadenoid goiter)        f

D. Sporadic goiter

E. Thyrotoxicosis (Basedow’s goiter)

123. A patient has got frequent internal
organs’ and mucosal hemorrhages.
The analysis indicated insufficiency of
hydroxyproline and hydroxylysine in the
collagenous fibres. The insufficiencuy of
what vitamin caused the impairment of
bydroxylysation of abovementioncd ami-
noacids in a patient?

A. Vitamin PP

B. Vitamin C
C Vitamin K

D. Vitamin A

E. Vitamin H

124. The increased intraocular tension is
observed in the patient with glaucoma.
Secretion of aqueous humor by the ciliar
body is normal. Injury of what structure of
the eyeball wall caused the disorder of fluid
flow-out from the anterior chamber?

A. Back epithelium of cornea

B. Choroid

C. Ciliar body

D. Ciliary muscle

E.  Venous sinus

125.  A patient because of pathological
process has thickened alveolar membrane.
What will be reduced directly as a result
this?

A.       Oxygen capacity of blood

B. Diffuse lung capacity

C. Minute respiratory capacity

D. Reserve expiratiory capacity

E. Alveolar lung ventilation

126. On blood grouping on the system
ABO, standard serum of the I and II groups
caused erythrocytes agglutination of the
examined blood and serum group of the
III didn’t. What agglutinogens are in this
erythrocytes?

A. A

B.B

C. D and C

D.C

E.A and B

127. An 18-year-old man with asthenic body
constitution: tall, narrow shoulders, broad
pelvis and with poor hair on his face came
to the geneticist. There was marked mental
retardatoin. The preliminary diagnosis was
Klinefelter’s syndrome. What method of
medical genetics can confirm the diagnosis?

A.        Dermaioglypbics

B. Population-statistic

C. Genealogic                                         9

D. Cytogenic

E. Gemellary

128. A 48-year-old patient after severe
psychoemotional exertion suddenly began
feeling sharp pain in the heart region,
irradiating into left arm. Nitroglycerin
releaved pain 10 minutes later. What
pathogenetic mechanism is responsible for
the development of pain in this case?

A.        Dilation of peripheral vessels

B. Compression of coronary vessels

C. Spasm of coronary vessels

D. Increase of myocardial needs in oxygen

E. Occlusion of coronary vessels

129. Local lymphonodulcs enlarged near
the infected wound. Increased amount
of macrophages, lymphocytes, lymphatic
follicles in the cortical layer and large
amount of plasma cells were revealed on
histological examination. What process in
the lymphatic nodules represent these hi­
stological changes?

A.   Innate insufficiency of the lymphoid
tissue

B. Hypersensibility reaction

C. Antigen stimulation

D.  Acquired insufficiency of the lymphoid
tissue

E. Tumour transformation

130.  The penetration of the irritable cell
membrane for potassium ions has been
increased experimentally. What changes of
membrane electric status can occur?

A. Hyperpolarization

B. Depolarization

C. Local response

D. No changes

E. Action potential

131.   A 56-year-old patient with complains of
thirst and frequent urination was diagnosed
to have diabetes mellitus and butamin was
prescribed. What is the mechanism of acti-

 

E9U02

14

 

 

 

on of this medicine?

A.  It stimulates beta-cells of Langergans’
islets

B. It inhibits alpha cells of Langergans] islets

C. It inhibits absorption of glucose in the
intestines

D. It helps to absorb the glucose by the cells
of the organism tissues

E. It relieves transport of glucose through
the cells’ membranes

132.    A 26-year-old woman is complaining of
thirst and dryness in her mouth. The exami­
nation has revealed glucosuria and blood
glucose content of 6,5 mmol/l. What condi­
tion are these symptoms the most typical
for?

A.       Diabetes mcllitus

B. Diabetes insipidus

C. Alimentary glucosuria

D. Renal diabetes

E. Steroid diabetes

133.   Karyotyping of healthy man cells
is carried out. In the karyotype there
was found out a fine acrocentric odd
chromosome. What chromosome is it?

A.        Group A chromosome

B. Group B chromosome
C X-chromosome

D. Y-chromosome

E. Group C chromosome

134. Highly injured person gradually died.
Please choose the indicator of biological
death:

A. Loss of consciousness

B. Absence of movements

C. Disarray of chemical processes

D. Autolysis and decay in the cells

E. Absence of palpitation

13fO A patient died with uremia.The autopsy demonstrated kidneys to be enlarged, of flabby consistence, cortex was broad, edematous, with red spots; medulla was dark red in colour. Microscopic exami­nation in the glomeruli capsule cavity revealed “semilunar formations”,squeezing capillaries, nephrocytes dystrophy, edema and infiltration of stroma. What disease caused the death of the patient?

A.        Glomeruloncph rosis

B. Nephrotic syndrome

C. Renal amyloidosis

D. Pyclonepn rosis

E. Nephrolithiasis

136. A student is thoroughly summarising a lecture. When his groupmates began talki­ng the quality of the summarising worsened

greatly. What type of the inhibition in the cerebral cortex is the cause of this?

A. Protective

B. External

C. Dying

D. Delayed

E.  Differential

137. Complaints and objective data allow to
suggest that a patient has inflammation in
his gallblader. bile colloidoclasia, probabili­
ty of gallstones formation. What can be the
main cause of gallstones formation?

A.        Oxolates

B. Urates

C. Chlorides

D. Cholesterol

E. Phosphates

138. The energy inputs of a healthy man
have been identified. What position was the
patient if his energy inputs were less than
the main exchange?

A.        Easy work

B. Sleep

C. Calmness

D. Rest

E. Nervous exertion

139.  The speed of excitement conducti­
on through the atrioventricular node in a
healthy adult is 0,02-0,05 mps. What does
artrioventricular delay supply?

A.        Sequence of atria and ventricles contracti­
on

B. Sufficient force of ventricles contraction

C. Simultaneous contraction of both atria

D.        Sufficient force of atria contraction

E. Simultaneous contraction of both ventri­
cles

140. After trauma a 44-year-old patient had
a rupture of left palm muscle tendons and of
the surface of blood vessels. A fter operation
and removal of the most part of the necroti-
cally changed muscle tissue the bloodstream
was normalized. What vessels helped to
restore the bloodstream?

A.        Aa. digitales palmares communes

B. Aa. metacarpeae palmares
C Arcus palmaris profundus

D. Arcus palmaris supcrficialis

E. Aa. perforantcs

141.   The patient has taken the mi­
xture prescribed by neuropathologist for
neurasthenia for 2 weeks. Patient fell better
but developed coryza, conjunctivitis, rash,
inertia, decrease ol memory. Bromizm was
diagnosed. What should be prescribed to
decrease symptoms?

 

E9U02

15

 

 

 

A.-

B. Asparcam

C Glucose solution 5%

D. Polyglucin   >

E. Natrium chloride

142.   The patient with complaints of
permanent thirst applied to the doctor.
Hyperglycemia, polyuria and increased
concentration of 17-ketosteroids in the uri­
ne were revealed. What disease is the most
likely?

A. Insulin-dependent diabetes mellitus

B. Type 1 glycogenosis

C. Steroid diabetes

D. Addison’s disease

E. Myxoedema

143.     Decreased blood supply to the organs
causes hypoxia that activates fibroblasts
function. Volume of what elements is
increased in this case?

A. Nerve elements

B. Intercellular substance

C. Parenchymatous elements of the organ

D. Lymphatic vessels

E. “Vessels of microcircular stream

144.     A patient in winter fall down into ice-
hole, froze and fall ill. The temperature
increased up to 39,7°C and ranged from
39,0°C to 39.8°C. Name the type of
temperature curve of this patient.

A. Febris remittens

B. Febris continua

C. Febris intermittens

D. Febris recurrens

E. Febris hectica

145.     A child was diagnosed to have fracture
of humerus. The broken arm began retardi­
ng in growth. What part of the bone was
damaged?

A. Diaphysis B.Metaphysis

C. Marrow canal

D.        Apophysis

E. Epiphysis

146.  A 6-month-old child is suffocating
while lying on his back.The palpation on
the anterior wall of trachea up to jugular
notch of the sternum revealed tumor-like
mass, stretching into the anterior mediasti­
num. What from the belowmentioned can
squeeze trachea?

A. Parathyroid glands

B. Parathyroid lymph nodes

C. Peritracheal lymph nodes

D. Thymus gland

E. Thyroid gland

147.  A palient’l liver can not normally
detoxify its natural metabolites and
xenobiotics.The reduced activity of what
cytochrome can cause this?

A. Cytochrome C — 1

B.  Cytochrome R – 450

C.  Cytochromoxidase

D. Cytochrome B

E. Hemoglobulin

148.  During investigation of patient, it was
found formation in the white substance of
cerebral hemispheres with location in the
knee and frontal part of posterior crus of
internal capsule. Fibres of what conductive
tract of the brain will be disrupted?

A.       Tr. frontopontinus

B. Tr. thalamocorticalis

C. Tr. parietooccipitopontinus

D.       Tr. frontothalamicus

E. Tr. pyramidalis

149.   A patient with iron deficiency
anemia has been treated at the in-patient
hematological department of the regional
children’s hospital during 13 years. What
hypoxia type does this patient have?

A.       Respiratory

B. Tissue

C. Circulatory

D.       Combined

E. Hemic

150.    After the radiactive exposure a patient
has stem cells disorder. The regeneration of
what cells of friable connective tissue will
be damaged?

A. Adipocytes

B.  Fibroblasts

C.  Macrophages

D. Pericytes

E. Pigment cells

151. A 2-year-old child has got intestinal
dysbacteriosis, which results in hemorrhagic
syndrome. What is the most likely cause of
hemorrhage of this child?

A. PP hypovitaminosis

B. Hypocalcemia

C. Activation of tissue thromboplastin

D. Fibrinogen deficiency

E. Vitamin K insufficiency

fl5X)The autopsy a 35-year-old man body “”revealed in the second segment of the right

 

E9U02

16

 

 

 

lung the focus of carnification, which was 5 cm in diametre and was enclosed in a thin capsule. The focus was a tough dry friable tissue with a dim surface. What disease are such morphological changes typical for?

A.       Chondroma

B. Postinflammatory pneumosclerosis

C. Tumorous form of silicosis

D. Lung cancer

E. Tuberculoma

153.  Child asked you to puff up the balloon
as much as possible for a one exhalati-
on.What air volume will you use?

A. Total volume of the lungs

B. Vital volume of the lungs

C. Inspiration volume

D. Functional residual volume

E.  Backup volume of the inspiration

154.    A 43-year-old man suffers from
chronic atrophic gastritis and megaloblastic
hyperchromic anemia. He also has
methylmalonic aciduria. Insufficiency of
what vitamin led to the development of such
complex of symptoms?

A.       Vitamin 65

B. Vitamin B12
C Vitamin B3

D. Vitamin Bi

E. Vitamin Bo

155. A patient with brain bloodstream di­
sorder has got difficulties with swallowing,
he can choke over while eating liquid food.
What part of the brain is damaged?

A.        Thalamencephalon

B. Midbrain

C. Medulla

D. Cerebellum

E. Cervical part of the spinal medulla

156. A 40-year-old man was admitted to the
surgical department with spleen rupture.
What anatomic formation will accumulate
the blood?

A. Right lateral canal

B. Hepatic bursa

C Rectovesical excavation

D. Bursa pregastrica

E. Omental bursa

157. On examination of the person it was
revealed that minute volume of heart is
3500 mL, systolic volume is 50 mL. What
is the frequency of cardiac contraction?

A. 80 bpm

B. 70 bpm

C. 90 bpm

D. 50 bpm

E. 60 bpm

158.   During the complicated labour the
symphysis pubis ruptured. What organ can
be damaged the most?

A. Urinary blader

B. Uterine tubes

C. Rectum

D. Uterus

E.  Ovaria

159.  During the merry-go-round riding a 25-
year-old woman began having nausia, vomi­
ting and intensive sweat. Activation of whi­
ch receptors caused the reflex development
of these symptoms?

A. Vestibular receptors of semicircular duct

B.  Optic receptors

C.  Receptors of Corti’s organ

D. Otolith vestibular receptors

E. Proprioreceptors of skeletal muscles

160.  Woman applied to the medico-genetic
consulting centre for information about the
risk of haemophilia in her son. Her husband
has been suffering from this disease since
birth. Woman and her parents are healthy
(don’t have haemophilia). Is the’boy likely
to have the disease in this family?

A. 25% of the boys will be ill

B. AH boys will be ill

C. All boys will be healthy

D. 50% of the boys will be ill

E. 75% of the boys will be ill

161.     A doctor made the diagnosis
of gonorrhoea. It was known from
the anamnesis that a patient had had
gonorrhoea before and he had been treated
completely. What type of infection can this
new disease be attributed to?

A. Secondary infection

B. Superinfection

C. Relapse

D. Autoinfection

E. Reinfection

162. Ion cells have been blocked in the exci­
tative cell. It doesn’t change significantly
the quiet potential, hut (he cell has lost its
capacity to the generation of AP (action
potential). What canals have been blocked?

 

E9U02

17

 

 

 

A.      Calcium

B. Chlorine

C. Sodium

D. Sodium and potassium

E. Potassium

163.   A 63-year-old man, suffering from
cancer of esophagus, has metastases to
mediastenum, lymph nodes and cancer
cachexy. What is the pathogenetic stage of
the given tumorous process?

A. Progression

B. Promotion
C-

D. Initiation

E. Transformation

164.   Because of present gallstone in the
common bile duct, a patient has no bile
excretion into duodenum. What disorder
can it cause?

A. Lipids digestion

B. Carbohydrates absorption

C. Proteins absorption

D. Proteins digestion

E. Carbohydrates digestion

165.   A patient with acute myeloblast
leucosis has developed liver and spleen
enlargement, anemia, myeloblasts in peri­
pheral blood. What principal sign allows
to differ, myeloblast leukosis from chronic
one?

A.      Blast cells in peripheral blood

B. Leukemic collapse

C. Pancytopenia

D. Anemia

E. Thrombocytopenia

(l66yfTht examination of skin biopsy of a patient with hemorrhage vasculitis revealed the next: the vessel wall was thickened, hjgmggenous, stained in yellow colour by means of picrofuchsin, PAS-positive. What pathologic process has developed in the vessels walls?

A.      Mucoid swelling

B. Lipidosis

C. Amyloidosis

D. Fibrinoid swelling

E. Hyalinosis

167. A 60-year-old man after cerebral hemorrhage felt asleep for a long time. Damage of what structure caused this state?

A.      Black substances

B. Reticular formation

C Cortex of the large hemispheres

D. Nuclears of the cerebral nerves

E. Hippocampus

168.  Careless student occasionaly met his
dean. The concentration of what hormone
will most likely increase in the blood of the
student?

A. Corticotropin

B. Adrenalin

C. Thyrotropin-releasing hormone

D. Cortisol

E. Somatotropin

169.   A man in a quiet condition has
increased activity of the muscles, responsi­
ble for the inspiration. What from the
belowmentioncd can cause this?

A.        Reduction of minute respiratory capacity

B. Superficial breathing

C. Infrequent breathing

D. Negative intrapleural pressure

E. Airways narrowing

170. A patient was admitted to the hospital
with a wound in the region of his neck. The
examination revealed the damaged nerve,
situated in front of the anterior ladder
muscle. What nerve is damaged?

A. Cervical part of sympathetic trunk

B. Glossopharyngeal

C. Sublingual

D. Phrenic

E. Vagus

171. A patient with chronic heart failure bei­
ng treated by digitalis drugs developed the
symptoms, which confirmed the beginning
of cardiac glycosides toxic activity. What
drug should be administered to reduce the
negative effect of cardiac glycosides?

A. Potassium chloride

B. Sodium caffeine-benzoate

C. Dipiroxim

D. Ethymizol

E. Atropine sulfate

172. A patient after hypertension stroke has
no voluntary movements in his right arm
and leg, muscle tone in these extrcmites
is increased. What type of dysfunction of
nervous system is it?

A. Peripheral paralysis

B. Reflex paresis

C. Central paresis

D. Central paralysis

E. Peripheral paresis

173.    A patient after tooth extraction
developed persistent substernal pain. Subli­
ngual antianginal substance releavcd the
substernal pain, but the patient complained
of headache and dizziness. What medicine
did the patient use?

 

E91I02

18

 

 

 

A. Metoprolol

B. Valklol

C. Anapriline

D. Verapamil

E. Nytroglycerin

174. The study of mitotic cycle phases of
onion root revealed the cell, in which the
chromosomes are situated in the equatorial
plane, forming a star. What stage of the cell
mitosis is it?

A. Interphase

B. Metaphase-

C. Prophase

D. Anaphase

E. Telophase

175. A patient becomes quickly tired duri­
ng his work. In vertical position with closed
eyes he is dizzying and loosing equilibri­
um. Skeletal muscle tone is reduced. Which
of the belowmentioned brain structures is
damaged?

A. Basal ganglia

B. Thalamus

C. Hypothalamus

D. Precentral gyrus of cerebral hemispheres
cortex

E. Cerebellum

176. Patients with similar complaints appli­
ed to the doctor: weakness, pain in the
intestines, disorder of GIT. Examination of
the faeces revealed that one patient with
four nucleus cysts should be hospitalized
immidiatcly. For what protozoa arc such
cysts typical?

A. Trichomonas

B. Dysenteric amoeba

C. Lamblia

D. Intestinal amoeba

K. Balaniidium

177. During the staining of sputum smear
of a patient with suspected croupous
pneumonia the following reactives and
stainers were used: gential violet soluti­
on, Lugol’s solution, 06° spiritus and water
fuchsin. What method of staining is used in
this case?

A.        Gram’s

B. Romanovsky’s

C. Lefflcr’s

D. Zichl-Neclsen’s

E. Neisscr’s

178. A 56-year-old patient with cardiac fai­
lure notices edematous feel and shins, the
skin in the place of edema is pale and cold.
What is the leading pathogenesis of this
patient’s edema?

A.     Increased    hydrostatic    pressure    in
vcnulaes

B. Increased capillary penetration

C. Reduced oncotic pressure in capillaries
I>. Lymph outflow impairment

E. Positive water balance

179. Patient was admitted to the infection
unit with diagnosis of bacterial dysentery.
On laboratory studies it was revealed
that causative element is sensitive to
many antimicrobial medicines, but patient
had anemia. What medicine was contra-
indicated to the patient?

A. Furazolidone

B. Enteroseptol

C. Levomycctin

D. Ampicillih

E. Phthalazol

180. Testosterone and it’s analogs increase
the mass of skeletal muscles that allows to
use them for treatment of dystrophy. Due
to interaction of the hormon with what cell
substance is this action caused?

A. Membrane receptors

B. Proteins-activators of transcription

C. Ribosomes

D. Nuclear receptors

E. Chromatin

181. During the football match a player
injuried his knee joint. The X-ray exami­
nation showed fracture of the bone, which is
situated in the tendon thickness of musculus
quadriceps femoris. What of the mentioned
groups does this bone belong to?

A. Flat

B. Tubular

C. Hollow

D. Combined

E. Sesamoid

182.    A patient suffers from arterial
hypertension strokes, accompanied by
tachycardia, excessive sweat, sharp pain in
the epigastric region. Which of the menti­
oned tumorous diseases are these symptoms
the most typical for?

I A. Pituitary basophil adenoma

B. Ovarian tumor

C.  Glomerular adrenal adenoma

D. Thyroid adenoma

E. Phcochromocytoma

183.  On bacteriological examination of the
defecation of a 4-months-old baby with the
symptoms of acute bowel iufection there
were revealed red colonies spread in the
large quantity in the Endo medium. What
microorganism can it be?

 

E9JI02

19

 

 

 

A.       Shigcll

B. Streptococcus
C Escherichia

D. Salmonella

E. Staphylococcus

184.   A patient, who had been blowing
a cloud (smoking tobacco) for a long
time developed lung cancer. Which of
the suggested cancerogenic substances is
contained in the tobacco smoke and is
attributed to the polyunsaturated aromatic
carbohydrates (PAC)?

A.        P- naphthylamine

B. Diethylnitrosamine

C. Benzpyrene

D.        Dimethylaminoazobenzene

E. Orthoaminoazotoluene

185.     A patient with symmetric dermatitis of
open dermal areas came to the doctor. Talki­
ng to the patient the doctor found out that
the patient mostly eats cereals and eats little
meat, milk and eggs. The deficiency of whi­
ch of the mentioned vitamins is dominant in
this patient?

A.        Nicotinamide

B. Folic acid

C. Biotin

D. Calciferol

E. Tocopherol

186. A child with diphtheria 10 days after
injection of antitoxic antidiphtherial serum
has developed skin rash, accompanied by
severe itch, rising temperature up to 38°C
and joints pain. What is the cause of these
symptoms?

A. Contact allergy

B. Delayed type of hypersensitivity

C. Atopia

D. Serum sickness

E. Anaphylactic reaction

187.   A group of men applied to the
doctor complaining of rising temperature,
headache, swelling of face and eyelids,
myalgia. From the history it became known
that they all were hunters and they often
ate meat of wild animals. What is the most
likely diagnosis?

A.        Trichinosis

B. Teniasis

C. Filariasis

p. Teniarrhinchosis E. Cysticercosis

188.     A patient has a suspected pneumonia.
In his sputum there were revealed
gramposilive diplococci, prolonged with the
slightly pointed opposite ands. What mi­
croorganisms are revealed in the sputum?

A.        Staphylococcus aureus

B. Streptococcus pneumoniae

C. Neisseria gonorrhoeae

D. Neisseria meningitidis

E. Klebsiella pneumoniae

J89^\ 46-year-old man on his head skin has got a little bulged out nevus pigmentosis of a dark colour which didn’t trouble him. For the last time the nevus began growing in size, a pain developed, the colour became black-brown; the palpati­on of it demonstrated a node at its base. The histological examination of the removed tissue revealed spindle-shaped and polymorphic cells, the cytoplasm of which contained the pigment of fulvous colour. What tumor is it about?

A. Hemangioma

B. Carcinoid

C. Basalioma

D. Hematoma

E. Melanoma

190yA patient with burn disease died due Toprogressive septicaemia. The autopsy in the anterior abdominal wall by means of microscopic examination revealed di­ffuse infiltration of intermuscular spaces by segmentonuclear leukocytes, tissues edema and lysis of muscular fibres. Determine the nature of the pathological process.

A. Necrosis

B. Catarrhal inflammation

C. Abscess

D. Phlegmon

E. Diphtheritic inflammation

191.     A 58-year-old man has a clinical picture
of acute pancreatitis. The increase of what
substance in the urine from belowmenti-
oned will confirm the diagnosis?

A. Urea

B. Amylase

C. Albumin

D. Nonprotein (rest) nitrogen

E. Uric acid

192.     In the blood smear, stained according
to Romanovsky-Giemsa method, there are
20% big (20 mem in diametre), rounded
cells with pale-basophilic cytoplasm and
bean-shaped nucleus. How is this conditi­
on characterised clinically?

A.        Leukopenia

B. Neutrophilosis

C. Monocytosis

D.        Lymphocytosis

E. Reticulocytosis

193.   A   40-year-old   pregnant   woman

 

E9U02

20

 

 

 

underwent amniocentesis. The examinati­on of fetus karyotype revealed 47Xy+21. What pathology of the fetus was found out?

A. Phenylketonuria

B. Patau’s disease

C. Klinefelter’s syndrome

D. Down’s syndrome

E. Schereschevsky-Terner’s disease

194.  Person has stable HR, not more than
40 bpm. What is the pacemaker of the heart
rhythm in this person?

A.       Sinoatrial node

B. Atrioventricular node

C. His’ bundle

D. Branches of His’ bundle

E. Purkinye’ fibers

195.  2 weeks since the blood transfusi­
on a recepient has developed fever. What
protozoal disease can it be?

A.       Trypanosomiasis

B. Malaria

C. Leishmaniasis

D. Toxoplasmosis

E. Amebiasis

196.  A 42-year-old man suffering from
gout has increased level of urinary acid in
the blood. Allopurinol was prescribed to
decrease the level of urinary acid. Competi­
tive inhibitor of what enzyme is allopurinol?

A.        Hypoxantinphosphoribosiltransferase

B. Xanthinoxidase

C Guaninedeaminase

D. Adenosinedeaminase

E. Adeninephosphbribosiltransferase

19Z)On autopsy of the 58-year-old man it !s~fevealed: mitral valve is deformed, thi-

ckened, not totally closed. Microscopically: centers of collagen fibers are eosinophilic, have positive fibrin reaction. The most li­kely it isf

A.. Hyalinosis

B. Mucoid swelling

C. Amyloidosis

D. Fibrinoid swelling

E. Fibrinoid inflammation

198.  While examining the child the doctor
revealed sym.netric cheeks roughness, di­
arrhea, disfunction of the nervous system.
Lack of what food components caused it?

A.        Nicotinic acid, tryptophane

B. Lysine, ascorbic acid

G. Threonine, pantothenic acid

D. Methionine, lipoic acid

E. Phenylalanine, pangamic acid

199.  A 40-year-old man ran 10 km per 60
minutes. What changes of energy exchange
will take place in his muscular tissue?

A. Glyconeogenesis intensification

B. Proteolysis intensification

C. Increase of fatty acids oxidation rate

D. Glycolysis intensification

E. Glycogenosis intensification

200.  Isolated muscle of a frog is rhythmi­
cally irritated with electric impulses. Every
next impulse is in a period of relaxati­
on from the previus contraction. What
contraction of the muscle appears?

A. Continuous (smooth) tetanus

B. Single

C. Tonic

D. Waved tetanus

E. Asynchronous

 

 

 

 

 

 

 

 

 

 

 

 

 

 

 

 

 

MINISTRY OF

Boa3rd of education and science of MPH of Ukraine Testing Boa

 

Test items for licensing examination

Krokl

MEDICINE

2005

 

General Instruction Every one of these numbered questions or unfinished statements in this chapter corresponds to answers or statements endings. Choose the answer (finished statements) that fits best and fill in the circle with the corresponding Latin letter on the answer sheet.

 

EEK

54.1*73 61

 

Authors of Items: Andronov D.YU., Amautova L.V., Barinov B.P., Byclousova I.R, Boiko O.V., Boriscnko B.O., Vaschuk A.A., Visockiy I.YU., Vilhova I.V., Volohova G.O., Vorobcts Z.D., Gavrilyuk O.M., Gargin V.V., Glamazdina N.M., Goldobina O.V., Guzik N.M., Gurcalova LP., Davidcnko I.S., Dcm’yancnko I.A., Demchcnko O.M., Dycitsova O.I., Dogadina I.V., Zhadinskiy M.V., Zhilyuk V.I., Zhulinskiy V.O., Zagorulko O.K., Zaharov V.I., Zclenina N.M., Zlcnko O.T., Ivascnko A.V., Kaptyuh R.F., Kirjakulov G.S., Klincova S.P., Klopockiy G.A., Koroicnko G.S., Korol A.P., Krushinska T.YU., Kryuk YU.YA., Kubishkin A.V.. Kuzik I.JA., Kukovska I.L., Kulaccnko B.V., Kulitka E.F., Kuccvol O.V., Lazarev K.L., Loiytra A.O., Lucyuk M.B., Makovctskiy O.V., Matyushcnko P.M., Michaiylyuk I.O., Mrug V.M., Navniko T.S., Ncchiporcnko G.V., Nikifrchin R.M., Nikonova A.A., Nikolcnko O.I., Nikolenko YU.I., Nikoncnko T.M., Ovchinnikov S.O., Pandikidis N.I., Piddubnyak YU.G., Poguda A.A., Pushkar M.S., Romancnko M.I., Savchcnko N.V., Semcnyuk T.M., Sidorchuk I.Y., Skorobagata T.G., Skrcbkova O.YU., Skripka O.V., Skryabina O.M., Sluchcnko O.M., Smolyakova G.M., Starlichanova L.D., Stcblyuk M.V., Slcts V.R., Tananakina T.P., Tatarko S.V., Tertishniy S.I., Tomashova S.A., Tumas’yants N.YU., Ulyanccva E.A., Fartushok N.V., Fcdorchenko O.V., Halaturnik G.M., Holodkova O.L., Hramova R.A., Chcrkasov V.G., Cherkashina L.P., Chcsnokova M.M., Shalanin V.V., Shcvtsova A.I., Shcmonacva K.F., Scherbakov S.M., Schcrbakov M.S., Scherbakov C.N., Yasinskiy V.I. and Committees of professional expertise.

Item reviewers. Andrycyeva V.F., Basiy R.V., Borzenko B.G., Visotskiy I.YU., Vinnikov YU.M., Golovatyuk O.L., Gorgol N.I., Deyneka S.YE., Dycitsova O.I., Zhadinskiy M.V, Zclenina N.M., Zinkovska L.YA., Yoltuhivskiy M.V, Kalikik K.G., Kalugina S.M., Kirichck L.T., Kiryuhin I.R, Kovalchuk L.YE., Korolenko G.S., Krushinska T.YU., Linchevska L.R, Lyubinets L.A., Mrug V.M., Nikolcnko O.I., Ovchinnikov S.O., Ostrovska S.S., Pcntyuk O.O., Pototska I.I., Pushkar M.S., Sikora V.Z., Tcleshova O.V, Trzhetsinskiy S.D., Fcdechko IY.M., Fomina L.V, Chemenko N.A.

Composers. Artemchuk L., Bulakh I., Homchenkova N.

The book Includes test items for use at licensing integrated examination “Krok 1. Medicine” and further use In teaching.

The book has been developed for students of medical, pediatric and mcdical-and-prophylactic faculties and academic staff of higher medical educational establishments.

 

Approved by Ministry or Public Health of Ukraine as examination and teaching publication based on expert conclusions (Orders of MPH of Ukraine of 14.08.1998 Jfl-251, of 27.12.1999 JVM03, of 18.06.2002 JVb22l, of 16.10.2002 JYW74, of 16.04.2003 M239, of 29.05.2003 JV4233).

© Copyright

Testing Board, 2005.

 

Krok 1 Medicine 2005

1

 

 

 

1. The conjugated protein necessarily contains special component as a non-protein part. Choose the substance that can’t carry out this function:

A. AMP

B.  Thiamine pyrophosphate
IC HN03

D.ATP E. Glucose

2.  Marked increase of activity of MB-forms
of CPK (creatinephosphokinase) and LDH-
1 was revealed by examination of the pati­
ent’s blood. What is the most probable
pathology?

A. Cholecystitis

B. Miocardial infarction

C. Pancreatitis

D. Hepatitis

E. Rheumatism

3.  A 46 year-old man complains of difficult
hose breathing. Mikulich cells, storage of
epithelioid cells, plasmocytes, lymphocytes,
hyaline balls are discovered in the biopsy
material of the nose thickening. What is the
most likely diagnosis?

A. Meningococcal nasopharyngitis

B. Scleroma

C. Rhinovirus infection

D. Virus rhinitis

E. Allergic rhinitis

4.  A man after 1,5 litre blood loss has
suddenly reduced diuresis. The increased
secretion of what hormone caused such di­
uresis alteration?

A. Corticotropin

B. Natriuretic

C. Parathormone

D. Cortisol

E. Vasopressin

5.  A 45-year-old woman suffers from allergic
seasonal coryza caused by the ambrosia
blossoming. What adipose cells group stabi­
lizer medicine can be used for prevention of
this disease?

A. Tavegyl

B.  Ketotifen

C.  Diazoline

D. Phencarol

E. Dimedrol

6.  A 32-year-old patient has been diagnosed
with bartholinitis (inflammation of Bartholi­
n’s glands ). In what part of the female
urogenital system are the Bartholin’s glands
located?

A.        The labia minor

B. The clitoris

C. The uterus

D. The vagina

E. The labia major

7.  The action of electric current on the
exitable cell caused depolarization of its
membrane. Movement of what ions through
the membrane caused depolarisation?

A.C/” B.K+

C. Ca2+

D. HCOJ
E.Na+

8.    A 10-year-old child complains of
weakness, nausea, irritability. Helminthes
of white color and 5-10 mm long have been
found on the underwear. On microscopy of
the scrape from the perianal folds achromic
ova of the unsymmetrical form have been
revealed. Which helminth is in the organism
of the child?

A. Trichuris

B. Trichina

C. Ascaris lumbricoides

D. Ancylostoma duodenalis

E. Enterobins vermicularis

9.  A 2-year-old child experienced convulsi­
ons because of lowering calcium ions
concentration in the blood plasma. Function
of what structure is decreased?

A. Adrenal cortex

B. Thymus

C. Parathyroid glands

D. Pineal gland

E. Hypophysis

10.  A patient with injured muscles of the
lower extremities was admitted to the
traumatological department. Due to what
cells is reparative regeneration of the muscle
fibers and restoration of the muscle function
possible?

A.        Myoepithelial cells

B. Myoblasts

C. Fibroblasts

D. Myofibroblasts

E. Satellite-cells

11.  A 27- year-old woman has used penicillin
containing eye drops. In a few minutes itchi­
ng, skin burning, lips and eyelids edema,
whistling cough, decreasing BP appeared.
What antibodies can lead to this allergic
reaction?

 

Krok 1 Medicine 2005

2

 

 

 

A. IgM and IgG

B. IgM and IgD
C IgG and IgD

D. IgE and IgG

E. IgA and IgM

12.   A 50 year-old patien was injured on
the occipital region of the head. The closed
skull’s trauma was diagnosed. She was taken
to the hospital. The medical examinati­
on: deregulation of walking and balance,
trembling of arms. What part of brain was
injured?

A.       The cerebellum

B. The inter-brain

C. The spinal cord

D. The medulla oblongata

E. The mind-brain

13.   An autopsy revealed large (1-2 cm)
brownish-red, easy crumbling formations
covering ulcerative defects on the external
surface of the aortic valve. What is the most
likely diagnosis?

A. Acute warty endocarditis

B. Recurrent warty endocarditis

C. Fibroplastic endocarditis

D. Polypus-ulcerative endocarditis

E. Diffusive endocarditis

14. A denaturation of proteins can be found
in some substances, specify the substance
that is used for the incomplete denaturation
of hemoglobin:

A. Nitric acid B.Urea

C. Toluene

D. Sulfuric acid

E. Sodium hydroxide

15.  A 6-year-old child fell on the cutti­
ng object and traumatized soft tissues
between tibia and fibula. What kind of bone
connection was injured?

A. Fontanel

B. Gomphosis

C. Suture

D. Membrane

E. Ligament

16.  A patient with hypersecretion of the
gastric juices was recomended to exclude
concentrated bouillons and vegetable
decoctions from the diet because of their sti­
mulation of gastric secretion. What is domi­
nating mechanism of stimulation of secreti­
on in this case?

A.   Stimulation  of gastrin  production  by
G-cells

B. Irritation of taste receptors

C.   Irritation of mcchanoreccptors of the
stomach

D. Stimulation of excretion of secretin in the
duodenum

E. Irritation of mechanoreceptors of the oral
cavity

17.  In the ovary specimen colored with
hematoxylin-eosin, follicle is determined
where cubic-shaped follicle epithelium cells
are placed in 1-2 layers, and scarlet covering
is seen around ovocyte. Name this follicle:

A. Primary B.Secondary

C. Mature

D. Atretic

E. Primordial

18. A 57-year-old patient was admitted to the
gastroenterological department with suspi­
cion of Zollinger-Ellison syndrom because
of rapid increase of gastrin level in the blood
serum. What the most probable disorder of
the secretory function of the stomach here?

A. Achylia

B. Hypoacidity hyposecretion
C Hyperacidity hyposecretion

D. Hypoacidity hypersecretion

E. Hyperacidity hypersecretion

19. The patient with diabetes mellitus has
been delivered in hospital in the state of
unconsciousness. Arterial pressure is low.
The patient has acidosis. Point substances,
which accumulation in the blood results in
these manifestations:

A.        Ketone bodies

B. Amino acids

C. High fatty acids

D. Monosaccharides

E. Cholesterol esters

20. The low specific gravity of the secondary
urine (1002) was found out in the sick
person. Wat is the most distant part of
nephron where concentration of secondary
urine takes place?

A. In the collecting duck

B. In the nephron’s glomerulus

C. In ascending part of loop of Henle

D. In proximal tubule of nephron

E. In distal tubule of nephron

21.     Glomerular filtration rate (GFR)
increased by 20% due to prolonged starvati­
on of the person. The most evident cause of
filtration changes under this conditions is:

 

Krok 1 Medicine 2005

3

 

 

 

A.   Decrease of oncotic pressure of blood
plasma

B. Increase of penetration of the renal filter

C. Increase of filtration coefficient

D. Increase of systemic blood pressure

E. Increase of renal plasma stream

22.  Scraps of.the mycelium of a fungus,
spores, air bubbles and fat drops were di­
scovered on microscopy of the patient’s
hair excluded from the infected areas. What
fungus disease is characterised by this mi­
croscopic picture?

A. Trichophytosis

B. Epidermophytosis
G. Favus

D. Sporotrichosis

E. Microspory

23.  Moving of the daughter chromatids to
the poles of the cell is observed in the mi-
totically dividing cell. On what stage of the
mitotic cycle is this cell?

A. Interfase

B. Telophase

C. Metaphase

D. Prophase

E. Anaphase

24.  A couple came for medical genetic
counseling. The man has hemophilia, the
woman is healthy and there were no cases of
hemophilia in her family. What is the risk of
having a sick child in this family?

A. 100% B.50% C. 75%

D.25% E.0

25.  After the trauma, the patient’s right
n.vagus was damaged. Which violation of
the cardiac activity is possible in this case?

A. Block of a conductivity in the atrio­
ventricular node

B. Violation of a conductivity in the right
auricle

C. Arrhythmia

D.  Violation of the automatism of a atrio­
ventricular node

E.  Violation of the automatism of a Kiss-
Fleck node

26.  Purulent endometritis developed in a
woman after delivery. Treating with anti­
biotics inhibitors of murein synthesis was
ineffective. Wide spectrum bactericidal anti­
biotic was administered lo her. In 6 hours
temperature rapidly increased up to 40°C
with shiver. Muscle pains have appeared.
BP dropped down to 70/40 mmHg. Oligura
has developed. What is the main reason for

the development of this condition?

A.        Bacteremia

B. Endotoxic shock

C. Internal bleeding

D. Anaphylactic shock

E. Toxic effect of preparation

27. When the pH level of the stomach lumen
decreases to less than 3, the antrum of
the stomach releases peptide that acts in
paracrine fashion to inhibit gastrin release.
This peptide is:

A.        Acetylcholine

B. Gastrin-releasing peptide (GRP)

C. Somatostatin

D. Vasoactive intestinal peptide (VIP)
E.GIF

28.   A 22-year-old’ patient was admitted
to the hospital with complaints of heavy
nasal breathing. During the examination
of her nasal cavity the doctors found thi­
ckened mucous membrane, a lot of mucus
and nodular infiltrates without erosions in
the nose.The nasal rhinoscleroma was di­
agnosed. The biopsy was taken. What typi­
cal morphological changes may be found?

A. Granulomas with foreign body cells

B. Granulomas with Mikulicz’s cells

C. Interstitial inflammation

D. Granulomas with Langhan’s cells

E. Granulomas with Virchow’s cells

29.  Synthesis of phospholipids is disordered
under the liver fat infiltration. Indicate
which of the following substances can
enhance the process of methylation during
phospholipids synthesis?

A. Glycerin

B. Citrate

C. Glucose

D. Methionine

E. Ascorbic acid

30.   Obturative jaundice developed in a
60-year-old patient because of malignant
tumour of the big papillary of the duodenal.
Lumen of what anatomical structure is
squeezed with tumour?

A. Hepatopancreatic ampulla

B. Left hepatic duct

C. Cystic duct

D. Right hepatic duct

E. Common hepatic duct

31.  Live vaccine is injected into the human
body. Increasing activity of what cells of
connective tissue can be expected?

 

Krok 1 Medicine 2005

4

 

 

 

A.       Fibroblasts and labrocytes

B. Macrophages and fibroblasts

C. Adipocytes and adventitious cells

D. Plasmocytes and lymphocytes

E. Pigmentocytes and pericytes

32. The patient with pneumonia was treated
with antibiotics for a long period. After
treatment patient complains of frequent and
watery stool, abdomenal pain. What is the
reason of intestine function disorder?

A. Intestinal disbacteriosis development

B. Bacteria toxins influence

C. Autoimmune reaction development

D. Hereditary enzyme defect

E. Antibiotics toxic influence on the GIT

33.   In the microspecimen of red bone
marrow multiple capillares were revealed
through the walls of which mature blood
cells penetrated. What type of capillares is
it?

A. Somatical

B. Fenestrational
C Visceral

D. Lymphatic

E. Sinusoidal

34.  There is only one hormone among the
neurohormones which refers to the derivati­
ves of amino acids according to classificati­
on. Point it out:

A. Somatotropin

B. Oxytocin
C Melatonin
P. Vasopressin
E. Thyroliberin

35.  A young man has an unpainfull formati­
on without marked borders in the soft ti­
ssues of his thigh. On the tissue bioptate
the formation lookes like flesh of fish and
consists of immature fibroblast-like cells
with multiple mitosis growing through the
muscles. What is the most likely diagnosis?

A.        Myoma

B. Fibroma

C. Fibrosarcoma

D. Myosarcoma

E. Cancer

36.  A worker has decreased buffer capacity
of blood due to exhausting muscular work
The influx of what acid substance in the
blood can cause this symptom?

A. 3-phosphoglycerate

B. Lactate

C. Pyruvate

D. 1,3-bisphosphoglycerate

E. a-ketoglutarate

37. Ovarian tumour was diagnozed in a

woman. Surgery should be perfomed. What ligament should be extracted by the surgeon to disconnect the ovary and the uterus?

A.        Broad ligament of uterus

B. The ovanal ligament

C. Lateral umbilical ligament

D. Round ligament of uterus

E. Suspensory ligament of ovary

38.  The electronic microphoto of kidney
fragment has exposed afferent glomerular
arteriole, which has giant cells under its
endothelium, containing secretory granules.
Name the type of these cells:

A. Juxtaglomerular

B. Interstitial

C Smoothmuscular

D. Mesangial

E. Juxtavascular

39.  The reason of occurrence of some di­
seases of an oral cavity is connected wi­
th structural peculiarities of its mucous
membrane. Wnat morphological attributes
characterize these features?

A. Well developed muscularis, no submucosa

B. Transitional epithelium, no submucosa

C.  Transitional epithelium, no muscularis
mucosa

D.     No    muscularis    mucosa,    stratified
squamous epithelium

E. Simple columnar ciliated epithelium

40. The energy inputs of a healthy man have
been measured. In what position was the
patient if his energy inputs were less than
the main exchange?

A. Calmness

B. Easy work
GRest

D. Sleep

E. Nervous exertion

41. A patient after hypertension stroke does
not have voluntary movements in his right
arm and leg with the increased muscle tone
in these extremites. What type of disfuncti­
on of nervous system is it?

A.        Peripheral paresis

B. Central paralysis

C. Peripheral paralysis

D. Reflex paresis

E. Central paresis

42. There is the change of teeth at the 6-8-
year-old children: deciduous are replaced by
permanent. What embrionic tissues are the
sources of formation of permanent teeth ti­
ssues?

 

Krok 1 Medicine 2005

5

 

 

 

A.        Mesodermal epithelium and mesenhime

B. Enlodermal epithelium of a tooth plate
and mesenhime

C. Entodermal epithelium and mesoderm

D. Ectodermal epithelium of a tooth plate
and mesenhime

E. I, II brachial arches

43. A tissue sample of benign tumor was
studied under the electron microscope. A lot
of small (15-20 nm) spherical bodies, consi­
sting of 2 unequal subunits were detected.
These are:

A. Smooth endoplasmic reticulum

B. Mitochondria

C. Golgi complex

D. Microtubules

E.  Ribosomes

44. Part of the DNA chain turned about 180
degrees due to gamma radiation. What type
of mutation took place in the DNA chain?

A. Replication

B. Deletion

C. Inversion

D. Doubling

E. Translocation

45. Red colonies spread in the large quantity
in the Endo culture medium were revealed
on bacteriological stool examination of a 4-
month-old baby with the symptoms of acute
bowel infection. What microorganism can it
be?

A. Salmonella

B. Staphylococcus
C Escherichia

D. Shigell

E. Streptococcus

46.  A patient’s blood was analyzed and
the decreased erythrocyte’s sedimentation
rate (ESR) was discovered. What disease
from the listed below is accompanied with
decreased ESR?

A. Polycytemia

B. Myocardial infarction

C. Hepatitis

D. Splenomegaly

E.  Vitamin B deficiency

47.  A 13-year-old girl with history of asthma
complained of cough, dyspnea and wheezi­
ng. Her symptoms became so severe that her
parents brought her to the emergency room.
Physical examination revealed diaphoresis,
dyspnea, tachycardia and tachypnea. Her
respiratory rate was 42/min, pulse rate was
110 beats per minute, and blood pressure
was 130/70 mm Hg. Choose from the
following list the most appropriate drug to
reverse the bronchoconstnetion rapidly:

A. Ipratropium

B. Salbutamol

C. Methylprednidsolone

D. Cromolyn

E. Beclomethasone

48. Slime, blood and protozoa 30-200 mi­
crons of length have been revealed in a
man’s feces. The body is covered with cili-
as and has correct oval form with a little bit
narrowed forward and wide round shaped
back end. On the forward end a mouth is vi­
sible. In cytoplasm there are two nucleuses
and two short vacuoles. For whom are the
described attributes typical?

A.        Dysenteric amoeba

B. Intestinal amoeba

C. Balantidium

D. Trichomonas

E. Lamblia

49. An autopsy has revealed that kidneys are
enlarged, surface is large-granular because
of multiple cavities with smooth wall, which
are filled with clear fluid. What kidney di­
sease did the patient have?

A. Necrotic nephrosis

B. Polycystic kidney

C. Pyelonephritis

D. Infarction

E. Glomerulonephritis

50. An isolated muscle of a frog is rhythmi­
cally irritated with electric impulses. Every
next impulse is in a period of relaxation from
the previus contraction. What contraction of
the muscle occurs?

A. Single

B. Tonic

C. Continuous (smooth) tetanus

D. Asynchronous

E. Waved tetanus

51.   During surgical operation a blood
transfusion was made. The blood must be
checked to find antigens of some disease.
What disease is expected to be found?

A. Virus of hepatitis E

B. Virus of hepatitis B

C. Enterovirus

D. Virus of hepatitis A

E. Adenovirus

52.  A patient with the symptoms of acute
alcoholic poisoning was brought to the
hospital, what carbohydrates metabolism
changes are typical for this condition?

 

Krok 1 Medicine 2005

6

 

 

 

A. The breakage of glycogen is increased in
liver

B.  The gluconeogenesis velocity in liver is
decreased

C.    The   anaerobic   glucose   metabolism
predominates in muscles

D. The gluconeogenesis is increased in liver

E.  The anaerobic breakage of glucose is
increased in muscles

53. A patient after pathological process has
a thickened alveolar membrane. The di­
rect consequence of the process will be the
reduction of:

A.        Oxygen capacity of blood

B. Minute respiratory capacity

C. Diffuse lung capacity

D. Alveolar lung ventilation

E. Reserve expiratiory capacity

54. During the breakout of acute respiratory
infection in order to diagnose influenza
the express-diagnosis, based on revealing
of specific viral antigen in the examined
material (nasopharyngial lavage), is carri­
ed out. Which reaction is used for this?

A. Agglutination

B. Complement binding

C. Precipitation

D. Opsonization

E. Immunofluorescence

55. Pyruvate concentration in the patient’s
urine has increased 10 times from normal
amount. What vitamin deficiency can be the
reason of this change:

A.        Vitamin A

B. Vitamin B{
G Vitamin C

D. Vitamin B6

E. Vitamin E

56.  A patient complains of frequent and
difficult urination. Imperfection of what
formation can cause it?

A. Sperm bubbles

B. Bulb-uretic glands

C. Testicle adnexa

D. Prostate

E. Testicles

57.   A patient in three weeks after acute
myocardial infarction has pain in the heart
and joints and pneumonia. What is the
main mechanism of development of post­
infarction Dressler’s syndrome?

A. Thrombosis vessels

B. Ischemia of myocardium

C. Autoimmune inflammation

D.  Resorption of enzymes from necrotized
area of myocardium

E. Secondary infection

58. Different functional groups can be presented in the structure of L-amino aci­d’s radicals. Identify the group that is able to form ester bond:

A.-OH

B. -NH2

C. -CONH2

D. -CH3
E.-SH

5Ps A patient has undergone an amputation of lower extremity. Some time later painful nodules appeared in a stump. Amputatious neuromas were found out at the microscopic examination. To what pathological processes do those formations relate?

A.        Metaplasia

B. Dystrophy

C. Regeneration

D. Inflammation

E. Hyperemia

60.  A patient, who suffers from congeni­
tal erythropoietic porphyria, has skin
photosensitivity. The accumulation of what
compound in the skin can cause it?

A. Coproporphyrinogen 3

B. Uroporphyrinogen 2
C Uroporphyrinogen 1

D. Protoporphyrin

E.  Heme

61.  A patient who came to the doctor
because of his infertility was administered
to make tests for toxoplasmosis and chronic
gonorrhoea. Which reaction should be
performed to reveal latent toxoplasmosis
and chronic gonorrhoea of the patient?

A. RDHA – Reverse direct hemagglutination
assay

B. Immunoblot analysis

C. RIHA – Reverse indirect hemagglutinati­
on assay

D.   (R)CFT- Reiter’s complement fixation
test

E. IFA – Immunofluorescence assay

62.    A 63-year-old woman developed
symptoms of rheumatoid arthritis. Their
increase of which blood values indicators
could be the most significant in proving the
diagnosis?

 

Krok I Medicine 2005

7

 

 

 

A. Lipoproteids

B. R-glycosidase

C Acid phosphatase

D. General cholesterol

E. Additive glycosaminoglycans

63.  The process of heart transplantation
determined the viability of myocardial cells.
The determination of what myocardium
parameter is the most important?

A. Concentration of calcium-ions in myofi­
brils

B. Heart temperature

C. Rest potential of cardiomyocytes

D. Concentration of oxygen in heart vessels

E. Concentration of Ca-ions in heart vessels

64.  During the fetal period of the
development in the vascular system of
the fetus a large arterial (Botallo’s)
duct is functioning which converts into
lig.arteriosum after birth. What anatomi­
cal formations does this duct connect?

A. Right and left auricle                            I

B. Pulmonary trunk and aorta

C. Aorta and superior vena cava

D. Aorta and inferior vena cava

IE. Pulmonary trunk and superior vena cava

65.  The formation of a secondary medi­
ator is obligatory in membrane-intracellular
mechanism of hormone action. Point out the
substance that is unable to be a secondary
mediator:

A. Glycerol

B. Ca?+

C. Inositol-3,4,5-triphosphate

D. CAMP

E. Diacylglycerol

66fA patient with bronchial asthma had been taking tablets which caused insomnia, headache, increased blood pressure. What medecine can cause such complications?

A. Euphyline

B. Adrenaline

C. Izadrine

D. Chromolin sodium

E. Ephedrine

67. A patient with abscess of the cut wound applied to the traumatological department .The wound was washed with 3% hydrogen peroxide to be cleaned from the pus. Foam was not observed. What caused inefficiency of the drug?

A. Shallow wound

B. Low concentration H2O2

C. Inherited insufficiency of catalase

D. Pus in the wound

E.    Inherited   insufficiency   erythrocyte’s
phosphatdehydrogenase

68.    On autopsy of a still-born infant
abnormalities have been revealed: ventri­
cles are not separated, a single arterial trunk
originates from the right part. For what class
of vertebrates is such heart construction
characteristic?

A.        Mammals

B. Reptiles

C. Amphibian

D.        Fishes

E. Birds

69.   A 37-year-old man was admitted
to the surgical department with the
symptoms of acute pancreatitis: vomiti­
ng, diarrhea, bradycardia, hypotention,
weakness, dehydration of the organism.
What medicine should be used first of all?

A.        Contrycal

B. Platyphylline

C. Etaperazine

D.        No-spa

E. Ephedrine

70.’ A businessman came to India from South America. On examination the physi­cian found that the patient was suffering from sleeping-sickness. What was the way of invasion?

A.        As a result of mosquito’s bites

B. With contaminated fruits and vegetables
C As a result of bug’s bites

D.        After contact with a’ sick dogs

E. Through dirty hands

71. The CNS stimulation produced by
methylxanthines, such as caffeine, is most
likely due to the antagonism of one of the
following receptors:

A. Adenosine receptors

B. Glutamate receptors

C. GABA receptors

D. Glycine receptors

E. Cholinergic muscarinic receptors

72. Nowadays about 50 minor bases have
been found in the t-RNA structure besides
the main four nitrogenous bases. Choose the
minor nitrogenous case:

A. Uracil

B. Cysteine

C. Dihydrouracil

D. Cytosine

E. Adenine

 

Krok 1 Medicine 2005

8

 

 

 

73. The donor who had not donated the
blood for a long time was examined wi­
th IFA method. Anti-HBs antibodies were
revealed. What does positive result of IFA
in this case mean?

A. Acute hepatitis C

B. Previous hepatitis B
C Chronic hepatitis C

D. Acute hepatitis B

E. Chronic hepatitis B

74. A 45-year-old man with domestic apper
arm injuiry came to the trauma unit. The
objective data are: there are no extension,
adduction or pronation functions of the arm.
What muscle damage caused this condition?

A.        Teres major

B. Subspinous

C. Supraspinous

D. Subscapular

E. Teres minor

75. Inflamation is characterised by increasi­
ng penetration of vessels of microcirculati­
on stream, increasing of their fluid dynamic
blood pressure. Increasing of the osmotic
concentration and dispersity of protein
structures can be found in the intercellular
fluid. What kind of edema are to be
observed in this case?

A. Membranogenic

B. Colloid-osmotic

C. Mixed

D. Hydrodynamic

E. Lymphogenic

76. A patient died from acute cardiac insuffi­
ciency. The histological examination of
his heart revealed the necrotized section
in myocardium/of the left ventricle, whi­
ch was separated from undamaged tissue
by the zone of hyperimic vessels, small
hemorrhages and leukocytic infiltration.
What is the most likely diagnosis?

A.        Diffuse exudate myocarditis

B. Focal exudate myocarditis

C. Myocardial infarction

D. Productive myocarditis

E. Myocardial ischemic dystrophy

77. An autopsy revealed: soft arachnoid
membrane of the upper parts of cerebral
hemisphere is plethoric, of yellowish-green
color, soaked with purulent and fibrose
exudate, it lookes like a cap. What disease
is characterised by these symtoms?

A. Meningitis at typhus

B. Influenza meningitis

C. Meningitis at anthrax

D. Tuberculous meningitis

E. Meningococcal meningitis

78.  The preventive radioprotector was gi­
ven to a worker of a nuclear power station.
What mechanism from the below mentioned
is considered to be the main mechanism of
radioprotection?

A.        Increasing of respiration

B. Inhibition of free radicals formation

C. Prevention of tissue’s hypoxia

D. Activation of oxidation reactions

E. Increasing of tissue blood supply

79.  A consumptive patient has an open
pulmonary form of disease. Choose what
sputum staining should be selected for findi­
ng out the tubercle (Koch’s) bacillus?

A. Method of Romanowsky-Giemsa

B. Method of Burry-Gins

C. Method of Neisser

D. Method of Gram

E. Method of Ziel-Neelsen

80.   Chronic glomerulonephritis was di­
agnosed in a 34-year-old patient 3 years
ago. Edema has developed within the last
6 monthes. What caused the edema?

A. Hyperproduction of vasopressin

B. Proteinuria

C. Hyperosmolarity of plasma

D. Liver disfunction of protein formation

E. Hyperaldosteronism

8J”. Inhibition of alpha-motoneuron of the extensor muscles was noticed after sti­mulation of a-motoneuron of the flexor muscles during the experiment on the spi­nal column. What type of inhibition can be caused by this process?

A.        Presynaptic

B. Reciprocal
C Lateral

D. Recurrent

E.  Depolarizational

82.  Necrosis focus was observed in the area
of hyperemia and skin edema in a few hours
after burn. What mechanism strengthens
destructive effects in the inflammation area?

A. Proliferation of fibroblasts

B. Diapedesis of erythrocytes

C. Secondary alteration

D. Primary alteration

E. Emigration of lymphocytes

83.  For a long time a 49-year-old woman
had suffered from glomerulonephritis win­
ch caused death. The autopsy revealed
that the size of her kidneys was 7x3x2,5
sm, weight 65,0 g, they were dense
and small-grained. Microscopically: libri-
nogenous inflammation of serous and
mucous capsules, dystrophic changes of

 

Krok I Medicine 2005

9

 

 

 

parenchymatous organs, brain edema. What complication can cause such changes of serous capsules and inner organs?

A.       DIC-syndrome

B. Anemia

C. Thrombopenia

D. Sepsis

E. Uraemia

84. Autopsy of the 58-year-old man had*
revealed that mitral valve is deformed, thi­
ckened, does not totally close. Microscopi­
cally: centers of collagen fibers are eosi­
nophilic, have positive fibrin reaction. The
most probable diagnosis is:

A. Hyalinosis

B. Amyloidosis

C. Mucoid swelling

D. Fibrinoid swelling

E. Fibrinoid inflammation

85. A patient with encephalopathy was
admitted to neurological department.
Correlation of increasing encephalopathy
and substances absorbed by the
bloodstream from the intestines was
revealed. What substances created in the
intestines can cause endotoxemia?

A. Butyrate

B. Acetacetate
G. Indole

D. Ornithine

E. Biotin

86. Punctata hemorrhage was found out in
the patient after application of a tourniquet.
With disfunction of what blood cells is it
connected?

A. Lymphocytes

B. Neutrophiles
C Platelets

D. Monocytes

E. Eosinophiles

87. A woman who was sick with rubella duri-
ngthe_pregnancy. gave birth to a deaf child”
wltHhare lip and cleft palate. This congeni­
tal defect is an example of:

A. Edward’s syndrome

B. Patau’s syndrome

C. Phenocopy

D. Genocopy

E. Down’s syndrome

88.    A person was selling “homemade
pork”sausages on the market. State sani­
tary inspector suspected falcification of the
sausages. What serological immune reaction
can identifiy food substance?

A. Indirect hemagglutination test

B. Complement- fixation test

C. Agglutination test

D. Immunofluorescence test

E. Precipitation test

89. While enrolling a child to school Mantu’s
test was made to define whether revaccinati-
on was needed. The test result is negative.
What does this test result mean?

A.  Absence of antitoxic immunity to the
tuberculosis

B.    Presence   of   cell   immunity   to   the
tuberculosis

C.    Absence   of   cell   immunity   to   the
tuberculosis

D. Absence of antibodies for tubercle baci­
llus

E. Presence of antibodies for tubercle baci­
llus

90. A patient had been taking glucocorticoi­
ds for a long time. When the preparation
was withdrawn he developed the symptoms
of disease aggravation, decreased blood
pressure and weakness. What is the reason
of this condition?

A.        Hyperproduction of ACTH

B. Sensibilization

C. Habituation

D. Appearance of adrenal insufficiency

E. Cumulation

91. A mother of a newborn complains of her
baby’s constant belching with undigested
milk. Which developmental anomaly is it
an evidence of?

A. Anal atresia

B. Esophageal fistula

C. Esophageal atresia

D. Labium leporium

E. Faux lupinum

92. A 30-year-old patient was hospitalized
due to bleeding of the facial artery . What
place on the face has to be pressed to stop
bleeding?^

A.        The mandible’s branch

B. The nose’s back

C. The molar bone

D. The mental process

E. The mandible’s edge

93. The concentration of albumins in human
blood sample is lower than normal. This
leads to edema of tissues. What blood functi­
on is damaged?

 

Krok 1 Medicine 2005

10

 

 

 

A. Maintaining the Ph level

B.   Maintaining  the  blood sedimentation
system

C. Maintaining the body temperature

D. All answers are correct

E. Maintaining the oncotic blood pressure

94. A young man felt sharp pain in the back
during active tightening on the horizontal
bar. Objectively: pain while moving upper
extremity, limited pronation and adducti­
on functions. Sprain of what muscle can be
observed here?

A.       M.latissimus dorsi

B. M.subscapularis

C. M.levator scapulae

D. M.romboideus major

E. M.trapezius

95.   From the nasopharynx of a 5-year-
old child a microorganism was excreted
which is identical to Corynebacterium di-
phtheriae dose according to morphological
and biochemical signs. Microorganism does
not produce exotoxin. As a result of what
process can this microorganism become
toxigenic?

A. Growing with antitoxic serum

B. Phage conversion

C. Chromosome mutation

D.   Passing through the organism of the
sensative animals

E. Cultivation in the telluric environment

96.   A patient with hypochromic anemia
has splitting and loss of hair, increased nail
britthng and taste alteration. What is the
mechanism of the symptoms development?

A. Decreased production of parathyrin      l\

B.    Decreased    production    of    thyroid
hormones

C. Deficiency of vitamin B12

D. Deficiency of vitamin A

E. Deficiency of iron-containing enzymes

97. A healthy woman has three sons affected
by color blindness who were born after
her two marriages. Children both of her
husbands are healthy. What is the most
possible pattern of inheritance of this di­
sease?

A. Y-linked

B. X-linked recessive

C. Autosomal dominant

D. X-linked dominant

E. Autosomal recessive

98.   The alternate usage of dichlotiazide,
etacrin acid and lasex did not cause marked
diuretic effect in the patient with marked
peripheral edema. The aldosterone level in
the blood is increased. Indicate which medi-

cine should be prescribed:

A. Spironolacton

B. Amilorid

C. Urea

D. Clopamid

E.  Mannit

99.   Blood sampling for bulk analysis is
recommended to be performed on an empty
stomack and in the morning. What changes
in blood composition can occur if to perform
blood sampling after food intake?

A. Increased contents of leukocytes

B. Increased contents of erythrocytes

C. Increased plasma proteins

D. Reduced contents of erythrocytes

E. Reduced contents of thrombocytes

100.   A patient has a malignisation of
thoracic part of esophagus. What lymphatic
nodes are regional for this organ?

A. Nodi lymphatici paratrachealis

B. Nodi lymphatici mediastinales posteriores

C. Anulus lymphaticus cardiae

D. Nodi lymphatici pericardiales laterales

E. Nodi lymphatici prevertebralis

     101. On autopsy a 35-year-old man the focus pLcamification 5 cm in diametre enclosed ‘ in a tbin capsule was revealed in the second segment of the right lung. The focus consi­sts of a tough dry friable -Hssue^with a dim surface. For what disease are these morphological changes typical?

A. Tumorous form of silicosis

B. Chondroma

C. Tuberculoma

D. Lung cancer

E. Postinflammatory pneumosclerosis

102. A 25-year-old woman with red and itchy
eczematoid dermatitis visits your office. She
had a dental procedure one day earlier with
administration of a local anesthetic. There
were no other findings, although she indi­
cated that she had a history of allergic reacti­
ons. Which of the following drugs is most
likely involved?

A. Procaine

B. Cocaine

C. Bupivacaine

D. Etidocaine

E. Lidocaine

103. A 55-year-old patien was hospitalized in
result of the trauma of the medial group of
femoral muscles. What kind of movements
is the patient unable to do?

 

 

Krok 1 Medicine 2005

11

 

 

 

A. Adduction of femur

B. Suppination of femur
C Flexion of femur

D. Abduction of femur

E. Extension of femur

104.  An old woman was hospitalized with
acute pain, edema in the right hip joint; the
movements in the joint are limited. Which
bone or part of it was broken?                   fcj

A.        Pubic bone

B. The body of the thigh bone

C. The neck of the thigh

D.        Condyle of the thigh

E. Ischial bone

105. The penetration of the irritable cell
membrane for potassium ions has been
increased during an experiment. What
changes of membrane electric status can
occur?

A. Hyperpolarization

B. No changes

C Depolarization

D. Action potential

E. Local response

106.     The alveolar ventilation of the patient is
5 L/rnin, the breath frequency is 10 per/min,
and the tidal volume is 700 ml. What is the
patient’s dead space ventilation?

A. 4,3 L/min

B. 1,0 L/min

C. 2,0 L/min

D. 0,7 L/min
E.-

107. A 50-year-old patient with typhoid fever
was treated with Levomycetin, the next day
his condition became worse, temperature
rose to 39,6°C. What caused the complicati­
on?

A. The effect of endotoxin agent

B. Secondary infection addition

C.   Irresponsiveness  of  an  agent  to  the
levomycetin

D. Allergic reaction

E. Reinfection

108. A patient with complaints of 3-day-
long fever, general weakness, loss of appeti­
te came to visit the infectionist. The doctor
suspected enteric fever. Which method of
laboratory diagnosis is the best to confirm
the diagnosis?

A.        Detachment of myeloculture

B. Detachment of blood culture
C Detachment of pure culture

D. Detachment of feces culture

E. Detachment of urine culture

109. A 60-year-old patient was hospitali-

sed to the surgical department because of infection caused by blue pus bacillus (Pseudomonas aeruginosa) which is sensati-ve to penicillin antibiotics. Indicate which of the given penicillins has marked activity to the Pseudomonas aeruginosa?

A.        Oxacillin

B. Phenoxymethylpenicillin

C. Carbenicillin disodium

D.        Methicillin

E. Benzylpenicillin

110.  Only one factor can influence the
charge of amino acid radicals in the active
centre of enzyme. Name this factor:

A.        The presence of a competitive inhibitor

B. Pressure

C. The surplus of a product

D.        Temperature

E. pH medium

111.     A doctor administered Allopurinol to a
26-year-old young man with the symptoms
of gout. What pharmacological action of
Allopurinol ensures therapeutical effect?

A. By inhibiting uric acid synthesis

B. By general analgetic effect

C. By inhibiting leucocyte migration into the
joint

D. By general anti-inflammatory effect

E.  By increasing uric acid excretion

112.     A patient with tissue trauma was taken
a blood sample for the determination of
blood clotting parameters. Specify the right
sequence of extrinsic pathway activation.

A. IV-Vila-Xa

B. Ill-VIII: TF-Xa

C. Ill-Vila-Xa
D.III-IV-Xa
E.IV-VIII:TF-Xa

113.     A 56-year-old patient complaining of
thirst and frequent urination was diagnosed
with diabete mellitus. Butamin was prescri­
bed. How does the medicine act?

A. It helps to absorb the glucose by the cells
of the organism tissues

B. It relieves transport of glucose through
the cells’ membranes

C. It stimulates /3-cells of Langergans’ islets

D. It inhibits a-cells of Langergans’ islets

E. It inhibits absorption of glucose in the
intestines

114.   Periodic renal colics attacks are
observed in a woman with primery
hyperparathyroidizm. Ultrasonic examinati­
on revealed small stones in the kidneys.
What is the most plausible reason of the
stones’s formation?

 

Krok 1 Medicine 2005

12

 

 

 

A. Hyperphosphatemia

B. Hyperkalemia

C. Hyperuricemia

D. Hypercalcemia

E. Hypercholesterinemia

115. A person has steady HR not exceeding
40 bpm. What is the pacemaker of the heart
rhythm in this person?

A. Sinoatrial node

B. Atrioventricular node

C. His’ bundle

D. Purkinye’ fibers

E. Branches of His’ bundle

116. An intraoperational biopsy of mammal
gland has revealed the signs of atypical ti­
ssue with disorder of parenchyma stroma
proportion with domination of the last,
gland structures of different size and shape,
lined with single-layer proliferative epitheli­
um. What is the most probable diagnosis?

A. Mastitis

B. Infiltrative cancer

C. Noninfiltrative cancer

D. Fibroadenoma

E. Papilloma

117.  A highly injured person has gradually
died. Please choose the indicator of biologi­
cal death:

A. Absence of movements

B. Loss of consciousness

C Disarray of chemical processes

D. Autolysis and decay in the cells

E. Absence of palpitation and breathing

118.    A histological spacemen presents
parenchymal organ, which has cortex and
medulla. Cortex consists of epitheliocytes
bars with blood capillaries between them;
the bars form three zones. Medulla consi­
sts of chromaffinocytes and venous sinusoi­
ds. Which organ has these morphological
features?

A. Kidney

B. Lymph node

C. Thyroid

D. Adrenal gland

E. Thymus

119. A 35-year-old man under the treatment
for pulmonary tuberculosis has acute-onset
of right big toe pain, swelling, and low-grade
fever. The gouty arthritis was diagnosed and
high serum uric acid level was found. Which
of the following antituberculosis drugs are
known for causing high uric acid levels?

A. Rifampicin

B. Cycloserine

C. Pyrazinamide

D. Aminosalicylic acid

E. Thiacetazone

120. After breathing with poisonous steams
there is an increased quantity of slime in
respiratory passages of a chemical producti­
on worker. What of respiratory tract epi­
thelial cells participate in mucousa moisteni­
ng?

A. Intercalated cells

B. Goblet cells

C. Fibroblasts

D. Langergans cells

E. Endocrine cells

121. The sterile Petri dishes and pipettes
are necessary to prepare for microbiologi­
cal tests in bacteriological laboratory. What
way of sterilization should be applied in this
case?

A. Dry-heat sterilization

B. Steam sterilization in autoclave

C. Tyndallization

D. Boiling

E. Pasteurization

122.  Methotrexate (structural analogue of
the folic acid which is competitive inhibi­
tor of the dihydrofolatreductase) is prescri­
bed for treatment of the malignant tumour.
On which level does methotrexate hinder
synthesis of the nucleic acids?

A.        Mononucleotide synthesis

B. Reparation

C. Processing

D. Replication

E. Transcription

123. A patient died 3 days after the operati­
on because of perforated colon with mani­
festations of diffuse purulent peritoni­
tis. The autopsy revealed: colon mucos
membrane was thickened and covered with
a fibrin film, isolated ulcers penetrated at di­
fferent depth. The histology result: mucous
membrane necrosis, leukocytes infiltration
with hemorrhages focuses. What disease
complication caused the patient’s death?

A. Typhoid

B. Amebiasis

C. Dysentery

D. Crohn’s disease

E. Nonspecific ulcerative colitis

124.    If strong oxidizers get into the
bloodstream, a methemoglobin is formed.
It is a compound, where iron (II) becomes
iron (III). What has to be done to save the
patient?

 

Krok J Medicine 2005                                                       13

 

A.  Interchangeable hemotransfusion has to
be done

B. He has to be calmed down and put to bed
C Respiratory centers have to be stimulated

D. Patient has to be exposed to the fresh air

E. He has to be given pure oxygen

125. During the operation on the hip joint of
a 5-year-old child tier ligament was damaged
which caused bleeding.What ligament was
damaged?

A. The head of the thigh

B. Iliofemoral
C Pubofemoral

D. Perpendicular of the acetabule

E. Ischiofemoral

126. The pulmonalis embolism has suddenly
developed in a 40 year-old patient with
opened fracture of the hip. Choose the possi­
ble kind of embolism.

A. Air

B. Thrombus-embolus
CFat

D. Foreign body

E. Tissue

127.   X-ray examination discovered lungs
emphysema in the patient. What is the
reason of short breath development in this
case?

A. Inhibition of respiratory center

B.  Increased lungs elasticity

C Decreasing of alveoli receptors sensitivity

D. Excitation of respiratory center

E. Decreased lungs elasticity

128. A19 year-old patient was diagnosed wi­
th appendicitis and was hospitalized. The
surgical operation on ablating appendix
vermiformis is to be performed. What artery
must be fixed to stop bleeding during the
surgical operation?

A. The colica media

B. The ileocolic artery

C. The colica sinistra

D. The colica dextra

E. The iliac

129. Succinate dehydrogenase catalyses the
dehydrogenation of succinate. Malonic acid
\HOOC CH-i – COOH is used to interrupt
the action of this enzyme. Choose the inhi­
bition type:

A.        Competitive

B. Dephosphorylation

C. Allosteric

p. Limited proteolysis E. Non-competitive

J30. Substitution of the glutamic acid on

valine was revealed while examining initi­al molecular structure. For what inherited pathology is this symptom typical?

A.        Thalassemia

B. Hemoglobinosis
C Sickle-cell anemia

D. Minkowsky-Shauffard disease

E. Favism

131.  A 16 year-old patient got numerous
traumas in automobile accident. Now the
patient is haning a shock. AP – 80/60 mm
Hg. daily urine volume 60-80 ml. What
pathogenic mechanism leads to kidneys
function violation?

A. Trauma of the urinary bladder

B. Increased osmotic pressure in glomerular
capillaries

C.    Decreased   hydrostatic   pressure    in
glomerular capillaries

D. Increased vasopressin blood concentrati­
on

E. Increased pressure in Bowman’s capsule

132.   A 42-year-old man who has been
injured in a car accident is brought into the
emergency room. His blood alcohol level on
admission is 250 mg/dL. Hospital records
show a prior hospitalization for alcohol
related seizures. His wife confirms that he
has been drinking heavily for 3 weeks. What
treatment should be provided to the patient
if he goes into withdrawal?

A. None

B. Phenobarbital

C. Phenytoin

D. Diazepam

E. Pentobarbital

133.  The patient has come to the hospital
from the smelting workshop in the conditi­
on of hyperthermia. What is the direct cause
of loss of consciousness at the heat stroke?

A.        Decrease of heart output

B. Increased water loss through sweating

C. Dilatation of peripheral vessels

D. Arterial pressure drop

E. Decreased brain blood supply

134.  Patient with diabetes mellitus experi­
enced loss of consciousness and convulsions
after an injection of insulin. What might be
the result of biochemical blood analysis for
concentration of sugar?

A.        1,5 mmol/L

B. 5,5 mmol/L

C. 10,0 mmol/L

D. 8,0 mmol/L

E. 3,3 mmol/L

135.   A patient has been taking a mi­
xture prescribed by neuropathologist for

 

Kmk I Medicine 2005

14

 

 

 

neurasthenia for two weeks. The pati­ent feels better but has developed coryza. conjunctivitis, rash, inertia, decrease of memory. She is diagnosed with bromizm. What should be prescribed to decrease the symptoms?

A. Glucose solution 5%

B. Asparcam

C. Natrium chloride

D. Polyglucin
E.-

136. A patient suffering from thyrotoxi­
cosis symptoms of vegetoasthenic syndrome
was revealed. What of the following would
show the histological appearance of a
thyroid gland being stimulated by thyroid-
stimulating hormone (TSH)?

A.   Decreased  numbers  of parafollicular
capillaries

B. Decreased numbers of follicular cells

C. Increased numbers of parafollicular cells

D. An abundance of colloid in the lumen of
the follicle

E. Columnar-shaped follicular cells

137.  A 10-year-old child complains of
weakness, nausea, irritability. Helminthes of
while color and 5-10 mm long were found on
the underwear. On microscopy of the scrape
from the perianal folds achromic ovums of
unsymmetrical form were revealed. Indicate
what helminth is parasiting on the child?

A.        Trichuris

B. Trichina

C. Ascaris lumbricoides

D. Enterobins vermicularis

E. Ancylostoma duodenalis

138. A patient has been brought to the
hospital with the complaints of headache,
pain in left hypochondrium. He has been
ill for 1,5 weeks. The sudden illness began
with the increase of body temperature up
to 39,9°C. In 3 hours the temperature
decreased and hydropoiesis began. The
attacks repeat rhythmically in 48 hours.
The patient had visited one an Afri­
can country. The doctors have suspected
malaria. What method of laboratory di­
agnostics is necessary to use?

A.        Stool examination

B. Blood examination

C. Urine examination

D.   Examination  of vaginal  and  urethral
discharge

E. Immunological tests

139. When a patient with traumatic impai­
rment of the brain was examined, it was di­
scovered that he had stopped to distinguish
displacement of an object on the skin. What

part of the brain was damaged?

A. Occipital zone of the cortex

B. Frontal central gurus
C Posterior central gurus

D. Parietal zone of the cortex

E. Frontal zone

140.   A 19-year-old female suffers from
tachycardia in rest condition, weight loss,
excessive sweating, exophtalmos and irri­
tability. What hormone would you expect to
find elevated in her serum?

A. Mineralocorticoids

B. Cortisol
CACTH

D.        Insulin

E. Thyroxine

141. A 62-year-old patient was admitted to
the neurological department due to cerebral
haemorrage. His condition is grave. There
is evident progression of deep and frequent
breath that turnes into reduction to apnoea
and the cycle repeates. What respiration
type has developed in the patient?

A. Cheyne-Stockes respiration

B. Apneustic respiration
G. Kussmaul respiration

D. Gasping respiration

E. Biot’s respiration

142. The gluconeogenesis is activated in the
liver after intensive physical trainings .What
substance is utilized in gluconeogenesis first
of all in this case:

A. Pyruvate

B. Glucose

C. Alanine

D. Lactate

E. Glutamate

143.   A 50-year-old man has felt vague
abdominal discomfort within past 4
months. Physical examination revealed
no lymphadenopathy, and no abdominal
masses or organomegaly at palpation. Bowel
sounds are heard. An abdominal CT scan
shows a 20 cm retroperitoneal soft tissue
mass obscuring the left psoas muscle. A
stool specimen tested for occult blood is
negative. Which of the following neoplasms
is this man most likely to have?

A. Hamartoma

B. Melanoma

C. Adenocarcinoma

D. Lipoma

E. Lymphoma

144.   Some diseases reveal symptoms of
aldosteronism with hypertension and edema
due to sodium retention in the organism.
What organ of the internal secretion is

 

Krok I Medicine 2005

15

 

 

 

affected on aldosteronism?

A. Adrenal glands

B. Testicle

C. Hypophysis

D. Ovaries

E. Pancreas

145.  The study of the genealogy of a family
with hypertrichosis (helix excessive pilosis)
has demonstrated that this symptom is mani­
fested in all generations only in men and is
inherited by son from his father. What is the
type of hypertrichosis inheritance?

A. X-linked recessive chromosome

B. Autosome-recessive
C Autosome-dominant

D.        X-linked dominant chromosome

E. Y-linked chromosome

146. A 2-year-old child has got intestinal
dysbacteriosis, which results in hemorrhagic
syndrome. What is the most likely cause of
hemorrhage of the child?

A. Vitamin K insufficiency

B. Fibrinogen deficiency
C PP hypovitaminosis

D. Hypocalcemia

E. Activation of tissue thromboplastin

147. A 50 year-old patient had hemorrhage
of the brain and was taken to the hospi­
tal. The place of hemorrhage was revealed
on the lateral hemispheres surfaces during
the medical examination. What artety was
injured ?

A.        The posterior communicating artery

B. The posterior cerebral artery
C The middle cerebral artery

D. The anterior communicating artery

E. The anterior cerebral artery

148.   A microscopic examination of the
enlarged neck gland of a 14-year-old girl
revealed destruction of the tissue structure
of the node, absence of the lymph follicles,
sclerotic and necrosis parts. Cell constitution
of the node is polymorphous, lymphocites,
eosinophilcs. atypical cells of the large si­
ze with multiple-lobule nuclei (Beresovsky-
Shternberg cells) and onenucleus large size
cells are observed. What is the most likely
diagnosis?

A. Fungous mycosis

B. Berkitt’s lymphoma
C Acute lympholeucosis
O. Chronic lympholeucosis
E. Lymphogranulomatous

149. A 40-year-old woman has had a feeli­
ng of abdominal discomfort for the past
8 months. On pelvic examination, there is

the right adnexal mass. Abdominal CT scan demonstrates a 7 cm cystic mass involving the right ovary with small areas of calcifi­cation. The uterus is normal in size. The right fallopian tube and ovary have been removed surgically. Grossly, the mass on sectioning is filled with abundant hair and sebum. Mi­croscopically, the mass has glandular spaces lined by columnar epithelium, squamous epithelium with hair follicles, cartilage, and dense connective tissue. What type of tumour is it?

A.        Teratoma

B. Sarcoma of ovary

C. Squamous cell carcinoma of ovary

D.        Melanoma

E. Metastase of cervical carcinoma

150. A 52 year-old patient with bronchi­
al asthma was treated with glucocorticoids.
Fever reaction appeared as a result of posti-
njective abscess. The patient had subfebri-
le temperature, which didn’t correspond
to latitude and severity of inflammatory
process. Why did patient have low fever
reaction?

A.        Inhibited endogen pyrogens production

B. Thermoregulation center inhibition

C. Inflammatory barrier formation in injecti­
on place

D.        Violation of heat-producing mechanisms

E. Violation of heat loss through lungs

151. A 58-year-old female has undergone
surgery for necrotic bowel. Despite havi­
ng been treated with antibiotics, on
postoperative day 5, she develops symptoms
(fever, hypotension, tachycardia, declining
urine output, and confusion) consistent with
septic shock. What hemodynamic support
would be helpful?

A.        Antibiotic administration

B. Fluid administration

C. Fluids and Dobutamine infusion

D.        Dobutamine infusion

E. Atropine administration

152.   A journalist’s body temperature
has sharply increased in the morning
three weeks after his mission in India,
it was accompanied with shivering and
bad headache. A few hours later the
temperature decreased. The attacks began
to repeat in a day. He was diagnosed with
tropical malaria. What stage of development
of Plasmodium is infective for anopheles-
female?

 

Krok 1 Medicine 2005

16

 

 

 

A. Microgamete

B. Merozoites
C Sporozoites

D. Shizontes

E. Gametocytes

153.     Examination of a person revealed that
minute volume of heart is 3500 mL, systolic
volume is 50 mL. What is the frequency of
cardiac contraction?

A. 80 bpm

B. 50 bpm
C 90 bpm

D. 70 bpm

E. 60 bpm

154.     A patient has elbow joint trauma with
avulsion of medial epicondyle of humerus1.
What nerve can be damaged in this trauma?

A. Musculocutaneous nerve

B. Cardiac cutaneous nerve

C Medial cutaneous nerve of forearm

D. Radial

E. Ulnar

155.     A 16-year-old boy was performed an
appendectomy. He has been hospitalized
for right lower quadrant abdominal pain
within 18 hours. The surgical specimen is
edematous and erythematous. Infiltration
by what of the following cells is the most
typical for the process occuring here?

A. Monocytes

B. Eosinophils

C. Neutrophils

D. Limphocytes

E. Basophils

156.     A 60-year-old patient was diagnosed
with hypothalamic lateral nuclei stroke.
What changes in patient’s behavior may be
expected?

A.        Thirst

B. Unsatisfied hunger

C. The rejection of food

D.        Depression

E. Aggressive behaviour

157. Blood analysis of a patient showed signs
of HIV infection (human immunodeficiency
virus). Which cells does HIV-virus primarily
affect?

A.  Proliferating cells (stem hematoplastic
cells)

B. Cells that contain receptor T4 (T-hclpers)
C Cells that contain receptor IgM (B-
lymphocytes)

D. Specialized nervous cells (neurons)

E. Mast cells

is observed in a patient with glaucoma. Secretion of aqueous humor by the ciliar body is normal. Injury of what structure of the eyeball caused the disorder of flow-out from the anterior chamber?

A. Venous sinus

B. Ciliar body

C. Choroid

D. Back epithelium of cornea

E. Ciliary muscle

159. A 68-year-old woman can not move
her upper and lower right extremities after
stroke. Muscle tone of these extremities and
reflexes are increased. There are pathologi­
cal reflexes. What form of the paralysis is it?

A. Monoplegia

B. Dissociation

C. Hemiplegia

D. Tetraplegia

E. Paraplegia

160.    Where should the catheter for evacuati­
on of the lymph from the thoracic lymph
duct be inserted?

A.       To the left venous corner

B. To the right venous corner

C. To the left inguinal vein

D.       To the inferior vena cava

E. To the superior vena cava

161.    M-r S presents all signs of the hepatic
coma: loss of consciousness, absence of
reflexes, cramps, convulsion, disorder of
heart activity, recurrent (periodical) respi­
ration. What are cerebrotoxical substances
which accumulate in blood under hepar
insufficiency?

A. IL-1

B. Necrosogenic substances

C. Ketonic body

D. Ammonia

E. Autoantibody

162.    During the endoscopy the inflammation
of a major papilla of the duodenum and the
disturbances of bile secretion were found. In
which’ part of duodenum were the problems
found?

A. Bulb

B. Descendent part

C.  Ascendant part

D. Upper horizontal part

E. Lower horizontal part

.•163. The calcium canals of cardiomyocytes have been blocked on an isolated rabbit’s heart. What changes in the heart’s activity can happen as a result?

 

158.   The   increased   intraocular   tension

 

Krok 1 Medicine 2005

17

 

 

 

A. Decreased heart beat rate

B. Heart stops in diastole

C. Heart stops in systole

D. Decreased rate and force of heart beat

E. Decreased force of the contraction

164.  Analeptical remedy of reflective type
from the H-cholinomimetics group was gi­
ven to the patient for restoration of breathi­
ng after poisoning with carbon monoxide.
What medicine was prescribed to the pati­
ent?

A. Lobeline hydrochloride

B. Mesaton

C Adrenalin hydrochloride

D. Pentamin

E. Atropine sulphate

165.  A 45-year-old man fell on the right
knee and felt the acute pain in the joint. On
examination: severe edema on the anterior
surface of the knee joint. Crunching sounds
are heard while moving the joint. Which
bone is destroyed?

A. Neck of the thigh bone

B. Knee-cap

C. Head of the thigh bone

D. Left epicondyle of the thigh

E. Right epicondyle of the thigh

166.  During histological examination of the
stomach it was found out that glands contai­
ned very small amount of pariental cells or
they were totally absent. Mucose membrane
of what part of the stomach was studied?

A. Fundus of stomach

B. Cardia

C Pyloric part

D. Body of stomach

E.-

167.   Decreased blood supply to the organs
causes hypoxia that activates fibroblasts
function. Volume of what elements is
increased in this case?

A. Lymphatic vessels

B. Intercellular substance

C. Nerve elements

D. Vessels of microcircular stream

E. Parenchymatous elements of the organ

168.  A damage of the atomic power plant
reactor resulted in the run-out of radi-
oelcments. People in the ‘ superstandard
radiation zone were radiated with approxi­
mately 250-300 r. and were immediately
hospitalized. What changes in the blood
count would be typical?

A. Lymphopenia

B. Anemia

C. Neutropenia

D. Leukopenia

E. Thrombopenia

169.  A patient with suspicion on epidemic
typhus was admitted to the hospital. Some
arachnids and insects have been found in his
flat. Which of them may be a carrier of the
pathogen of epidemic typhus?

A. Cockroaches

B. Bed-bugs

C. Lice

D. Spiders

E. Houseflies

170.  A man died 8 days after the beginni­
ng of the disease. He was diagnosed with
dysentery. At the autopsy it was found out a
thickened wall of the sigma and rectum, fi­
brinous membrane on the surface of mucous
membrane. Histologically: there is a deep
necrosis of mucous membrane with infiltrati­
on of necrotic masses with fibrin. What kind
of colitis does correspond to the changes?

A.        Catarrhal

B. Chronic

C. Diphtheritic

D. Gangrenous

E. Ulcerative

171.  A 55-year-old patient with continui­
ng ventricular arrhythmias was admitted to
the hospital. The patient is taking timolol
drops for glaucoma, daily insulin injections
for diabetes mellitus, and an ACE inhibitor
for hypertension. You have decided to use
phenytoin instead of procainamide. What is
the reason?

A.        The local anesthetic effect of procainami­
de would aggravate the hypertension

B. The hypertensive effects of procainamide
would aggravate the hypertension

C. The cholinergic effects of procainamide
would aggravate the diabetes

D. The local anesthetic effect of procainami­
de would potentiate diabetes

E. The anticholinergic effect of procainamide
would aggravate glaucoma

172.  In the blood of a 26-year-old man
18% of erythrocytes of the spherical, ball-
shaped, flat and thorn-like shape have been
revealed. Other eritrocytes were in the form
of the concavo-concave disks. How is this
phenomenon called?

A.        Pathological poikilocytosis

B. Physiological anisocytosis

C. Erytrocytosis

D. Pathological anisocytosis

E. Physiological poikilocytosis

 

Krok 1 Medicine 2005

18

 

 

 

173.    An individual is characterized by
rounded face, broad forehead, a mongoli­
an type of eyelid fold, flattened nasal bri­
dge, permanently open mouth, projecting
lower lip, protruding tongue, short neck, flat
hands, and stubby fingers. What diagnosis
can be put to the patient?

A. Klinefelter’s syndrome

B. Down’s syndrome

C. Turner’s syndrome

D. Supermales

E.  Alkaptonuria

174.      A patient with clinical signs
of immunodeficiency has unchanged
number and functional activity of
T and B lymphocytes. Dysfunction’s
defect of antigen-presentation to the
immunocompetent cells was found during
investigation on the molecule level. Defect
of what cells is the most probable here?

A. Macrophages, monocytes

B. NK-cells

C. O-lymphocytes

D. T-lymphocytes, B-lymphocytes

E.         Fibroblasts,      T-lymphocytes,      B-
lymphocytes

175. Oval and round organelles with double
wall are seen at the electron micrograph.
The outer membrane is smooth, the inner
membrane folded into cristae contain
enzyme ATPase synthetase. These are:

A.  Centrioles

B.  Golgi complex

C Ribosomes

D.  Lysosomes

E.  Mitochondria

176.      During complicated labour the
symphysis pubis ruptured. What organ can
be damaged mostly?

A.  Ovaria

B.  Rectum

C Uterine tubes

D.  Urinary blader

E.  Uterus

177.   A 59-year-old man has symptoms
of parenchymatous jaundice and portal
hypertension. Histological examination
of the puncture of the liver bioptate
has revealed an affected beam-lobule
structure, part of hepatocytes has signs of
fat dystrophy, port-portal connective tissue
septa with formation of pseudo-lobules, with
periportal lympho-macrophage infiltrations.
What is the most probable diagnosis?

A.  Alcohol hepatitis

B.  Viral hepatitis

C.  Chronic hepatosis

D.  Liver cirrhosis

E.  Toxic dystrophy

178.  A 18-year-old patient came to the out­
patient department with the complaints of
bleeding trauma in the vestibule of his nose.
On examination: the mechanical injure of
the mucous layer of the vestibule without
continuation into nasal cavity proper. What
is the boundary between the vestibule and
nasal cavity proper?

A.  Nasal septa

B.  Nasal roller
C Nasal limen

D.  Choanes

E.  Nostrils

179.       The     high     level     of     Lactate
Dehydrogenase (LDH) isozymes concentrati­
on showed the increase of LDH-1 and LDH-
2 in a patient’s blood plasma. Point out the
most probable diagnosis:

A.  Viral hepatitis

B.  Diabetes mellitus

C.  Skeletal muscle dystrophy

D.  Myocardial infarction

E.  Acute pancreatitis

180. A patient visited a dentist with complai­
nts of redness and edema of his mouth
mucous membrane in a month after dental
prosthesis. The patient was diagnosed wi­
th allergic stomatitis. What type of allergic
reaction by Gell and Cumbs underlies this
disease?

A.  Anaphylactic

B.  Stimulating

G Immunocomplex

D.  Delayed type hypersensitivity

E.  Cytotoxic

181.    To anaesthetize the manipulation
related to burn surface treatment, a patient
was intravenously injected a medication for
short-acting narcosis. 1 minute later the pati­
ent being under anaesthesia had increased
blood pressure, tachycardia, increased tone
of sceletal muscles; reflexes remained. After
awakening the patient had desorientation
and visual hallucinations. What medication
was the patient injected?

A.  Sombrevin

B.  Diethyl ether
C Nitrous oxide

D.  Thiopental sodium

E.  Ketamine

182. During the experiment on the influence
of chemical substances in the muscles the

 

 

Krokl Medicine 2005

19

 

 

 

reaction of Ca2+ -pump is weakened. Which phcnomcnum will be observed?

A. Activation of the sodium-potassium pump

B. Prolonged duration of the AP

C. Decreased velocity of the AP distribution

D. Decreased AP

E. Prolonged relaxation

183. Intrapleural pressure is being measured
in a person. In what phase does a person
hold his breath if the pressure is – 25 cm

HoO?

A. Forced-inspiration

B.-

C Forced expiration

D. Quiet expiration

E. Quiet inspiration

184.      Hydroxylation of endogenous
substrates and xenobiotics requires a donor
of protons. Which of the following vitamins
can play this role?

A.        Vitamin A

B. Vitamin Be

C. Vitamin P

D.        Vitamin C

E. Vitamin E

185.   Electrocardiogram of a 45-year-old
man showed absence of P-wave in all the
leads. What part of the conducting system is
blocked?

A. Branches of the bundle of His

B. Atrioventricular node

C. Sinu-atrial node

D. Purkinje’s fibres

E. Common branch of the bundle of His

186.  A woman who was infected with
toxoplasmosis during the pregnancy has a
child with multiple congenital defects/This is
a result of:

I A. Biological mutagenesis

B. Cancerogenesis

C. Recombination

D. Teratogenesis

E. Chemical mutagenesis

187.     At the aboratory experiment
the eukocyte culture was mixed wi­
th staphylococci. Neutrophile leukocytes
engulfed and digested bacterial cells. This
processes are termed:

A. Diffusion

B. Pinocytosis
C Osmosis

D. Phagocytosis

E. Facilitated diffusion

188. There is an inhibited coagulation in the
patients with bile ducts obstruction, bleedi-

ng due to the low level of absorbtion of a vitamin. What vitamin is in deficiency?

A.K                                                        8

B. Carotene

CD

D.A

E.E

189. A student is thoroughly summarising a
lecture. When his groupmates begin talki­
ng the quality of the summarising worsens
greatly. What type of inhibition in the
cerebral cortex is the cause of it?

A.        External

B. Differential

C. Dying

D.        Delayed

E. Protective

190.  A lung of a premature infant is
presented on electronic photomicrography
of biopsy material. Collapse of the alveolar
wall caused by the deficiency of surfactant
was revealed. Disfunction of what cells of
the alveolar wall caused it?

A.        Fibroblasts

B. Alveocytes type I

C. Alveocytes type II

D.        Alveolar macrophages

E. Secretory cells

191. Galactosemia has been revealed in a
child. Concentration of glucose in the blood
has not considerably changed. What enzyme
deficiency caused this illness?

A.        Amylo-l,6-glucosidase

B. Hexokinase

C. Galactose-1-phosphate uridyltransferase

D. Galactokinase

E. Phosphoglucomutase

192. A 50-year-old male farm worker has
been brought to the emergency room. He
was found confused in the orchard and since
then has remained unconscious. His heart
rate is 45 and his blood pressure is 80/40 mm
Hg. He is sweating and salivating profusely.
Which of the following should be prescri­
bed?

A.        Proserine

B. Atropine

C. Physostigminc

D.        Norepinephrine

E. Pentamine

193.  In case of enterobiasis acrihine – the
structural analogue of vitamin Bi – is admi­
nistered. The synthesis disorder of which
enzymes does this medicine cause in mi­
croorganisms?

 

Krok 1 Medicine 2005

20

 

 

 

A. FAD-dependent dehydrogenases

B. Aminotransferases

C. Cytochromeoxidases

D. Peptidases

E. NAD-dependet dehydrogenases

194.  Usually the intravenous injection is
done into median cubital vein because it is
slightly movable due to fixation by the soft
tissues. What does it fix in the cubital fossa?

A. Brachioradial muscle

B. Aponeurosis of biceps muscle

C. Brachial muscle

D. Tendon of the triceps muscle

E. Anconeus muscle

195. The B cells of endocrine portion of
pancreas are selectively damaged oy alloxan
poisoning. How will it be reflected in blood
plasma?

A. The content of globulins decreases

B. The content of fibrinogen decrease

C. The content of sugar increases

D. The level of sugar decreases

E. The content of albumins decreases

196.     Patient 54 year-old, 5th day after surgi­
cal operation. Blood count: Erythrocytes
3,6*1012/1, Hemoglobin 95 g/1, Erythrocyte’s
hemoglobin content (color index) 0,78;
Leukocytes 16 * 109/1, Platelets 450 * 109/1
Blood picture: anizocytosis, poikilocytosis,
reticulocytes- 3,8%. What anemia does this
patient have?

A.        Hypoplastic anemia

B. Acquired hemolytic anemia

C. Anemia from iron deficiency

D.        Chronic posthemorragic anemia

E. Acute posthemorragic anemia

197. A 46 year-old patient has complained
of headache, fatigue, thirst, pains in the spi­
ne and joints for the last 2 years. Clinically
observed disproportional enlargement of
hands, feet, nose, superciliary arches. He
notes that he needed to buy bigger shoes
three times. What is the main reason of such
disproportional enlargement of different
parts of the body?

A. Joints chronic inflammation development

B.  Increased sensitivity of the tissues to
growth hormone

C. Joints dystrophy development

D.  Increased sensitivity of the tissues to
insulin

E.  Cartilaginous tissue proliferation under
growth hormone influence

198. A woman suffering from dysfunctional
metrorrhagia was made a diagnostic aborti­
on. Histologically in the scrape there were a
lot of small stamped glandulars covered with
multirowed epithelium. The lumens of some
glandulars were cystically extended. Choose
the variant of general pathologic process in
the endometrium.

A. Neoplasm of endometrium

B.      Glandular-cystic      hyperplasia      of
endometrium

C. Atrophy of endometrium

D. Hypertrophic growth

E. Metaplasia of endometrium

199.     A sick man with high temperature and
a lot of tiny wounds on the body has been
admitted to the hospital. Lice have been
found in the folds of his clothing. What di­
sease can be suspected in the patient?

A. Plague

B. Malaria
C Tularemia

D. Scabies

E. Epidemic typhus

200.     A patient suffering from trombophlebi-
tis of deep veins suddenly died. The autopsy
has shown freely lying red friable masses
with dim crimped surface in the trunk and
bifurcation of the pulmonary artery. What
pathologic process was revealed by the
morbid anatomist?

A.        Tissue embolism

B. Thrombosis

C. Embolism with foreign body

D.        Tromboembolism

E. Fat embolism

 

t o

INSTRUCTIONAL BOOK

 

 

Ministry of public health of Ukraine (MPH of Ukraine) loard of education and science of MPH of Ukraine esting Board

 

TEST ITEMS FOR LICENSING EXAMINATION: KROK I. MEDICINE. 2005.

Composers:              Artemchuk’ Ludmila

Bulakh Iryna Homchenkova Natalya

 

Kyiv. Testing Board. (English language).

Editor A.Potapova

Technical editor: P.Ostapenko, V.Zhdanov

Approved to print 06.10.2005. <DopiuaT 60×84 1/8 Offset paper. Typeface. Times New Roman Cyr. Offset print. Conditional print pages 12. Accounting publishing pages 12. Issue. 470 copies

 

MINISTRY OF PUBLIC HEALTH OF UKRAINE

Department of manpower policy, education and science of MPH of Ukraine

 

Krok 1

•medicine

2006

 

General Instruction Every one of these numbered questions or unfinished statements in this chapter corresponds to answers or statements endings. Choose the answer (finished statements) that fits best and fill in the circle with the corresponding Latin letter on the answer sheet.

Authors of items: Abramets 1.1., Andreeva V.F., Andronov D.YU., Arnautova L.V., Basiy R.V., Byelousova LP., Bilets M.V., Borisenko B.O., Brazhnikov A.M., Bruk B.B., Vasilenko I.V., Vaschuk A.A., Verchova O.O., Visockiy I.YU., Vlasenko M.S., Vlasenko O.V., VovkV.V., Volohova CO.. Vorobec Z.D.,   Gaydash I.S., Gargin V.V., Garets B.I., Goldobina O.V., Gorgol N.I., Grebenik L.I., Gudivok YA.S., Gurcalova I.P., Davidenko I.S., Dem’yanenko I.A., Demchenko O.M., Dyelcova O.I., Yelskiy V.M., Zhadinskiy M.V., Zhilyuk V.I., Zhulinskiy V.O., Zagorulko O.K., Zelenina N.M., Zlenko O.T., Ivascnko A.V., Izmaylova L.V., Kaplyuh R.F., Kir”yakulov G.S., Klincova S.P., Klish I.M., Klopockiy G.A., Koldunov V.V., Komissarov I.V., Korolenko G.S., Korol A.P., Kosik O.G. Krushinska T.YU., Kryuk YU.YA., Kubishkin A.V., Kuzik I.YA., Kukovska I.L., Kulachenko B.V., Kulikova N.A., Kulitka E.F., Kulish A.S., Lazarev K.L.,   Linchevska L.P., Loiytra A.O., Lukovich I.M., Lucyuk M.B., Luchko I.M., Makats Ye.F., Makovetskiy O.V., Matyushenko P.M., Michaiylyuk I.O., Mishalov V.D., Mischenko K.M., Mozhayeva O.B.,   MotaO.M.,   NavnikoT.S., Nechiporenko G.V., Nikonova A.A., Nikolenko O.G., Nikolenko YU.I. Nikonova A.O., Okulov V.I., Pentyuk O.O., Piddubnyak YU.G., Poguda A.A., Popova L.D., Primova L.O., Pushkar M.S., Romanenko M.I., Savchenko N.V., Samura I.B., Svislun YU.D., Serdyuchenko I.YA., Sidorchuk I.Y., Skorobagata T.G., Skorobogatova Z.M., Skrebkova O.YU., Skripka O.V., Smirnova T.V., Smolyakova G.M., Sokolovska L.V., Starlichanova L.D., Steblyuk M.V., Stepanyuk G.I., Stec V.R., Tananakina T.P., Tatarko S.V., Tertishniy S.I., Titarenko V.M., Tomashova S.A., Tumas’yants N.YU., Ugrin O.M., Fartushok N.V., Fedonyuk YA.I., Fedorchenko O.YE., Fedorchenko O.V., Feketa V.P., Filipec N.D., Halaturnik G.M., Holodkova O.L., CherVyak M.M., Cherkasov V.G.,   Cherkashina L.P.,   Chemenko G.P., Chesnokova M.M., Shalanin V.V., Shanko V.M., Shevtsova A.I., Shemonaeva K.F., Shpak S.I., Scherbakov S.M., Scherbakov C.N., Yakubishina L.V., Yasinskiy V.I. and Committees of professional expertise.

Item reviewers. Andrycyeva V.F., Basiy R.V., Borzenko B.G., Visotskiy I.YU., Vinnikov YU.M., Golovatyuk O.L., Gorgol N.I., Deyneka S.YE., Dyeltsova O.I., Zhadinskiy M.V., Zelenina N.M., Zinkovska L.YA., Yoltuhivskiy M.V., Kalikik K.G., Kalugina S.M., Kirichek L.T., Kiryuhin I.F., Kovalchuk L.YE., Korolcnko G.S., Krushinska T.YU., Linchevska L.P., Lyubinets L.A., Mrug V.M., Nikolenko O.I., Ovchinnikov S.O., Ostrovska S.S., Pentyuk O.O., Pototska I.I., Pushkar M.S., Sikora V.Z., Teleshova O.V., Trzhetsinskiy S.D., Fedechko IY.M., Fomina L.V., Chernenko N.A.

Composers. Artemchuk L., Bulakh I., Homchenkova N.

The book includes test items for use at licensing integrated examination “Krok 1. Medicine” and further use in teaching.

The book has been developed for students of medical, pediatric and mcdical-and-prophy lactic faculties and academic staff of higher medical educational establishments.

Approved by Ministry of Public Health of Ukraine as examination and teaching publication based on expert conclusions (Orders of MPH of Ukraine of 14.08.1998 ISSSb °i27A2A9W ^303, of 18.06.2002 JV«221, of 16.10.2002 JVb374, of 16.04.2003 Jfe239, of 29.05.2003 JV«233).

© Copyright

Testing Board, 2006.

 

910406

I

 

 

 

1. A 46-year-old patient has complained of headache, fatigue, thirst, pains in the spi­ne and joints for the last 2 years. Clinically observed disproportional enlargement of hands, feet, nose, superciliary arches. He notes that he needed to buy bigger shoes three times. What is the main reason of such disproportional enlargement of different parts of the body?

A.   Increased sensitivity of the tissues to
insulin

B. Joints chronic inflammation development

C.  Cartilaginous tissue proliferation under
growth hormone influence

D.  Increased sensitivity of the tissues to
growth hormone

E. Joints dystrophy development

2. Low level of albumins and fibrinogen was
detected in the patient’s blood. Decreased
activity of what organelle of the liver
hepatocytes can most probably cause it?

**   j.

A. Lysosomes

B. Granular endoplasmatic reticulum

C. Agranular endoplasmatic reticulum

D. Golgi complex

E. Mitochondrions

3. A 55-year-old patient with continuing
ventricular arrhythmias was admitted to
the hospital. The patient is taking timolol
drops for glaucoma, daily insulin injections
for diabetes mellitus, and an ACE inhibitor
for hypertension. You have decided to use
phenytoin instead of procainamide. What is
the reason?

A.  The cholinergic effects of procainamide
would aggravate the diabetes

B. The anticholinergic effect of procainamide
would aggravate glaucoma

C. The local anesthetic effect of procainami­
de would potentiate diabetes

D. The hypertensive effects of procainamide
would aggravate the hypertension

E. The local anesthetic effect of procainami­
de would aggravate the hypertension

4. Diagnostic scraping was performed to the
woman with dysfunctional uterine bleedi­
ng. Multiple convoluted glands, gangli-
ally dilated lumens of some glands were
revealed histologically in the scrape. Name
the type of general pathological process in
endometry:

A. Atrophy

B. Hyperplasia glandulocystica
C Hypertrophic excrescence

D. Displasia

E. Metaplasia

5. Synthesis of phospholipids is disturbed
as a result fatty infiltration of liver. Indi-

cate which of the following substances can enhance the process of methylation during phospholipids synthesis?

A. Ascorbic acid

B. Citrate

C. Methionine

D. Glucose

E. Glycerin

6.   The alternate usage of dichlotiazide,
etacrin acid and lasex did not influence
diuretically upon the patient with marked
peripheral edemata. The aldosterone rate in
the blood is increased. Indicate which medi­
cine should be prescribed:

A. Clopamid

B. Urea

C. Amilorid

D. Mannit

E. Spironolacton

7.  A patient with diabetes mellitus experi­
enced loss of consciousness and convulsi­
ons after injection of insulin. What is the
result of biochemical blood analysis for
concentration of the sugar?

A.                                                             5,5 mmol/L

B. 10,0 mmol/L

C. 3,3 mmol/L

D. 8,0 mmol/L

E. 1,5 mmol/L

8.   A 40-year-old patient complains of
intensive heartbeats, sweating, nausea, vi­
sual impairment, arm tremor, hypertensi­
on. From his anamnesis: 2 years ago he
was diagnosed with pheochromocytoma.
Hyperproduction of what hormones causes
the given pathology?

A.        Thyroidal hormones

B. Catecholamines

C. Glucocorticoids
D.ACTH

E. Aldosterone

9.  A 34-year-old patient was diagnosed wi­
th chronic glomerulonephritis 3 years ago.
Edemata have developed within the last 6
monthes. What caused the edemata?

A. Liver dysfunction of protein formation

B. Hyperaldosteronism

C. Hyperproduction of vasopressin

D. Proteinuria

E. Hyperosmolarity of plasma

10.   A 60-year-old patient fell ill with
obturative jaundice as a result of malignant
tumour of the big papillary of the duodenal.
Lumen of what anatomical structure is
compressed by tumour?

 

910406

 

 

 

 

A. Cystic duct

B. Left hepatic duct
C Right hepatic duct

D. Common hepatic duct

E. Hepatopancreatic ampulla

11.   In course of metabolic process active
forms of oxygen including superoxide anion
radical are formed in the human body. By
means of what enzyme is this anion inacti­
vated?

A. Glutathioneperoxidase

B. Glutathionereductase
C Catalase

D. Peroxidase

E. Superoxide dismutase

12.  A 68-year-old woman can’t move her
upper and lower right extremities due to
the stroke. Muscle tone of these extremiti­
es and their reflexes are increased. There are
pathological reflexes. What form of paralysis
is it?

A.       Monoplegia

B. Tetraplegia

C. Dissociation

D. Paraplegia

E. Hemiplegia

13.   A physician examined a patient and
found inguinal hernia. Through what
anatomic formation does it penetrate into
the skin?

>    *:<;.’.”}.;

A.        Canalis adductorius

B. Anulus femoralis

C. Hiatus saphenus

D. Anulus inguinalis superficialis

E. Lacuna musculorum

14.   An 18-year-old patient has enlarged
inguinal lymphnodes, they are painless, thi­
ckened on palpation. In the area of geni­
tal mucous membrane there is a small-
sized ulcer with thickened edges and
“laquer” bottom of greyish colour. What is
the most probable diagnosis?

A.        Tuberculosis

B. Lepra

C. Trophic ulcer

D.        Syphilis

E. Gonorrhea

15.   A person was selling “homemade
pork”sausages at the market. State sani­
tary inspector suspected falcification of the
sausages. What serological immune reaction
can identifiy food substance?

A. Complement-fixation test

B. Agglutination test

C. Indirect hemagglutination test

D. Immunofluorescence test

E. Precipitation test

16. During the operation on the hip joint of a
5-year-old child her ligament was damaged
which caused bleeding.What ligament was
damaged?

A.       Ischiofemoral

B. The head of the thigh

C Perpendicular of the acetabule

D. Pubofemoral

E.  Iliofemoral

17. While preparing a patient to the operati­
on the neart chambers’ pressure was
measured. In one of them the pressure
changed during one heart cycle from 0 to
120 mm Hg. What chamber of heart was it?

A. Right ventricle

B. Left atrium
C Left ventricle
D.-

E. Right atrium

18.   A genetics specialist analyzed the
genealogy of a family and found that both
males and females may have the illness, not
across all the generations, and that healthy
parents may have ill children. What is the
type of illness inheritance?

A.       Autosomal recessive

B. Y-linked

C. Autosomal dominant

D. X-linked dominant

E. X-linked recessive

19. Tuberculine was introduced intracutaneously
to the child for tuberculin test. Marked
hyperemia, tissue infiltration developed on
the place of injection in 24 hours. What
mechanism caused these modifications?

A.        Cell cytotoxity

B. Antibody cytotoxity

C. Immunocomplex cytotoxity

D. Reagin type cytotoxity

E. Granuloma formation

20.  A patient has got a spasm of smooth
muscles of bronchi. Activators of what
membrane cytoreceptors are phisiologically
reasoned to stop an attack?

A.        /3-adrenoreceptors

B. M-cholinoreceptors

C. o-adrenoreceptors

D. n- and /3-adrenoreceptors

E. H-cholinoreceptors

21.  A 17-year-old boy fell seriously ill,
body temperature rose up to 38,5°C, there

 

910406

3

 

 

 

is cough, rhinitis. lacrimation. nasal di­scharges. What kind of inflammation is it?

A. Serous inflammation

B. Hemorrhagic inflammation
G. Catarrhal inflammation

D. Suppurative inflammation

E. Fibrinous inflammation

22.    A 50-year-old man has felt vague
abdominal discomfort within past 4
months. Physical examination revealed
no lymphadenopathy, and no abdominal
masses or organomegaly at palpation. Bowel
sounds are heard. An abdominal CT scan
shows a 20 cm retroperitoneal soft tissue
mass obscuring the left psoas muscle. A
stool specimen tested for occult blood is
negative. Which of the following neoplasms
is this man most likely to have?

A. Lipoma

B. Lymphoma
C Hamartoma

D. Adenocarcinoma

E. Melanoma

23.    Inflammatory process of modified
subserous layer around cervix of the uterus
caused an intensive pain syndrome. In what
region of genitals does the pathological
process take place?

A. Mesometrium

B. Perimetrium

C. Myometrium

D. Endometrium

E. Parametrium

24.   Under some diseases it is observed
aldosteronism accompanied by hypertensi­
on and edema due to sodium retention in
the organism. What organ of the internal
secretion is affected under aldosteronism?

A. Ovaries

B. Pancreas

C. Testicle

D. Hypophysis

E. Adrenal glands

25.  Concentration of pyruvate is increased
in the patient’s blood, the most of which is
excreted with urine. What avitaminosis has
the patient?

A. Avitaminosis E

B. Avitaminosis B|

C. Avitaminosis Bo

D. Avitaminosis B2

E. Avitaminosis B3

26.  A considerable increase of activity of
MB-forms of CPK (creatinephosphokinase)
and LDH-1 was revealed on the examinati­
on of patient’s blood. What is the most likely

pathology?

A. Cholecystitis

B. Pancreatitis

C. Rheumatism

D. Miocardial infarction

E. Hepatitis

27.   Different functional groups can be
presented in the structure of L-amino aci­
d’s radicals. Identify the group that is able to
form ester bond:

A.-OH B.-SH

C.  -NH2                                           t

D. -CONH2

E. -C.H3

28. Blood analysis of a patient showed signs
of HIV infection (human immunodeficiency
virus). Which cells does HIV-virus primarily
affect?

A.  Proliferating cells (stem hematoplastic
cells)

B. Specialized nervous cells (neurons)

C. Mast cells

D. Cells that contain receptor T4 (T-helpers)

E.   Cells  that  contain  receptor IgM  (B-
lymphocytes)

29.   Patient with injured muscles of the
lower extremities was admitted to the
traumatological department. Due to what
cells is reparative regeneration of the muscle
fibers and restoration of the muscle function
possible?

A.        Myoepithelial cells

B. Myoblasts

C. Satellite-cells

D. Myofibroblasts

E. Fibroblasts

30.   Objective examination of a patient
revealed: slender figure, big skull, highly
developed frontal region of face, short
extremities. What constitutional type is it
characteristic for?

A. Cerebral

B. Digestive

C. Mixed

D. Muscular

E. Respiratory

31. A sick man with high temperature and a
lot of tiny wounds on the body has been
admitted to the hospital. Lice have been
found in the folds of his clothing. What di­
sease can be suspected?

 

910406

4

 

 

 

A. Epidemic typhus

B. Plague

C Tularemia

D. Scabies

E. Malaria

32.  Slime, blood and protozoa 30-200 mi­
crons long have been revealed in a man’s
feces. The body is covered with cilias and has
correct oval form with a little bit narrowed
anterior and wide round shaped posterior
end. At the anterior end a mouth is visi­
ble. In cytoplasm there are two nucleui and
two short vacuoles. What are the described
features typical for?

A. Trichomonas

B.  Lamblia

C.  Balantidium

D. Dysenteric amoeba

E. Intestinal amoeba

33.  An old woman was hospitalized with
acute pain, edema in the right hip joint; the
movements in the joint are limited. Which
bone or part of it was broken?

A.       Pubic bone

B. Condyle of the thigh
C The neck of the thigh

D. The body of the thigh bone

E. Ischial bone

34. A student is writing a thorough summary
of a lecture. Quality of summarizing has
considerably worsened when his neighbours
started talking. What kind of inhibition in
the cerebral cortex caused this effect?

A. Fading

B. Differentiated
C Delayed

D. External

E. Protective

35. A 39-year-old woman has madescence in
the region of mammilla, a small ulcer wi­
th inflammatory hyperemia and cutaneous
edema. Histologic examination of tissue
sampling from this area revealed in the
malpighian layer of thickened epidermis
atypical cells with light and optically empty
cytoplasm, with no intracellular bridges.
Such cells were also found in the orifice of
big mammal gland ducts. What is the most
probable diagnosis?

A. Paget’s disease

B. Mclanocarcinoma

C. Epidermoid cancer

D. Intraductal cancer

E. Basal cell carcinoma

36. Analeptical remedy of reflective type
from the H-cholinomimetics group was gi­
ven to the patient for restoration of breathi-

ng after poisoning with carbon monoxide. What medicine was prescribed to the pati­ent?

A* Pentamin

B. Atropine sulphate

C. Adrenalin hydrochloride

D.       Lobeline hydrochloride

E. Mesaton

37.  A 46-year-old man complains of di­
fficulties with nasal breathing. Mikulicz’s
cells, accumulation of epithelioid cells,
plasmocytes, lymphocytes, hyaline balls
were discovered in the biopsy material of the
thickened nasal mucosa. What is the most li­
kely diagnosis?

A. Allergic rhinitis

B. Scleroma

C. Virus rhinitis

D. Meningococcal nasopharyngitis

E. Rhinovirus infection

38.  The ovary specimen colored with
hematoxylin-eosin contains a follicle, in whi­
ch cubic-shaped follicle epithelium cells are
placed in 1-2 layers, and scarlet membrane is
seen around the ovocyte. Name this follicle:

A. Atretic

B. Primary

C. Secondary

D. Mature

E. Primordial

39.  Autopsy revealed that right lung is   J
enlarged, solid, there are fibrin layers on
the pleura. Lung tissue is light grey color on
incision with muddy liqued exudates. What
lung disease are these symptoms typical for?

A. Fibrosing alveolitis

B. Bronchopneumonia
C Croupous pneumonia

D. Interstitial pneumonia

E. Pulmonary gangrene

40. The pulmonalis embolism has suddenly
developed in a 40-year-old patient with
opened fracture of the hip. Choose the possi­
ble kind of embolism:

A. Foreign body

B. Thrombus-embolus
G Tissue

D.Fat E.Air

41. A 27-year-old woman has dropped peni­
cillin containing eye drops. In a few minutes
there appeared feeling of itching, burning
of the skin, lips and eyelids edema, whistling
cough, decrease of BR What imunoglobulins
take part in the development of this allergic
reaction?

 

910406                                                                   5

 

A. IgE and IgG

B. IgG and IgD

C. IgA and IgM

D. IgM and IgD

E. IgM and IgG

42. A pregnant woman had been haying
toxicosis with severe repeated vomiting
for 24 hours. In the end of the day there
appeared tetanic convulsions and fluid loss,
what shift of acid-base state caused these
changes?

A. Excretory acidosis

B. Gaseous alkalosis

C. Metabolic acidosis

D. Excretory alkalosis

E. Gaseous acidosis

43. The calcium canals of cardiomyocytes
have been blocked on an isolated rabbit’s
heart. What changes in the heart’s activity
can result from it?

A. Decreased rate and force of heart beat

B. Heart stops in diastole

C. Decreased heart beat rate

D. Decreased force of the contraction

E. Heart stops in systole

44. Microscopic examination of the enlarged
neck gland of a 14-year-old girl revealed
destruction of the tissue structure of
the node, absence of the lymph folli­
cles, sclerotic areas and necrosis foci, cell
constitution of the node is polymorphous,
lymphocites, eosinophiles, big atypical
cells with multilobular nuclei (Beresovsky-
Sternberg cells) and mononuclear cells of
the large size are present. What is the most
likely diagnosis?

A.        Berkitt’s lymphoma

B. Lymphogranulomatosis

C. Fungoid mycosis

D.        Acute lympholeucosis

E. Chronic lympholeucosis

45. A young man has a painlessl formation
without marked borders in the soft tissues of
his thigh. On the tissue bioptate the formati­
on looks like flesh of fish and consists of
immature fibroblast-like cells with multiple
mitosis growing through the muscles. What
is the most likely diagnosis?

A. Fibroma

B. Myosarcoma
C Cancer

D.                                                             Myoma

E. Fibrosarcoma

46.  Diabetes mellitus causes ketosis as a
result of activated oxidation of fatty aci­
ds. What disorders of acid-base equilibrium
may be caused by excessive accumulation of

ketone bodies in blood?

A. Metabolic alcalosis

B. Metabolic acidosis

C. Respiratory alcalosis

D. Any changes woun’t happen

E. Respiratory acidosis

47. A 56-year-old patient complaining of thi­
rst and frequent urination was diagnosed
with diabete mellitus. Butamin was prescri-
bed. How does the medicine act?

A.  It inhibits absorption of glucose in the
intestines

B. It helps to absorb the glucose by the cells
of the organism tissues

C It inhibits a-cells of Langergans’ islets D. It relieves transport of glucose through the cells’ membranes E.’ It stimulates /3-cells of Langergans’ islets

48.       Children often have heavy nasal breathi­
ng resulting from excessive development
of lymphoid tissue of pharyngeal mucous
membrane. What tonsils growth may cause
this effect?

A. Tonsilla palatina

B. All above mentioned tonsils
C Tonsilla tubaria

D. Tonsilla pharyngea

E. Tonsilla lingualis

49.       A patient with continious bronchopneumonia
was admitted to the therapeutic department.      \
Antibiotic therapy didn’t give much effect.
What   medication   for   improvement   of
immune  state should  be  added  to  the
complex treatment of this patient?

A. Analgin

B. Paracetamol

C. Benadryl

D. Timaline

E. Sulfocamphocaine

50.       A patient with hypochromic anemia has
splitting hair and loss of hair, increased nail
brittling and taste alteration. What is the
mechanism of the development of these
symptoms?

A. Decreased production of parathyrin

B. Deficiency of vitamin A

C.     Decreased    production    of    thyroid
hormones

D. Deficiency of iron-containing enzymes

E. Deficiency of vitamin B12

51.  A patient’s blood was analyzed and
the decreased erythrocyte’s sedimentation
rate (ESR) was discovered. What disease
from the listed below is accompanied with
decreased ESR?

 

910406

6

 

 

 

A.       Myocardial infarction

B. Polycytemia

C. Hepatitis

D. Vitamin B deficiency

E. Splenomegaly

152. A 38-year-old woman was admitted to the admission-diagnostic department with uterine bleeding. What are the most likely changes of blood?

A. Leukopenia

B. Polycythemia

C. Increase of haematocrite rate

D. Reduction of haematocrite rate

E. Leucocytosis

53. Desulfiram is widely used in medical
practice to prevent alcocholism. It inhi­
bits aldehyde dehydrogenase. Increased
level of what metabolite causes aversion to
alcochol?

A. Ethanol

B. Propionic aldehyde
C Methanol

D. Malonyl aldehyde

E. Acetaldehyde

54.  A patient with rheumatoid arthritis who
had been treated with indometacin has got
signs of gastropathy. What activity of the
drug can this complication be connected wi­
th?

A. Antiserotonin

B. Antihistamine

C. Antikinine

D. Anticyclooxygenase

E. Locally irritating

55. A 16-year-old boy was performed an
appendectomy. He has been hospitalized
for right lower quadrant abdominal pain
within 18 hours. The surgical specimen is
edematous and erythematous. Infiltration
by what of the following cells is the most
typical for the process occuring here?

A. Neutrophils

B. Basophils

C. Monocytes

D. Limphocytes

E. Eosinophils

56. A patient visited a dentist with complai­
nts of redness and edema of his mouth
mucous membrane in a month after dental
prosthesis. The patient was diagnosed wi­
th allergic stomatitis. What type of allergic
reaction by Gell and Cumbs underlies this
disease?

A.        Anaphylactic

B. Immunocomplex

C. Stimulating

D.        Delayed type hypersensitivity

E. Cytotoxic

57. A 40-year-old woman was admitted to
the infectious diseases department with hi­
gh body temperature. Objectively: marked
meningeal symptoms. A spinal cord puncti-
on was made. What anatomic formation was
puncturated?

A. Spatium epidurale

B. Spatium subarachnoideum

C. Cavum trigeminale

D. Cisterna cerebellomedullaris posterior

E. Spatium subdurale

58. Decreased blood supply to the organs
causes hypoxia that activates fibroblasts
function. Volume of what elements is
increased in this case?

A. Nerve elements

B. Vessels of microcircular stream

C. Parenchymatous elements of an organ

D. Intercellular substance

E. Lymphatic vessels

59. A doctor administered Allopurinol to a
26-year-old young man with the symptoms
of gout. What pharmacological action of
Allopurinol ensures therapeutical effect?

A. By general analgetic effect

B. By increasing uric acid excretion

C. By general anti-inflammatory effect

D. By inhibiting uric acid synthesis

E. By inhibiting leucocyte migration into the
joint

60. A businessman came to India from
South America. On examination the physi­
cian found that the patient was suffering
from sleeping-sickness. What was the way of
invasion?

A. As a result of bug’s bites.

B. As a result of mosquito’s bites
C After contact with sick dogs

D. Through dirty hands

E. With contaminated fruits and vegetables

61. A mother of a newborn complains of her
baby’s constant belching with undigested
milk. Which developmental anomaly is it
an evidence of?

A. Faux in pin um

B. Labium leporium

C. Esophageal fistula

D. Anal atresia

E. Esophageal atresia

62. Part of alveoles of a preterm infant didn’t
spread because of enhanced elastic recoil of

 

910406

7

 

 

 

lungs. How can this recoil be reduced?

A.       By surfactant introduction

B. By fluid suction from the respiratory tracts

C. By artificial pulmonary ventilation

D. By glycose introduction

E. By pure oxygene inhalation

63.  A patient, who suffers from congeni­
tal erythropoietic porphyria, has skin
photosensitivity. The accumulation of what
compound in the skin cells can cause it?

A.        Heme

B. Uroporphyrinogen 1

C. Protoporphyrin

D.        Uroporphyrinogen 2

E. Coproporphyrinogen 3

64.   A 38-year-old patient died during
intractable attack or bronchial asthma.
Histologic examination revealed mucus
accumulation in bronchial lumen, a lot of
fat cells (labrocytes) in the wall of bronches,
many of them are in the state of degranulati-
on, there are also a lot of eosinophils. What
pathogenesis of bronchial changes is it?

A. Immunocomplex mechanism

B.  Granulomatosis

C.  Atopy

D. Cytotoxic, cytolytic action of antibodies

E. Cellular cytolysis

65.       A woman with 0 (I) bllod group has born
a child with AB blood group. This woman’s
husband has A blood group. What genetic
interaction explains this phenomenon?

A. Incomplete dominance

B. Codominance
C Polymery

D. Recessive epistasis

E. Complementation

66. A child has inhaled a button. Where is it
likely to be?

A. In the left main bronchus

B. In the right main bronchus

C. In the larynx

D. In the trachea

E. In the esophagus

67. A patient had been taking glucocorticoi­
ds for a long ti me. When the preparation
was withdrawn he developed the symptoms
of disease aggravation, decreased blood
pressure and weakness. What is the reason
of this condition?

A. Cumulation

B. Hyperproduction of ACTH

C. Habituation

D. Sensibilization

E. Appearance of adrenal insufficiency

68. A young man felt acute pain in the back
during active drawing up on the horizontal
bar. Objectively: pain while moving upper
extremity, reduced pronation and adducti­
on functions. Sprain of what muscle can be
observed here?

A. lA.trapezius

B. M.romboideus major

C. M.levator scapulae

D. M.latissimus dorsi

E.  M.subscapularis

69.  A 45-year-old man applied to the
trauma station because of domestic shoulder
trauma. Objectively: extension, reduction
and pronation functions of the shoulder are
absent. What muscle was injured?

A. Supraspinous muscle

B. Infraspinous muscle

C. Subscapular muscle

D. Teres major muscle

E. Teres minor muscle

70.  A 10-year-old girl often experiences
acute respiratory infections with multi­
ple punctate haemorrages in the places of
clothes friction. Hypovitaminosis of what vi­
tamin has the girl?

A.C B.B2

CBi D.A

E.B6

71.       While enrolling a child to school Mantu’s
test was made to define whether revaccinati-
on was needed. The test result is negative.
What does this test result mean?

A.  Absence of antibodies for tubercle baci­
llus

B.  Absence of antitoxic immunity to the
tuberculosis

C   Presence   of   cell   immunity   to   the tuberculosis

D. Presence of antibodies for tubercle baci­
llus

E.    Absence   of   cell   immunity   to   the
tuberculosis

72.   Thyrotoxicosis leads to increased
production of thyroidal hormones I3 and
T.\, weight loss, tachycardia, psychic exci­
tement and so on. How do thyroidal
hormones effect energy metabolism in the
mitochondrion of cells?

A. Activate substrate phosphorylation

B. Activate oxidative phosphorylation

C. Stop respiratory chain

D.    Disconnect   oxidation   and   oxidative
phosphorylation

E. Slop substrate phosphorylation

 

910406

8

 

 

 

73.      A 58-year-old patient with acute cardiac
insufficiency has decreased volume of daily
urine – oliguria. What is the mechanism of
this phenomenon?

A. Decreased glomerular filtration

B.    Decreased   number   of   functioning
glomerules

C. Reduced permeamility of renal filter

D.  Rise of hydrostatic blood pressure in
capillars

E. Drop of oncotic blood pressure

74.  Blood sampling for the haematology
is recommended to carry out on an empty
stomach and in the morning. What changes
in blood formula are possible if blood
sampling was carried out after food intake?

A. Increase of plasm proteins

B. Increase of leukocyte number
C Increase of erythrocyte number

D. Decrease of thrombocyte number

E. Decrease of erythrocyte number

75. An intraoperational biopsy of mammal
gland has revealed the signs of atypical ti­
ssue presented by disorder of parenchyma
stroma proportion with domination of the
last, gland structures of different size and
shape, lined with single-layered proliferati­
ve epithelium. What is the most probable
diagnosis?

A. Noninfiltrative cancer

B.  Mastitis

C.  Infiltrative cancer

D. Fibroadenoma

E. Papilloma

76. A 25-year-old woman with red and itchy
eczematoid dermatitis visits your office. She
had a dental procedure one day earlier with
administration of a local anesthetic. There
were no other findings, although she indi­
cated that she had a history of allergic reacti­
ons. Which of the following drugs is most
likely involved?

A. Cocaine

B.  Bupivacaine
C Etidocaine
D. Procaine
IE. Lidocaine

77. A 7-year-old girl has signs of anemia.
Laboratory examination revealed pyruvate
kinase deficiency in erythrocytes. What
process disturbance plays the main role in
anemia development?

A. Tissue respiration

B. Oxidative phosphorylation

C. Anaerobic glycolysis

D. Aminoacids desamination

E. Peroxide decomposition

78.   A woman who was infected with
toxoplasmosis during the pregnancy has
born a child with multiple congenital
defects.This is a result of:

A. Cancerogenesis

B. Recombination

C. Biological mutogenesis

D. Chemical mutogenesis

E. Teratogenesis

79. On autopsy of the 58-year-old man it
was revealed: mitral valve is deformed, thi­
ckened, not totally closed. Microscopically:
foci of collagen fibers are eosinophilic, have
positive fibrin reaction. The most likely it is:

A. Fibrinoid swelling

B. Fibrinoid inflammation

C. Hyalinosis

D. Amyloidosis

E. Mucoid swelling

80. An 18-year-old patient came to the out­
patient department with the complaints of
bleeding trauma in the vestibule of his nose.
On examination: the mechanical injury of
the mucous layer of the vestibule without
continuation into nasal cavity proper. What
is the boundary between the vestibule and
nasal cavity proper?

A. Nasal septa

B. Choanes

C Nasal limen

D. Nostrils

E. Nasal roller

81.  After a serious viral infection a 3-
year-old child has repeated vomiting, loss
of consciousness, convulsions. Examinati­
on revealed hyperammoniemia. What may
have caused changes of biochemical blood
indices of this child?

A.  Disorder of biogenic amines neutralizati­
on

B.  Disorder of ammonia neutralization in
ornithinic cycle

G   Inhibited   activity   of   transamination enzymes

D.   Increased purtefaction of proteins in
intestines

E.   Activated   processes   of   aminoacids
decarboxylation

82.  In the blood of a 26-year-old man
18% of erythrocytes of the spherical, ball-
shaped, flat and spinous shape have been
revealed. Other eritrocytes were in form
of the concavo-concave disks. How is this
phenomenon called?

 

910406

9

 

 

 

A.       Pathological poikilocytosis

B. Pathological anisocytosis

C. Erytrocytosis

D. Physiological anisocytosis

E. Physiological poikilocytosis

83.  RNA-polymerase B(II) is blocked due to
amanitine poisoning (poison of death-cup).
It disturbs:

A. Synthesis of t-RNA

B.  Reverse transcription

C.  Primers synthesis

D. Synthesis of m-RNA

E. Maturation of m-RNA

84.  A 52-year-old patient has the followi­
ng diagnosis: systemic amebiasis with
involvment of intestines, liver, lungs. What
drug should be prescribed?

A. Quiniofone

B. Quingamine

C. Enteroseptol

D. Tetracycline

E. Metronidasol

85.  Necrosis focus appeared in the area of
hyperemia and skin edema in few hours
after burn. What mechanism strengthens
destructive effect in the inflammation area?

A. Primary alteration

B. Secondary alteration

C. Proliferation of fibroblasts

D. Emigration of lymphocytes

E. Diapedesis of erythrocytes

86. The energy inputs of a healthy man have
been measured. In what state was the pati­
ent if his energy inputs were less than the
main exchange?

A.        Calmness

B. Rest

C. Sleep

D. Nervous exertion

E. Easy work

87. The process of heart transplantation
determined the viability of myocardial cells.
The determination of what myocardium
parameter is the most important?

A. Concentration of oxygen in heart vessels

B. Concentration of Ca-ions in heart vessels

C. Heart temperature

D. Concentration of calcium-ions in myofi­
brils

E. Rest potential of cardiomyocytes

88. Hypertrychosis of auricles is caused by
a gene that is localized in Y-chromosome.
Father has this feature. What is the probabi­
lity to give birth to a boy with such anomaly?

A. 35%

B. 100%

C. 0%

D. 25%

E. 75%

89.       Autopsy of a 46-year-old man revealed
multiple brown-and-green layers and
hemmorhages on the mucous membrane
of rectum and sigmoid colon; slime and
some blood in colon lumen; histologically
– fibrinous colitis. In course of bacteriologi­
cal analysis of colon contents S.Sonne were
found. What is the most probable diagnosis?

A. Salmonellosis

B. Dysentery
C Cholera

D. Crohn’s disease

E. Yersiniosis

90.       A 50-year-old patient with typhoid fever
was treated with Levomycetin, next day his
condition became worse, temperature rose
to 39,6°C. What caused the complication?

A.   Irresponsiveness of an  agent  to  the
levomycetin

B. The effect of endotoxin agent
C Allergic reaction

D. Secondary infection addition

E. Reinfection

91.  A patient with suspected diphtheria
went through bacterioscopic examination.
Examination of throat swab revealed rod-
shaped bacteria with volutin granules. What
etiotropic preparation should be chosen in
this case?

A.        Diphtheria antitoxin

B. Bacteriophage

C. Interferon

D. Antidiphtheric antitoxic serum

E. Eubiotic

92.  A 37-year-old man was admitted
to the surgical department with the
symptoms of acute pancreatitis: vomiti­
ng, diarrhea, bradycardia, hypotention,
weakness, dehydration of the organism.
What medicine should be used first of all?

A. Ephedrine

B. ContrycaL

C. No-spa

D. Platyphylline

E. Etaperazine

93.    A person has reduced diuresis,
hypernatremia, hypokalemia. Hypersecreti­
on of what hormone can cause such
changes?

 

910406

10

 

 

 

A. Vasopressin

B. Parathormone

C. Aldosterone

D. Adrenalin

E. Auricular sodiumuretic factor

94.   A 60-year-old man  felt asleep after
cerebral   hemorrhage   for   a   long   time.

Damage of what structure caused this state?

A. Nuclears of the cerebral nerves

B. Hippocampus

C. Black substance

D. Cortex of the large hemispheres

E. Reticular formation

95.  During examination of a patient, there
was found a neoplasm in the white substance
of cerebral hemispheres with localization in
the knee and frontal part of posterior crus of
internal capsule. Fibres of what conductive
tract of the brain will be disrupted?

A. TV. frontothalamicus

B. Tr. parietooccipitopontinus

C. Tr. frontopontinus

D. Tr. thalamocorticalis

E. Tr. pyramidalis

96.  Intake of oral contraceptives containi­
ng sex hormones inhibits secretion of the
hypophysiae hormones. Secretion of whi­
ch of the indicated hormones is inhibited
while taking oral contraceptives with sex
hormones?

A. Follicle-stimulating

B. Oxytocin
C Thyrotropic

D. Vasopressin

E. Somatotropic

97. A patient died from acute cardiac insuffi­
ciency. The histological examination of
his heart revealed the necrotized section
in myocardium of the left ventricle, whi­
ch was separated from undamaged tissue
by the zone of hyperimic vessels, small i
hemorrhages and leukocytic infiltration.
What is the most likely diagnosis?’

A. Focal exudate myocarditis

B.  Myocardial infarction

C.  Productive myocarditis

D. Myocardial ischemic dystrophy

E. Diffuse exudate myocarditis

98. A patient with tissue trauma was taken
a blood sample for the determination of
blood clotting parameters. Specify the right
sequence of extrinsic pathway activation:

A.       Ill-Vila-Xa

B. Ill-VIII: TF-Xa

C. IV – Vila – Xa

D. IV -VIII:TF-Xa

E. Ill – IV – Xa

99.  Intrapleural pressure of an individual is
being measured. In what phase did he hold
his breath if the pressure is – 25 cm H-iOl

A. Forced expiration B.-

C. Forced inspiration

D. Quiet inspiration

E. Quiet expiration

100.     Donor skin transplantation was
performed to a patient with extensive burns.
On the 8-th day the graft became swollen
and changed colour; on the 11-th day graft
rejection started. What cells take part in this
process?

A.       T-lymphocytes

B. B-lymphocytes

C. Erythrocytes

D. Basophils

E. Eosinophils

101.  A 38-year-old man who poisoned hi­
mself with mercury dichloride was taken
to the admission room in grave conditi­
on. What antidote should be immediately
introduced?

A.       Unithiol

B. Isonitrosine

C. Dipiroxim

D.       Atropine

E. Nalorphine

102.  A man who was bitten by the unknown
dog applied to the surgeon. Wide ragged
woundes were localized on the face. What
curative-prophylactic aid should be given to
prevent rabies?

A.    Immediately   inject   normal   gamma
globulin

B. Start immunisation with rabies vaccine

C. Immediate injection of DPT(Diphtheria,
Pertusis, Tetanus) vaccine

D. Prescribe combined antibiotic therapy

E. Hospitalize the patient and keep under
the doctor’s supervision

103.  Scraps of the mycelium of a fungus,
spores, air bubbles and fat drops were di­
scovered on microscopy of the patient’s
hair excluded from the infected areas. What
fungus disease is characterised by this mi­
croscopic picture?

 

910406

11

 

 

 

A.        Epidermophytosis

B. Trichophytosis

C. Sporotrichosis

D. Microspory

E. Favus

104.  Live vaccine was injected into the
human body. Increasing activity of what
connective tissue cells can be expected?

A. Macrophages and fibroblasts

B. Plasmocytes and lymphocytes

C Adipocytes and adventitious cells

D. Pigmentocytes and pericytes

E. Fibroblasts and labrocytes

105.    Buffer capacity of a worker’s blood was
decreased due to exhausting muscular work.
By coming of what acid substance in the
blood can this state be explained?

A. 1,3-bisphosphoglycerate

B. a-ketoglutarate
C Pyruvate

D. Lactate

E. 3-phosphoglycerate

106.   During the fetal period of the
development in the vascular system of
the fetus a large arterial (Botallo’s)
duct is functioning which converts into
lig.arteriosum after birth. What anatomi­
cal formations does this duct connect?

A. Pulmonary trunk and superior vena cava

B. Pulmonary trunk and aorta
C Aorta and superior vena cava

D. Right and left auricles

E. Aorta and inferior vena cava

107. On autopsy it was revealed: pia mater
of the upper parts of cerebral hemisphere
is plethoric, of yellowish-green color, soaked
with purulent and fibrose exudate, looks like
a cap. What disease is it typical for?

A. Meningitis connected with typhus

B. Meningitis connected with anthrax
C Grippal meningitis

D. Tuberculous meningitis

E. Meningococcal meningitis

108.  A patient with thrombophlebitis of
lower extremities had got chest pains, blood
spitting, growing respiratory failure that
caused his death. Autopsy revealed multi­
ple pulmonary infarctions. What is the most
probable reason of their development?

A.        Pulmonary venous thrombosis

B. Bronchial artery embolism
C Pulmonary artery embolism

D.        Bronchial artery thrombosis

E. Pulmonary artery thrombosis

109.  Patient with abscess of the cut wound
applied to the traumatological department.

In order to clean the wound from the pus doctor washed it with 3% hydrogen peroxi­de. Foam was absent. What caused the absence of the drug activity?

A. Inherited insufficiency of catalase

B.    Inherited insufficiency of erythrocyte
phosphatdehydrogenase

C. Shallow wound

D. Pus in the wound

E. Low concentration H-iPi

110. Autopsy of a man who died from
chronic cardiacvascular collapse revealed
“tiger heart”. Sidewards of endocardium
a yellowish-white banding can be seen;
myocardium is dull, dark-yellow. What
process caused this pathology?

A. Fatty parenchymatous degeneration

B. Carbohydrate degeneration

C Fatty vascular-stromal degeneration

D. Amyloidosis

E. Hyaline degeneration

111. Cerebral trauma caused increase of
ammonia formation. What aminoacid takes
part in removal of ammonia from cerebral
tissue?

A. Tryptophan

B. Tyrosine

C. Valine

D. Lisine

E. Glutamic

112. A 5-year-old child who often fells ill
with respiratory diseases has eczematous
appearances after consumption of some
food products, tendency to prolonged
course of inflammatory processes. What ki­
nd of diathesis can be suspected in this case?

A. Lymphohypoplastic

B. Asthenic

C. Exudative-catharral

D. Hemmorhagic

E. Arthritism

113. ATP synthesis is totaly blocked in a
cell. How will the value of membrane rest
potential change?

A. First it will decrease, then increase

B. It will disappear

C. It will be considerably increased
I). First it will increase, then decrease
E. It will be slightly increased

114.  A 44-year-old woman complains of
common weakness, heart pain, considerable
increase of body weigt. Objectively: moon­
like face, hirsutism, AP- 16S/100 mm He,
height -164 cm. weight -103 kg; fat is mostly
accumulated in the region of neck, upper
shoulder girdle, stomach. What is the main
pathogenetic mechanism of obesity?

 

910406

 

A. Decreased production of glucagon

It.   Decreased   production   of   thyroidal

hormones

C.  Increased production of mineralocorticol-
cls

D. Increased production of insulin

E. Increased production of glucocorticoids

115.   Glomerular filtration rate (GFR)
increased by 20% as a result of prolonged
starvation of an individual. The most evi­
dent cause of filtration changes under this
conditions is:

A.  Decrease of oncotic pressure of blood
plasma

B. Increase of systemic blood pressure >

C. Increase of penetration of the renal Alter
1). Increase of nitration coefficient

E. Increase of renal plasma stream

116.   Inhibition of o-motoneuron of the
extensor muscles was noticed after sti­
mulation of n-motoneuron of the flexor
muscles during the experiment on the spinal
cord. What typo of inhibition is this process
based upon?

A. Recurrent

B. Reciprocal

C. Lateral

D. Depolarizational

E. Presynaptic

117.    A 45-year-old woman suffers from
seasonal allergic rhinitis caused by the
ambrosia blossoming. What medicine from
the stabilizer of the adipose cells group can
be used for prevention of this disease?

A. Tavegyl

B. Dimearol

C. Ketotifen

D. Phencarol

E. Diazoline

118.   A 57-year-old patient was admitted
to the gastroenterological department wi­
th suspicion of /.ollinger-Kllison syndrom
because of rapid increase of gastrin level
in the blood serum. What is the most
probable disorder of the secretory functi­
on of stomach?

A. Hypcracidic hypersecretion

B. Achylla

C I lypei acidic hyposecrelion

D. Hypoacidic hyposccretion

E.  Hypoacidic hypersecretion

119. Inflamatlon is characterised by Increased permeability of vessels of mi­crocirculation stream, increase of their hydrodynamic blood pressure. Increasing or the osmotic concentration and dlspersi-

ty of protein structures is present In the intercellular fluid. What kind of edema will appear in this case?

A. Mcmbi allogenic

B. Mixed

C. Hydrodynnmic

D. Lymphogenic

E. Colloid-osmotic

120. Tuberculosis can be treated by means
of combined chemotherapy that includes
substances with different mechanisms of
action.What nntituberculous medication
inhibits transcription of RNA into DNA in
mycobacteria?

A. Ethionamide

B. Isoniazid

C. Streptomycin

D. Para-aminosalicylic acid

E. Rifampicin

121. A patient who was previously ill with
mastectomy as a result of breast cancer was
prescribed radiation therapy. What vitamin
preparation has marked radioprotective
action caused by antioxidant activity?

A. Folic acid

B. Riboflavin

C. Thiamine chloride

D. Tocopherol acetate

E. Ergocalciferoi

122. A person has steady MR not exceedi­
ng 40 hpni. What is the pacemaker of this
person’s heart rhythm?

A. Sinoatrial node

B. Purkinje’s fibers

C. Atrioventricular node

D. His’ bundle

E. Branches of His’ bundle

123. A patient has undergone an amputation
of lower extremity. Some time later painful
nodules appeared in a stump. Amputations
neuromas were found out at the microscopic
examination. To what pathological processes
do those formations relate?

A. Regeneration

B. Hyperemia
G. Inflammation
I). Metaplasia
E. Dystrophy

124. Upper neck node of sympathetic trunk
was removed from the rabbit on experiment.
Reddening and increased temperature of
the skin of head is observed. What disorder
of peripheral circulation of the blood has
developed?

 

910406

13

 

 

 

A.        Neurotonic arterial hyperemia

B. Metabolic arterial hyperemia
C Venous hyperemia

D. Neuroparalytic arterial hyperemia

E. Stasis

125. A 30-year-old woman was diagnosed with insufficiency of exocrinous function of

ancreas. Hydrolisis of what nutrients will

e disturbed?

A. Proteins

B. Proteins, fats

C. Proteins, fats, carbohydrates

D. Fats, carbohydrates

E. Proteins, carbohydrates

126. A patient has elbow joint trauma with
avulsion of medial epicondyle of humerus.
What nerve can be damaged in this trauma?

A.        Ulnar

B. Medial cutaneous nerve of forearm
C Radial

D. Musculocutaneous nerve

E. Cardiac cutaneous nerve

127.  A patient operated on complicated
appendicitis has the following changes of
blood count: erythrocytes – 4,0 • 1012/f,
Hb – 120 g/1, color index – 0,9, leukocytes
– 18 • 109//, basophils – 0, eosinophils – 0,
myelocytes – 0, juvenile – 0, stab neutrophi­
ls – 20, segmentonuclear neutrophils – 53,
lymphocytes – 21, monocytes – 5. How is such
nuclear shift of leukocytic formula called?

A. Degenerative left shift

B. Regeneratively-degenerative

C. Hyperregenerative

D. Regenerative left shift

E. Right shift                                               |

128.  A 60-year-old patient has reduced
perception of high-frequency sounds. What
structures’ disorder of auditory analizer
caused these changes?

A.  Main membrane of cochlea near heli-
cotrema

B. Eustachian tube

C Tympanic membrane

D. Muscles of middle ear

E. Main membrane of cochlea near the oval
window

129.  A patient with clinical signs of
immunodeficiency has no changes of
the number and functional activity of
T- and B- lymphocytes. Defect with di­
sfunction of antigen-presentation to the
immunocompetent cells was found during
examinatio on the molecule level. Defect of
what cells is the most probable?

A. Macrophages, monocytes

B. 0-lympnocytes

C T-lymphocytes, B-lymphocytes

B-

D.       Fibroblasts,       T-lymphocytes,
lymphocytes

E. NK-cells

130. A 50-year-old patient was injured on
the occipital region of the head. The closed
skull trauma was diagnosed. She was taken
to the hospital. The medical examinati­
on: deregulation of walking and balance,
trembling of arms. What part of brain was
injured?

A.        The mind-brain

B. The spinal cord

C. The cerebellum

D.        The medulla oblongata

E. The inter-brain

131. A patient with diabetes mellitus has
been delivered in hospital in the state of
unconsciousness. Arterial pressure is low.
The patient has acidosis. Point substances,
which accumulation in the blood results in
these manifestations:

A. Ketone bodies

B. High fatty acids

C. Cholesterol esters

D. Monosaccharides

E. Amino acids

132. Autopsy of a 12-year-old girl revealed:
multiple cutaneous hemmorhages (mostly
into the skin of buttocks, lower extremi­
ties), serous and mucous memrane
hemmorhages, cerebral hemmorhages.
Adrenal glands show focal necrosis and
massive hemmorhages; kidneys show
necrotic nephrosis, suppurative arthritis,
iridocyclitis, vasculitis. What is the most
probable diagnosis?

A. Meningococcemia

B. Epidemic typhus

C. Systemic lupus erythematosus

D. Periarteritis nodosa

E. Radiation sickness

133.  A 2-year-old child has got intesti­
nal dysbacteriosis, which resultsed in
hemorrhagic syndrome. What is the most
likely cause of hemorrhage of the child?

A. Activation of tissue thromboplastin

B. PP hypovitaminosis

C. Fibrinogen deficiency

D. Vitamin K deficiency

E. Hypocalcemia

134. An isolated muscle of a frog is rhythmi­
cally irritated with electric impulses. Every
next impulse is in a period of relaxation from
the previus contraction. What contraction of

 

910406

14

 

 

 

the muscle occurs?

A.        Asynchronous

B. Single

C Waved tetanus

D. Tonic

E. Continuous (smooth) tetanus

135.  It is planned to use the territory of an
old cattle burial ground (which is not used
for more than 50 years) for building houses.
But ground analysis revealed presence of
the pathogen of a very dangerous illness.
Which of the indicated microorgonisms is li­
kely to remain in the ground for such a long
time?

A. Francisella tularensis

B. Mycobacterium bovis

C. Brucella abortus

D. Yersinia pestis

E. Bacillus anthracis

136.   An experiment proved that UV-
radiated cells of patients with xeroderma
pigmentosum restore the native DNA
structure slower than cells of healthy indi­
viduals as a result of reparation enzyme
defection. What enzyme helps this process?

| A. Primase

B. DNA polymerase III

C. Endonuclease

D. DNA gyirase

E. RNA ligase

137. Albinos can’t stand sun impact – they
don’t aquire sun-tan but get sunburns. Di­
sturbed metabolism of what aminoacid
underlies this phenomenon?

A.        Histidine

B. Methionine

C. Tryptophan

D. Glutamic acid

E. Phenilalanine

138. Examination of a newborn boy’s geni­
tals revealed a cleft of urethra that opens
on the inferior surface of his penis. What
developmental anomaly is meant?

A. Cryptorchism

B. Hermaphroditism

C. Epispadia

D. Monorchism

E. Hypospadia

139. A 54-year-old man was admitted to the
hospital with complaints of pain in the ri­
ght subcostal region, vomiting with blood.
Objectively: enlarged liver, varicose veins in
the stomach and esophagus. Disfunction of
what vessel is likely to have taken place?

A. Vena porta

B. Vena hepatica

C. Vena cava superior

D. Vena cava inferior

E. Aorta abdominalis

140. A patient who has been strictly keeping
to a certain diet for 10 days went through
examination of respiratory coefficient. It
was determined that it was equal 1. What
have the patient been keeping to?

A. With domination of proteins and fat

B.    With   domination   of   proteins   and
carbohydrates

C. Mixed

D. With domination of fat and carbohydrates

E. With domination of carbohydrates

141.  In course of prophylactic medical
examination a 7-year-old boy was diagnosed
to have daltonism. Parents are healthy, color
vision is normal. But grandfather from the
mother’s side has the same disorder. What is
the type of inheriting of this anomaly?

A. Autosomal-dominant

B. Autosomal-recessive
C Dominant, sex-linked

D. Incomplete domination

E. Recessive, sex-linked

142.  A 25-year-old patient complained of
the decreased vision. Accommodation di­
sorders, dilated pupil, lack of reaction for
the light were revealed on examination.
What muscles function is disturbed?

A. Inferior oblique muscle, ciliary

B. Lateral rectus muscle, pupil narrowing

C. Pupil narrowing and dilating muscles

D. Pupil narrowing muscle, ciliary

E. Pupil dilating muscle, ciliary

143.  A 63-year-old woman shows symptoms
of rheumatoid arthritis. The increase of what
blood indices could be the most significant
for proving the diagnosis?

A. Additive glycosaminoglycans

B. Lipoproteids

C.  Acid phosphatase

D. R-glycosidase

E. General cholesterol

144.  A 60-year-old patient was admitted to
the surgical department because of infection
caused by blue pus bacillus (Pseudomonas
aeruginosa) which is sensative to penici­
llin antibiotics. Indicate which of the gi­
ven penicillins has marked activity to the
Pseudomonas aeruginosa?

 

910406

15

 

 

 

 

 

A.       Carbenicillin disodium

B. Phenoxymethylpenicillin
C Benzylpenicillin

D. Methicillin

E.  Oxacillin

145.  A duodenal content smear of a pati­
ent with indigestion contains protosoa 10-18
mem large. They have piriform bodies, 4 pai­
rs of filaments, two symmetrically located
nuclei in the broadened part of body. What
kind of the lowest organisms is it?

A. Lamblia

B. Balantidium

G. Intestinal ameba

D. Dysentery ameba

E. Trichomonas

146.  A 22-year-old patient was admitted
to the hospital with complaints of heavy
nasal breathing. During the examination
of her nasal cavity the doctors found thi­
ckened mucous membrane, a lot of mucus
and nodular infiltrates without erosions in
the nose.The nasal rhinoscleroma was di­
agnosed. The biopsy was taken. What typi­
cal morphological changes may be found?

A. Granulomas with Virchow’s cells

B. Granulomas with Langhan’s cells
C Granulomas with foreign body cells

D. Granulomas with Mikulicz’s cells

E. Interstitial inflammation

147.    A patient with encephalopathy
was admitted to the neurological in­
patient department. There was revealed
a correlation between increasing of
encephalopathy and substances absorbed by
the bloodstream from the intestines. What
substances that are formed in the intestines
can cause endotoxemia?

A. Acetacetate

B. Butyrate

C. Ornithine

D. Biotin

E. Indole

148. Nowadays about 50 minor bases have
been found in the t-RNA structure besides
the main four nitrogenous bases. Choose the
minor nitrogenous base:

A.        Dihydrouracil

B. Adenine

C. Cysteine

D. Uracil

E. Cytosine

149. A patient with suspicion on epidemic
typhus was admitted to the hospital. Some
arachnids and insects have been found in his
flat. Which of them may be a carrier of the
pathogen of epidemic typhus?

A. Bed-bugs

B. Spiders

C. Cockroaches

D. Lice

E. Houseflies

150.    Secretion of what gastrointestinal
hormones will be primarily decreased as a
result of iduodenum removal?

A. Gastrin

B. Gastrin and histamine

C. Histamine

D. Cholecystokinin and secretin

E. Neurotensin

151.     Microspecimen of spinal cord contai-‘
ns a nucleus that should be analyzed. Its
neurons form motor endings in the skeletal
muscles. What nucleus of spinal cord is
meant?

A. Proper nucleus of the anterior horn

B. Proper nucleus of the posterior horn
C Intermediate lateral nucleus

D. Thoracic nucleus

E. Proper nucleus of gray substance

152.     The donor who didn’t donate the
blood for a long time was investigated wi­
th IFA method. Anti-HBs antibodies were
revealed. What does positive result of IFA
in this case mean?

A. Chronic hepatitis C

B. Previous hepatitis B

C. Acute hepatitis B

D. Acute hepatitis C

E. Chronic hepatitis B

153.     A patient has tissue ischemia below the
knee joint accompanied with intermittent
claudication. What artery occlusion should
be suspected?

A. Popliteal artery

B. Posterior tibial artery

C. Peroneal artery

D. Anterior tibial artery
E.JProximal part of femoral artery

154.     The preventive radioprotector was gi­
ven to a worker of a nuclear power station.
What mechanism from the below mentioned
is considered to be the main mechanism of
radioprotection?

A. Activation of oxidation reactions

B. Increasing of tissue blood supply

C. Prevention of tissue’s hypoxia

D. Increasing of respiration

E. Inhibition of free radicals formation

155. A patient who came to the doctor
because of his infertility was administered
to make tests for toxoplasmosis and chronic

 

910406

16

 

 

 

gonorrhoea. Which reaction should be performed to reveal latent toxoplasmosis and chronic gonorrhoea of the patient?

A. IFA – Immunofluorescence assay

B. (R)CFT- Reiter’s complement fixation
test

C Immunoblot analysis

D. RIHA – Reverse indirect hemagglutinati­
on assay

E. RDHA – Reverse direct hemagglutination
assay

156.    While shifting the gaze to the closely si­
tuated object the refracting power of eye’s
optical mediums will increase by 10 di­
opters. It results from changing of such eye
structure:

A. Cornea

B. Muscle that dilatates pupil
CLens

D. Vitreous body

E. Liquid of the anterior chamber of eye

157.      A large-scale reaction with
parapertussis and pertussis diagnosticums
was made in order to make serological di­
agnostics of the whooping cough. At the
bottom of the test-tubes with diagnosti­
cum of Bordetella parapertussis a granular
sediment formed. What antibodies did this
reaction reveal?

A. Opsonins

B. Antitoxins
C Precipitins

D. Agglutinins

E. Bacteriolysins

158.  While having the dinner the child
choked and aspirated the food. Meavy
cough has started, skin and mucose are
cyanotic, pulse is rapid, respiration is
infrequent, expiration is prolonged. What
disorder of the external respiration has the
child?

A. Alternating respiration

B. Biot’s respiration

C Stage of expiratory dyspnea on asphyxia

D. Stenotic respiration

E. Stage of inspiratory dyspnea on asphyxia

159.  An isolated cell of human heart
automatically generates excitation impulses
with frequency 60 times pro minute. What
structure does this cell belong to?

A. Sinoatrial node

B. Atrium

C. Atrioventricular node

D. Ventricle

jE. His’ bundle

160.   A  patient  working at  a  chemical

plant was admitted to the toxicological department with mercury poisoning, what medicine should be used?

A. Unithiol

B. Naloxone
G. Isonitrozin

D. Activated carbon

E. Enterosorbent

161.     Healthy parents have got a fair-haired,
blue-eyed girl. Irritability, anxiety, sleep and
feeding disturbance developed in the fi­
rst months of the infant’s life. Neurologi­
cal examination revealed developmental
lag. What method of genetic investigation
should be used for the exact diagnosis?

A. Population-statistical

B.  Cytological

C. Genealogical

D. Gemellary

E. Biochemical

162.  An individual is characterized by
rounded face, broad forehead, a mongoli­
an type of eyelid fold, flattened nasal bri­
dge, permanently open mouth, projecting
lower lip, protruding tongue, short neck, flat
hands, and stubby fingers. What diagnosis
can be put to the patient?

A. Klinefelter’s syndrome

B. Down’s syndrome
C Alkaptonuria

D.        Supermales

E. Turner’s syndrome

163.     As a result of craniocerebral trauma
a patient reveals the following symptoms:
intention tremor, dysmetry, adiadochoki-
nesis, dysarthria. What structure of the brain
is injured?

A. Black substance

B. Pale sphere

C. Cerebellum

D. Striatum

E. Motor cortex

164.     A patient suffering from trombophlebi-
tis of the deep crural veins suddenly di­
ed. Autopsy has shown freely lying red
friable masses with dim crimped surface
in the trunk and bifurcation of the
pulmonary artery. What pathologic process
was revealed by morbid anatomist?

A. Tromboembolism

B. Tissue embolism

C. Embolism with foreign body

D. Thrombosis

E. Fat embolism

165.     A 35-year-old man under the treatment
for pulmonary tuberculosis has acute pain
onset of right big toe, swelling, and low-

 

910406

17

 

 

 

grade fever. The gouty arthritis was di­agnosed and high serum uric acid level was found. Which of the following anti-tuberculous drugs are known for causing hi­gh uric acid levels?

A.        Pyrazinamide

B. Cycloserine

C. Thiacetazone

D.        Rifampicin

E. Aminosalicylic acid

166. A patient had been ill with bronchi­
al asthma for many years and died from
asthmatic fit. Histologic lung examination
revealed: lumen of bronchioles and small]
bronches contain a lot of mucus with some
eosinophils, there is sclerosis of alveolar
septums. dilatation of alveole lumen. What
mechanism of development of hypersensibi-
lity reaction took place?

A. Reagin reaction

B. Immunocomplex reaction

C. Cytotoxic reaction

D. Cytolysis determined by lymphocytes

E. Granulomatosis

167. During the endoscopy the inflammation
of a major papilla of the duodenum and thel
disturbances of bile secretion were found. In
which part of duodenum were the problems
found?

A. Lower horizontal part

B. Descendent part
C Ascendant part

D. Bulb

E. Upper horizontal part

168. Moving of the daughter chromatids to
the poles of the cell is observed in the mi-
toticaliy dividing cell. At what stage of the
mitotic cycle is this cell?                                    \

A. Prophase

B. Telophase
C Anaphase

D. Interphase

E. Metaphase

169.   A patient with serious damage of
muscular tissue was admitted to the
t Hematological department. What bi­
ochemical urine index will be increased in
this case?

A. Common lipids

B. Creatinine

C. Glucose

D. Uric acid

E. Mineral salts

170. Microscopic examination of the sputum
of a patient with pneumonia occasionally
revealed some larvae. Eosinophiles were
detected  on   blood  examination.   What

helminthiasis can be diagnosed?

A. Trichocephaliasis

B. Ascariasis

C. Opisthorchosis

D. Enterobiosis

E. Paragonimiasis

171.  As a result of the damage of one of
the Atomic Power Plant reactors the run-|
out of radioelements took place. People
in the high-radiation area were radiated
with approximately 250-300 r. They were
immediately hospitalized. What changes in
the blood count would be typical for the vi­
ctims?

A. Neutropenia

B. Lymphopenia

C. Anemia

D. Leukopenia

E. Thrombopenia

172.  A patient with complaints of dryness in
the mouth, photophobia and vision impai­
rment was admitted to the reception-room.
Skin is hypercmic, dry, pupils are dilated,!
tachycardia. Poisoning with belladonna
alkaloids was diagnosed on further exami­
nation. What medicine should be prescri­
bed?

A. Prozerin

B. Dipiroxim

C. Diazepam

D. Armine

E. Pilocarpine

173.  Microspecimen of red bone marrow
contains multiple capillares through the
walls of which mature blood cells penetrate
into the bloodstream. What type of capi­
llares are these?

A. Somatical

B. Visceral

C. Sinusoidal

D. Lymphatic

E.  Fenestrational

174.  In the specimen of one of the parts
of respiratory system a tubular organ wasj
found. It has low epithelium, well developed
muscular tunic, glands and cartilage are
absent. Name this organ:

A. Trachea

B. Larynx

C. Major bronchs

D. Median bronchs

E. Minor bronchs

175.  Increased fragility of vessels, enamel
and   dentine   destruction   resulting   from!
scurvy are caused by disorder of collagen
maturation.   What   stage   of  procollagen
modification is disturbed under this avitami-

 

910406

18

 

 

 

nosis?

A. Hydroxylation of proline

B. Detaching of N-ended peptide

C   Removal   of   C-ended   peptide   from procollagen

D. Glycosylation of hydroxylysine residues

E. Formation of polypeptide chains

176.   Dystrophic alterations of heart are
accompanied with dilation of heart caviti­
es, decreased force of heart contractions,
increased blood volume that remains duri­
ng systole in the heart cavity, vein overfill.
What heart condition is it typiccal for?

A. Myogenic dilatation

B. Tonogenic dilatation

C. Emergency stage of hyperfunction and
hypertrophy

D. Cardiosclerosis

E. Cardiac tamponade

177. A patient with infectious mononucleosis
had been taking glucocorticoids for two
weeks. He was brought into remission, but
he fell ill with acute attack of chronic tonsi­
llitis. What action of glucocorticoids caused
this complication?

A. Anti-inflammatory

B.  Antishock.

C.  Antiallergic

D. Immunosuppressive

E. Antitoxic

178. From the nasopharynx of a 5-year-old
child it was excreted a microorganism which
is identical to Corynebacterium diphtherias
dose according to morphological and bi­
ochemical signs. But this microorganism
does not produce exotoxin. As a result
of what process can this microorganism
become toxigenic?

A. Growing with antitoxic serum

B. Phage conversion

C.   Passing through  the organism of the
sensative animals

D. Chromosome mutation

E. Cultivation in the telluric environment

179.  A 1-year-old child with symptoms of
muscle involvement was admitted to the
hospital. Examination revealed carnitine
deficiency in his muscles. What process di­
sturbance is the biochemical basis of this
pathology?

A. Transporting of fatty acids to mitochodri-
ons

B. Substrate phosphorylation
C Actin and myosin synthesis

D. Lactic acid utilization

E.  Regulation of 6’«n level in mitochondri-
ons

180.  A patient who has been treated
with diazepam on account of neurosis
complains of toothache. Doctor admini­
stered him an analgetic, but its dose was
lower than average therapeutic dose. What
phenomenon did the doctor take into
account while prescribing the patient an
underdose?

A. Potentiation

B. Tolerance

C. Drug dependence

D. Summation

E. Cumulation

181. A 42-year-old man suffering from gout
has increased level of urinary acid in blood.
Allopurinol was prescribed to decrease the
level of urinary acid. Competitive inhibitor
of what enzyme is allopurinol?

A. Guanine deaminase

B. Xanthine oxidase

C Adenine phosphoribosiltransferase

D. Adenosine deaminase

E. Hypoxanthine phosphoribosiltransferase

182.  Part of the DNA chain turned 180
degree as a result of gamma radiation. What
type of mutation took place in the DNA
chain?

A. Translocation

B. Deletion

C. Doubling

D. Inversion

E. Replication

183.  An autopsy revealed large (1-2 cm)
brownish-red, easy crumbling formations
covering ulcerative defects on the external
surface of the aortic valve. What is the most
likely diagnosis?

A. Fibroplastic endocarditis

B. Recurrent warty endocarditis

C. Diffusive endocarditis

D. Acute verrucous endocarditis

E. Polypous-ulcerative endocarditis

184.  Analysis of amniotic fluid that was
obtained as a result of amniocentesis
(puncture of amniotic sac) revealed cells
the nuclei of which contain sex chromatin
(Barr’s body). What can it be evidence of?

A.        Genetic disorders of fetus development

B. Polyploidy

C. Trisomy

D.        Development of female fetus

E. Development of male fetus

185. To anaesthetize the surgical treatment
of burn surface, a patient was intravenously
injected a medication for short-acting
narcosis. 1 minute later the patient being

 

910406

19

 

 

 

under anaesthesia showed increased blood pressure, tachycardia, increased tone of skeletal muscles; reflexes remained. After recovering from anaesthesia the patient had disorientation and visual hallucinations. What medication was the patient injected?

A.       Sombrevin

B. Ketamine

C. Diethyl ether

D.       Nitrous oxide

E. Thiopental sodium

186. Patients with bile ducts obstruction
suffer from inhibition of blood coagulation,
bleedings as a result of low level of vitamin
assimilation. What vitamin is in deficiency?

A.K B.A CE D.D E. Carotene

187.  A person felt thirsty after staying under
the conditions of hot weather for a long ti­
me. Signals of what receptors caused it first
of all?

A. Osmoreceptors of liver

B. Glucoreceptors of hypothalamus
C Sodium receptors of hypothalamus

D. Baroreceptors of aortic arch

E. Osmoreceptors of hypothalamus

188. To prevent long-term effects of 4-day
malaria a 42-year-old patient was prescri­
bed primaquine. On the 3-rd day from the
begin of treatment there appeared stomach
and heart pains, dyspepsia, general cyanosis,
hemoglobinuria. What caused side effects of
the preparation?

A. Cumulation of the preparation

B. Drug potentiation by other preparations

C.    Delayed   urinary   excretion   of   the
preparation

D.    Genetic   insufficiency   of   glucose   6-
phosphate dehydrogenase

E.  Decreased activity of microsomal liver
enzymes

189.    A child has got galactosemia.
Concentration of glucose in blood has not
considerably changed. Deficiency of what
enzyme caused this illness?

A.        Phosphoglucomutase

B. Amylo-l,6-glucosidase

C. Galactose-1-phosphate uridyltransferase

D.        Hexokinase

E. Galactokinase

190. When a patient with traumatic impai­
rment of the Brain was examined, it was di­
scovered that he had stopped to distinguish
displacement of an object on the skin. What

part of the brain was damaged?

A.        Posterior central gurus

B. Frontal zone

C. Occipital zone of the cortex

D. Parietal zone of the cortex

E. Frontal central gurus

191.    A tissue sample of benign tumor was
studied under the electron microscope. A lot
of small (15-20 nm) spherical bodies, consi­
sting of 2 unequal subunits were detected.
These are:

A. Microtubules

B. Smooth endoplasmic reticulum

C. Mitochondria

D. Ribosomes

E. Golgi complex

192.    Most participants of Magellan expediti­
on to America died from avitominosis. This
disease declared itself by general weakness,
subcutaneous hemmorhages, falling of
teeth, gingival hemmorhages. What is the
name of this avitiminosis?

A. Rachitis

B. Polyneuritis (beriberi)

C. Biermer’s anemia

D. Pellagra

E. Scurvy

193.  Parents with an ill child consulted
an infectionist. They had been working in
one of Asian countries for a long time.
The child has sallow skin, loss of appeti­
te, laxity, enlarged liver, spleen, peripheral
lymph nodes. What protozoal illness can be
suspected?

A. Balantidiasis

B. Amebiasis

C. Lambliasis

D. Visceral leishmaniasis

E. Toxoplasmosis

194. Kidneys of a man under examination
show increased resorbtion of calcium ions
and decreased resorbtion of phosphate ions.
What hormone causes this phenomenon?

A.        Parathormone

B. Vasopressin

C. Aldosterone

D.                                                             Hormonal form D3

E. Thyrocalcitonin

195.   Examination of initial molecular
structure revealed substitution of the
glutamic acid by valine. What inherited
pathology is it typical for?

 

A.        Hemoglobinosis

B. Minkowsky-Shauffard disease

C. Thalassemia

D. Favism

E. Sickle-cell anemia

196. After a trauma a 44-year-old patient
had a rupture of left palm muscle tendons
and of the superficial blood vessels. After
operation and removal of the most part of
the necrotically changed muscle tissue the
bloodstream was normalized. What vessels
have helped to restore the bloodstream?

A.        Arcus palmaris profundus

B. Arcus palmaris super ficialis

C. Aa. perforantes

D. Aa. metacarpeae palmares

E. Aa. digitales palmares communes

197. The effect of electric current on the
exit able cell caused depolarization of its
membrane. Movement of what ions through
the membrane caused depolarisation?

A. HCOJ

B.cr

D.jVa+ E. Ca2+

198. Testosterone and it’s analogs increase
the mass of skeletal muscles that allows to
use them for treatment of dystrophy. Due to
interaction of the hormone with what cell
substrate is this action caused?

20

A. Proteins-activators of transcription

B. Membrane receptors

C. Chromatin

D. Nuclear receptors

E. Ribosomes

199.  For a long time a 49-year-old woman
had suffered from glomerulonephritis which
caused death. The autopsy revealed that the
size of her kidneys was 7 x 3 x 2,5 cm, weight
65,0 g, they were dense and small-grained.
Microscopically: iibrinogenous inflammati­
on of serous and mucous membranes,
dystrophic changes of parenchymatous
organs, brain edema. What complication can
cause such changes of serous membranes
and inner organs?

A.        Uraemia

B. Anemia

C. Thrombopenia

D. Sepsis

E. DIC-syndrome

200.    A patient experienced a sudden
temperature rise up to 39°C. After 6 hours
the temperature normalized. On the 2-nd
day the attack recurred: in the period of
paroxysm the temperature reached 4l°C,
apyrexial period began after 8 hours. What
type of temperature profile is it?

A.        Continued

B. Intermitting

C. Recurrent

D. Hectic

E. Septic

 

INSTRUCTIONAL       BOOK

Ministry of public health of Ukraine (MPH of Ukraine)

Department of manpower policy, education and science of MPH of Ukraine

Testing Board

TEST ITEMS FOR LICENSING EXAMINATION: KROK 1. MEDICINE. 2006.

Composers:            Artcmchuk Ludmila

Bulakh Irynn Homchcnkova Natalya

03

Kyiv.Tcsting Board. (English language).

Editor. A.Potapova

Technical editor: P.Ostapcnko, V.Zhdanov, V.Sobko

Approved to print 09.10.2006. Format 84 xlOSO 1/8 Offset paper. Typeface. Times New Roman Cyr. Offset print. Conditional print pagcs.-3,0;i„ Accounting publishing pages..- 6 ji., Issue. 750 copies

 

List of abbreviations

 

 

A/G Albumin/globulin ratio HR Heart rate
A-ANON Alcoholics anonymous IDDM Insulin dependent diabetes mellitus
ACT Abdominal computed tomography

I FA       1

Immunofluorescence assay                1
ADP Adenosine diphosphate D3D Ischemic heart disease
ALT Alanin aminotranspherase IU International unit
AMP Adenosine monophosphate LDH Lactate dehydrogenase
AP Action potential MSEC Medical and sanitary expert committee
ARF Acute renal failure NAD Nicotine amide adenine dinucleotide A
AST Aspartat aminotranspherase NADPH Nicotine amide adenine dinucleotide
ATP Adenosine triphosphate   phosphate restored
BP Blood pressure NIDDM Non-Insulin dependent diabetes mellitus
bpm Beats per minute PAC Polyunsaturated aromatic carbohydrates
CI. Color Index PAS Periodic acid & shifF reaction
CBC Complete blood count pC02 C02 partial pressure
CHF Chronic heart failure p02 C02 partial pressure
CT Computer tomography pm Per minute
DIC Disseminated intravascular coagualtion Ps Pulse rate
DCC Doctoral controlling committee r roentgen
DM-2 Non-Insulin dependent diabetes mellitus RBC Red blood count
DTP      1 Anti diphtheria-tetanus vaccine RDHA Reverse direct hemagglutination assay
ECG Electrocardiogram Rh Rhesus
ESR Erythrocyte sedimentation rate (R)CFT Reiter’s complement fixation test
FC Function class RIHA Reverse indirect hemagglutination assay
FAD Flavin adenine dinucleotide RNA Ribonucleic acid                            4fc Respiratory rate
FADH2 Flavin adenine dinucleotide restored RR
FEGDS Fibro-esphago-gastro-duodenoscopy SI Heart sound 1
FMNH2 Flavin mononucleotide restored S2 Heart sound 2
GIT Gastrointestinal tract TU Tuberculin unit
GMP Guanosine monophosphate U Unit
Hb Hemoglobin USI Ultrasound investigation
HbAlc Glycosylated hemoglobin V/f Vision field
Hct Hematocrit WBC White blood count
HIV Human immunodeficiency virus X-ray Roentgenogram

 

Comments
  1. Alexis says:

    I used to be suggested this blog by means of my cousin.
    I’m not positive whether or not this post is written by means of him as nobody else recognize such precise approximately my problem. You’re amazing!

    Thank you!

Leave a comment